BMAT Strategy Book 9781506237473


130 2 5MB

English Pages [207] Year 2018

Report DMCA / Copyright

DOWNLOAD PDF FILE

Table of contents :
Cover
Table of Contents
Getting Started with Your Kaplan Course
Strategy Session 1: Scoring and Triage
Lesson 1: Critical Reasoning Basics
Lesson 2: Problem Solving Basics
Lesson 3: Applied Science Basics
Lesson 4: Writing Basics
Strategy Session 2: Pacing and Practising
Lesson 5: Advanced Critical Reasoning
Lesson 6: Advanced Problem Solving
Strategy Session 3: Advanced Triage
Lesson 7: Advanced Applied Science
Lesson 8: Advanced Writing
Strategy Session 4: Preparing for Test Day
Recommend Papers

BMAT Strategy Book
 9781506237473

  • 0 0 0
  • Like this paper and download? You can publish your own PDF file online for free in a few minutes! Sign Up
File loading please wait...
Citation preview

Kaplan’s BMAT Prep course: the most complete preparation available. ®

Continue your BMAT preparation in your Kaplan Online Study Plan:





Detailed essay feedback from our Essay Review Service





Recorded video lessons to boost your revision





Access until BMAT Test Day

Strategy Book

6 full-length practice tests to build your Test Day endurance

®



BMAT



Log in to your account at www.kaptest.com/login.

BMAT Strategy Book 2018 Edition

Want more BMAT practice? Visit us online: kaptest.co.uk/BMAT ISBN: 978-1-5062-3747-3

9 781506 237473

Printed in UK ©2018 Kaplan, Inc.

Need help with this book? Contact us at [email protected]. BMAT® is a registered trademark of Cambridge Assessment, which neither sponsors nor endorses this product.

2018 Edition

NOT FOR RESALE

BMAT® is a registered trademark of Cambridge Assessment, which neither sponsors nor endorses this product.

®

BMAT Strategy Book 2018 Edition

BMAT® is a registered trademark of Cambridge Assessment.

®

Special thanks to the team who made this book possible: David Annan, Rohin Athavale, Robin Bendix-Hickman, Maxine Black, Louise Cook, Jennifer Daskevich, Joanna Graham, Brian Holmes, Katie Hunt, Elise Johnson, Piriyanga Kesavan, Bharat Krishna, Jennifer Lerner, Jeremy Miller, Walt Niedner, Mia Olorunfemi, Marianna Parker, Asha Przybyl, Teresa Rupp, Amanda Russo, Amjed Saffarini, Anna Samarina, Martha P. Torres

*BMAT® is a registered trademark of Cambridge Assessment. This publication is designed to provide accurate information in regard to the subject matter covered as of its publication date, with the understanding that knowledge and best practice constantly evolve. The publisher is not engaged in rendering medical, legal, accounting, or other professional service. If medical or legal advice or other expert assistance is required, the services of a competent professional should be sought. This publication is not intended for use in clinical practice or the delivery of medical care. To the fullest extent of the law, neither the Publisher nor the Editors assume any liability for any injury and/or damage to persons or property arising out of or related to any use of the material contained in this book. © 2018 by Kaplan, Inc. Published by Kaplan Publishing, a division of Kaplan, Inc. 750 Third Avenue New York, NY 10017 All rights reserved. The text of this publication, or any part thereof, may not be reproduced in any manner whatsoever without written permission from the publisher. This book may not be duplicated or resold, pursuant to the terms of your Kaplan Enrollment Agreement. 10 9 8 7 6 5 4 3 2 1 ISBN: 978-1-5062-3747-3

Dear Student, Congratulations on taking this vital step toward meeting the first challenge of your biomedical career. By enrolling with Kaplan, you’ve chosen to maximise your performance on the BMAT, a major factor in your application to study the biomedical sciences. Universities use the BMAT to differentiate applicants amongst a large pool of qualified hopefuls. The BMAT is your opportunity to stand out from the crowd and your performance on it enables you to distinguish yourself from other applicants. Our experience shows that the best test scores result from active engagement in the preparation process. Kaplan will give you direction and focus your training. We will teach you the specific skills the test demands, as well as ways in which you can employ the most effective test-taking strategies. We will train you to achieve peak performance on test day, but your effort is crucial. The more you invest in preparing for the BMAT, the greater the dividends you will realise on test day. You can feel confident. Kaplan has been preparing students for standardised tests for more than 75 years; this expertise has been applied to the BMAT and your Kaplan programme. We wish you the best of success in your biomedical programme. Sincerely,

Bharat Krishna Vice President–GM Kaplan Test Prep International

TA B L E O F CO N T E N T S Introduction . . . . . . . . . . . . . . . . . . . . . . . . . . . . . . . . . . . . . . . . . . . . . . . . . . . . . . . . . . . . . . . . . . . vi Orientation and Overview of Your Kaplan Programme . . . . . . . . . . . . . . . . . . . . . . . . . . . . . . . . . vii BMAT Scoring and Structure . . . . . . . . . . . . . . . . . . . . . . . . . . . . . . . . . . . . . . . . . . . . . . . . . . . . . . viii How the BMAT Is Different from School. . . . . . . . . . . . . . . . . . . . . . . . . . . . . . . . . . . . . . . . . . . . . ix Sitting the BMAT. . . . . . . . . . . . . . . . . . . . . . . . . . . . . . . . . . . . . . . . . . . . . . . . . . . . . . . . . . . . . . . . x

STRATEGY SESSION 1: SCORING AND TRIAGE BMAT Score Conversion. . . . . . . . . . . . . . . . . . . . . . . . . . . . . . . . . . . . . . . . . . . . . . . . . . . . . . . . . . 2 How Will I Score on Test Day?. . . . . . . . . . . . . . . . . . . . . . . . . . . . . . . . . . . . . . . . . . . . . . . . . . . . . 3 Standardised Testing. . . . . . . . . . . . . . . . . . . . . . . . . . . . . . . . . . . . . . . . . . . . . . . . . . . . . . . . . . . . 4 Attacking Multiple-Choice Questions. . . . . . . . . . . . . . . . . . . . . . . . . . . . . . . . . . . . . . . . . . . . . . . 5 How to Achieve Your Best Score. . . . . . . . . . . . . . . . . . . . . . . . . . . . . . . . . . . . . . . . . . . . . . . . . . . 6

LESSON 1: CRITICAL REASONING BASICS Introducing Critical Reasoning. . . . . . . . . . . . . . . . . . . . . . . . . . . . . . . . . . . . . . . . . . . . . . . . . . . . . 9 The Kaplan Method for Critical Reasoning. . . . . . . . . . . . . . . . . . . . . . . . . . . . . . . . . . . . . . . . . . . 12 Lesson 1 Quiz. . . . . . . . . . . . . . . . . . . . . . . . . . . . . . . . . . . . . . . . . . . . . . . . . . . . . . . . . . . . . . . . . . 23 The Kaplan Method for Critical Reasoning. . . . . . . . . . . . . . . . . . . . . . . . . . . . . . . . . . . . . . . . . . . 26 Attacking Critical Reasoning Questions . . . . . . . . . . . . . . . . . . . . . . . . . . . . . . . . . . . . . . . . . . . . . 26 Answers and Explanations. . . . . . . . . . . . . . . . . . . . . . . . . . . . . . . . . . . . . . . . . . . . . . . . . . . . . . . . 27

LESSON 2: PROBLEM SOLVING BASICS Introducing Problem Solving. . . . . . . . . . . . . . . . . . . . . . . . . . . . . . . . . . . . . . . . . . . . . . . . . . . . . . 33 The Kaplan Method for Problem Solving . . . . . . . . . . . . . . . . . . . . . . . . . . . . . . . . . . . . . . . . . . . . 36 Lesson 2 Quiz. . . . . . . . . . . . . . . . . . . . . . . . . . . . . . . . . . . . . . . . . . . . . . . . . . . . . . . . . . . . . . . . . . 47 The Kaplan Method for Problem Solving . . . . . . . . . . . . . . . . . . . . . . . . . . . . . . . . . . . . . . . . . . . . 51 Attacking Problem Solving Questions. . . . . . . . . . . . . . . . . . . . . . . . . . . . . . . . . . . . . . . . . . . . . . . 51 Answers and Explanations. . . . . . . . . . . . . . . . . . . . . . . . . . . . . . . . . . . . . . . . . . . . . . . . . . . . . . . . 52

LESSON 3: APPLIED SCIENCE BASICS Introducing Applied Science. . . . . . . . . . . . . . . . . . . . . . . . . . . . . . . . . . . . . . . . . . . . . . . . . . . . . . 59 The Kaplan Method for Applied Science . . . . . . . . . . . . . . . . . . . . . . . . . . . . . . . . . . . . . . . . . . . . 62 Lesson 3 Quiz. . . . . . . . . . . . . . . . . . . . . . . . . . . . . . . . . . . . . . . . . . . . . . . . . . . . . . . . . . . . . . . . . . 73 The Kaplan Method for Applied Science . . . . . . . . . . . . . . . . . . . . . . . . . . . . . . . . . . . . . . . . . . . . 76 Attacking Applied Science Questions. . . . . . . . . . . . . . . . . . . . . . . . . . . . . . . . . . . . . . . . . . . . . . . 76 Answers and Explanations. . . . . . . . . . . . . . . . . . . . . . . . . . . . . . . . . . . . . . . . . . . . . . . . . . . . . . . . 77

LESSON 4: WRITING BASICS Introducing Writing. . . . . . . . . . . . . . . . . . . . . . . . . . . . . . . . . . . . . . . . . . . . . . . . . . . . . . . . . . . . . . 83 Scoring . . . . . . . . . . . . . . . . . . . . . . . . . . . . . . . . . . . . . . . . . . . . . . . . . . . . . . . . . . . . . . . . . . . . . . . 85 The Kaplan Method for Writing. . . . . . . . . . . . . . . . . . . . . . . . . . . . . . . . . . . . . . . . . . . . . . . . . . . . 86 Application . . . . . . . . . . . . . . . . . . . . . . . . . . . . . . . . . . . . . . . . . . . . . . . . . . . . . . . . . . . . . . . . . . . . 92 The Kaplan Method for Writing. . . . . . . . . . . . . . . . . . . . . . . . . . . . . . . . . . . . . . . . . . . . . . . . . . . . 95

iv

STRATEGY SESSION 2: PACING AND PRACTISING Pacing . . . . . . . . . . . . . . . . . . . . . . . . . . . . . . . . . . . . . . . . . . . . . . . . . . . . . . . . . . . . . . . . . . . . . . . . 99 Section Management. . . . . . . . . . . . . . . . . . . . . . . . . . . . . . . . . . . . . . . . . . . . . . . . . . . . . . . . . . . . 100 Practising With Paper-Based Tests. . . . . . . . . . . . . . . . . . . . . . . . . . . . . . . . . . . . . . . . . . . . . . . . . 101

LESSON 5: ADVANCED CRITICAL REASONING Kaplan Method for Critical Reasoning . . . . . . . . . . . . . . . . . . . . . . . . . . . . . . . . . . . . . . . . . . . . . . Lesson 5 Quiz. . . . . . . . . . . . . . . . . . . . . . . . . . . . . . . . . . . . . . . . . . . . . . . . . . . . . . . . . . . . . . . . . . Next Steps. . . . . . . . . . . . . . . . . . . . . . . . . . . . . . . . . . . . . . . . . . . . . . . . . . . . . . . . . . . . . . . . . . . . . Answers and Explanations. . . . . . . . . . . . . . . . . . . . . . . . . . . . . . . . . . . . . . . . . . . . . . . . . . . . . . . .

105 117 120 121

LESSON 6: ADVANCED PROBLEM SOLVING Kaplan Method for Problem Solving. . . . . . . . . . . . . . . . . . . . . . . . . . . . . . . . . . . . . . . . . . . . . . . . Lesson 6 Quiz. . . . . . . . . . . . . . . . . . . . . . . . . . . . . . . . . . . . . . . . . . . . . . . . . . . . . . . . . . . . . . . . . . Next Steps. . . . . . . . . . . . . . . . . . . . . . . . . . . . . . . . . . . . . . . . . . . . . . . . . . . . . . . . . . . . . . . . . . . . . Answers and Explanations. . . . . . . . . . . . . . . . . . . . . . . . . . . . . . . . . . . . . . . . . . . . . . . . . . . . . . . .

127 137 140 141

STRATEGY SESSION 3: ADVANCED TRIAGE Triage Considerations. . . . . . . . . . . . . . . . . . . . . . . . . . . . . . . . . . . . . . . . . . . . . . . . . . . . . . . . . . . . 147 Kaplan Triage Strategy for BMAT Section 1 and Section 2. . . . . . . . . . . . . . . . . . . . . . . . . . . . . . 148

LESSON 7: ADVANCED APPLIED SCIENCE Kaplan Method for Applied Science. . . . . . . . . . . . . . . . . . . . . . . . . . . . . . . . . . . . . . . . . . . . . . . . Lesson 7 Quiz. . . . . . . . . . . . . . . . . . . . . . . . . . . . . . . . . . . . . . . . . . . . . . . . . . . . . . . . . . . . . . . . . . Next Steps. . . . . . . . . . . . . . . . . . . . . . . . . . . . . . . . . . . . . . . . . . . . . . . . . . . . . . . . . . . . . . . . . . . . . Answers and Explanations. . . . . . . . . . . . . . . . . . . . . . . . . . . . . . . . . . . . . . . . . . . . . . . . . . . . . . . .

151 165 168 169

LESSON 8: ADVANCED WRITING Writing Refresher . . . . . . . . . . . . . . . . . . . . . . . . . . . . . . . . . . . . . . . . . . . . . . . . . . . . . . . . . . . . . . . Idea Bank . . . . . . . . . . . . . . . . . . . . . . . . . . . . . . . . . . . . . . . . . . . . . . . . . . . . . . . . . . . . . . . . . . . . . How to Practise. . . . . . . . . . . . . . . . . . . . . . . . . . . . . . . . . . . . . . . . . . . . . . . . . . . . . . . . . . . . . . . . . Review Exercise . . . . . . . . . . . . . . . . . . . . . . . . . . . . . . . . . . . . . . . . . . . . . . . . . . . . . . . . . . . . . . . . A Few Words from the Experts. . . . . . . . . . . . . . . . . . . . . . . . . . . . . . . . . . . . . . . . . . . . . . . . . . . . Next Steps. . . . . . . . . . . . . . . . . . . . . . . . . . . . . . . . . . . . . . . . . . . . . . . . . . . . . . . . . . . . . . . . . . . . .

175 178 179 180 183 183

STRATEGY SESSION 4: PREPARING FOR TEST DAY Knowing Test Day Is Key. . . . . . . . . . . . . . . . . . . . . . . . . . . . . . . . . . . . . . . . . . . . . . . . . . . . . . . . . . Stress Management. . . . . . . . . . . . . . . . . . . . . . . . . . . . . . . . . . . . . . . . . . . . . . . . . . . . . . . . . . . . . What to Do Until a Week Before the Test. . . . . . . . . . . . . . . . . . . . . . . . . . . . . . . . . . . . . . . . . . . . What to Do a Week Before the Test . . . . . . . . . . . . . . . . . . . . . . . . . . . . . . . . . . . . . . . . . . . . . . . . Test Day. . . . . . . . . . . . . . . . . . . . . . . . . . . . . . . . . . . . . . . . . . . . . . . . . . . . . . . . . . . . . . . . . . . . . . . After the Test. . . . . . . . . . . . . . . . . . . . . . . . . . . . . . . . . . . . . . . . . . . . . . . . . . . . . . . . . . . . . . . . . . .

187 188 190 191 192 193

v

BMAT Strategy Book

GETTING STARTED WITH YOUR KAPLAN BMAT COURSE Welcome to Kaplan! Congratulations on taking this vital step towards meeting the first challenge of your medical career. By enrolling with Kaplan, you’ve chosen to maximise your performance on the BMAT. The BMAT, like all standardised tests, is vulnerable to test-taking strategies. On a standardised test, it is not ‘getting the question correct’ that earns a top score; rather, it is getting the questions correct in the shortest time possible that yields the best scores. The BMAT is not testing your knowledge; it’s testing your ability to answer a variety of questions accurately and quickly. Thus, you must have a methodical approach to each question type, and arm yourself with a mental ‘toolbox’ of strategies so you are best prepared to achieve the greatest possible speed and accuracy on Test Day.

How to Use Your BMAT Strategy Book The lessons in this Strategy Book are designed to accompany the recorded video lessons in your BMAT Online Study Plan. Self-Study students will complete the course using these video lessons, following the instructions below. Live Online students may use the video lessons to review course content after completing their Live Online course. Each lesson consists of a series of exercises, which you will complete in this book as you follow along with the corresponding video lessons. Do not start a question or a set until the video instructor tells you to do so. If the video instructor tells you to keep to strict timing for a question or a set, please follow that timing. Each lesson is filled with timed practice at every opportunity, so that you can refine your speed and accuracy throughout your BMAT course. There are also a series of Strategy Sessions, covering the fundamentals of BMAT scoring and the Kaplan BMAT triage strategy, along with tips for pacing, practising and preparing for Test Day. You should start with Strategy Session 1, after which you may complete the lessons and strategy sessions in any order. In your BMAT Online Study Plan, each lesson or strategy session has been divided into short videos; most videos are 5 to 7 minutes in length. Each lesson will include a list of videos, so you can cover a particular topic by watching just one or two videos, or you can complete a lesson by working through all the videos in the list. Whilst you may complete the lessons in any order, we ask that you complete the videos within each lesson in the prescribed order. Many of the topics within each lesson will build on tips and strategies covered in earlier videos within the lesson, which may become unclear if the order is not followed.

How to Use Your BMAT Online Study Plan Your BMAT Online Study Plan includes your Kaplan BMAT video lessons and the webgrids for your full-length Kaplan BMAT practice tests. You will take each BMAT practice test using answer sheets in the Big Book of BMAT Practice Tests; be sure to time yourself strictly, to simulate the Test Day experience. Self-Study students may also print the test booklets and answer sheets from your Online Study Plan. When you finish a BMAT practice test, enter the answers into the corresponding webgrid in your Online Study Plan and submit your essay. You will then receive a score report analysing your performance, which will also allow you to access the correct answers and explanations. Be sure to review the worked answers for every question, including those you got right as well as those you got wrong, to make sure you are selecting the right answers for the right reasons and to ensure you understand why every incorrect answer choice is wrong. It may take up to a week to receive your score for the essay, which will then appear in your score reports in your Online Study Plan. There will be a deadline for submitting essays to be marked, normally in the second half of October; check with your BMAT teacher for the exact date. To access your BMAT Online Study Plan, log in at www.kapteststudy.com with the username and password that you received by email. If you cannot find this login email, double-check your spam folder; if it’s not there, then contact us at [email protected] for your username and password.

vi

BMAT Strategy Book

ORIENTATION AND OVERVIEW OF YOUR KAPLAN PROGRAMME Lessons and Strategy Sessions The teacher-led experience is the foundation of Kaplan’s BMAT programme. Our lessons will train you on the methods and strategies required to master each section of the BMAT. As you work through BMAT practice questions with your instructor, you will learn how to recognise common wrong answer choices and how to work quickly and accurately, using the critical thinking skills necessary to analyse questions and to master the BMAT. The first day of the course includes a lesson that teaches the basic skills and strategies required in each of four fundamental question types on the BMAT. The second day of the course builds on these basic skills, with an advanced lesson for each of these question types, designed to sharpen your approach and accuracy with more difficult and time-consuming questions. Each lesson ends with a quiz, giving you a chance to use the skills and strategies you have learned, whilst also practising for pacing, which is essential for ensuring you answer all the questions in each section. Depending on timing, your instructor may assign the quizzes for homework. The eight lessons are supplemented by a series of strategy sessions that cover the essentials of BMAT scoring, pacing, practising and preparing for Test Day—the fundamentals that will help you achieve your best score.

BMAT Classroom Course Schedule DAY 1

DAY 2

Welcome and Introduction Strategy Session 1: Scoring and Triage Lesson 1: Critical Reasoning Basics Short Break Lesson 2: Problem Solving Basics Break for Lunch Lesson 3: Applied Science Basics Short Break Lesson 4: Writing Basics Wrap-Up and Preview Tomorrow

Welcome Back Strategy Session 2: Pacing and Practising Lesson 5: Advanced Critical Reasoning Short Break Lesson 6: Advanced Problem Solving Break for Lunch Strategy Session 3: Advanced Triage Lesson 7: Advanced Applied Science Short Break Lesson 8: Advanced Writing Strategy Session 4: Preparing for Test Day

After Your Course Lessons After finishing your course lessons, you will complete six full-length BMAT practice tests in the same paper-andpencil format you will encounter on Test Day. When sitting a Kaplan BMAT practice test, you will submit your essay at the end of each test in the Online Study Plan. The option to submit an essay will appear after you finish Section 2. NB This is the only way to submit an essay to be marked. Please note: the deadline for submitting essays is 3pm BST on Friday 26th October. Essays received after the deadline will not be marked. Please allow time to read and reflect on the feedback on each essay before writing and submitting further essays. If you are sitting the BMAT in September, the deadline for submitting essays is 3pm BST on Monday 27th August. Essays received by this deadline will be marked in time for you to review the feedback before the September test date. Essays received after this deadline may not be reviewed before the September test date. You should also allow time to sit the official BMAT practice tests, which are downloadable from the test-maker’s website at www.bmat.org.uk. These do not include worked answers, but try to work out the reasoning for the questions you answered incorrectly.

vii

BMAT Strategy Book

BMAT SCORING AND STRUCTURE The BMAT has been designed to assess your cognitive powers and other attributes considered to be valuable for health care professionals. The BMAT is also meant to be challenging for students applying to the top medical and veterinary schools in the UK, and, indeed, in the world. There are three sections on the BMAT; use of a calculator is not allowed.

Test Section and What Is Tested

Number and Format of Questions

Aptitude and Skills:

35 total:

Three types of questions:

·· 13 Problem Solving ·· 10 Understanding Argument ·· 12 Data Analysis and Inference

·· Understanding Argument questions present a series of logical arguments and require you to identify conclusion, evidence and assumptions, detect flaws and draw conclusions.

Time Allotted

Score

60 minutes

1.0-9.0 (scaled)

30 minutes

1.0-9.0 (scaled)

30 minutes

0-5 (for quality of content); A-E (for quality of written English)

·· Problem Solving questions present numerical information and require you to select the relevant figures and solve problems, using simple arithmetic and algebraic operations. ·· Data Analysis and Inference draws on your ability to understand and process data, interpret data and draw appropriate conclusions. Scientific Knowledge and Applications:

27 total:

These questions require you to apply your GCSE science and maths knowledge in a new format.

·· 6-8 Biology ·· 6-8 Chemistry ·· 6-8 Physics ·· 5-7 Maths

Writing Task:

1 essay, from a choice of 3 tasks

You must select one task from a set of four. Most essay tasks involve topics from science or medicine; you must write an essay on one side of A4 that explains a proposition, suggests a counterproposition and suggests a possible resolution of the two sides.

BMAT TEST TIP Data Analysis and Inference questions require you to use the same skills as either an Understanding Argument question, or a Problem Solving question, or both. The only difference is that these questions include data in the form of information, graphs or statistics.

viii

BMAT Strategy Book

HOW THE BMAT IS DIFFERENT FROM SCHOOL If you approach the BMAT in the same fashion as exams you take in school, you are very likely to be dissatisfied with your result. This is because the BMAT is different from school, in a number of fundamental ways. On the BMAT, timing is everything. You cannot succeed on the BMAT without managing time effectively. Throughout each section of the test, you must make strategic decisions about how to spend your time and effort on the activities and questions that will maximise your marks. The time management skills required by the BMAT are not generally required on other exams, which are generally not designed to be difficult to finish in the time allowed. The BMAT does not replicate GCSE/AS grades or coursework. The BMAT forms a part of your application, in addition to your GCSE/AS grades, predicted A2s, work experience, personal statement, interview, and so forth. The test adds something more to your application beyond grades already achieved by adding something different. The BMAT cohort is a different, and highly competitive, peer group. Students applying to medical and veterinary programmes at top universities are usually among the top students in their year at school. The BMAT cohort consists entirely of these top students. Thus, the BMAT is designed to differentiate among this cohort of extremely bright and capable students. The test is meant to be challenging for top students. ‘Active’ v ‘Passive’ The BMAT requires an ‘active’ approach to test-taking: you must analyse questions, identify key topics and knowledge, predict strategically, and work with multiple-choice answers. Most other exams, such as GCSEs and A-levels, fit with a more ‘passive’ approach to test-taking: the problems are set clearly, information is provided helpfully, content is subject/module-based, and the format is short-answer. ‘Problem-solving’ v ‘Curriculum’ Perhaps the fundamental difference between the BMAT and school exams. On the BMAT, you must continually identify what you need to solve each question. On GCSEs and A-levels, you must demonstrate what you have learnt. The BMAT tests problem-solving skills and aptitude, rather than knowledge. Added stress factors. You are not allowed a calculator on the BMAT. There are many ‘oddball’ questions, in formats you may not have previously encountered. The time pressure of the BMAT is the ultimate challenge, as you have less than 2 minutes to answer most of the questions. This is very different from school, and more stressful.

ix

BMAT Strategy Book

SITTING THE BMAT You must sign up for the BMAT directly with your school, or with a BMAT open centre. You may find a list of schools operating as open centres on the BMAT website at www.bmat.org.uk. NB Space at open centres is often limited, so it’s advisable to register for the test as early as possible once registration opens. Download and sit the free BMAT practice tests. The BMAT test-maker provides all the past papers on their website, along with shorter-form specimen papers, all of which are free to download. You should therefore allow time to sit and review all these official practice tests before Test Day, as they will provide essential further practice for pacing, while sharpening your BMAT skills and strategies so they are fully optimised on Test Day. NB The test format and National Curriculum have changed over the years, so there is less value in practising with past papers from before 2009.

Key Dates for the 2018 BMAT BMAT—September

BMAT—October

BMAT registration opens

25 June 2018

1 September 2018

Standard entry closing date

12 August 2018

1 October 2018

Late entry closing date



15 October 2018

BMAT Test Day

1 September 2018

31 October 2018

BMAT results released

21 September 2018

23 November 2018

BMAT Content Section 2 of the BMAT requires familiarity with the concepts, terms and knowledge specified by GCSE-level double-award science and mathematics. The BMAT also tests certain skills including level 3 national framework curriculum key skills. These include the ability to read and follow instructions in formal English, good pacing and accuracy, mental computation and simple graphical analysis. The BMAT also includes the analysis of text and its conclusion, evidence and assumption, and making inferences and deductions. The BMAT also includes a written section that is intended to test the ability to communicate knowledge in a clear and concise written format.

BMAT TEST TIP The BMAT Assumed Subject Knowledge Guide will be an invaluable resource as you prepare for Test Day. This guide is provided as as a read-only ebook; you cannot download it or print it out. Be sure to register and use the Assumed Subject Knowledge Guide as you revise, but note that it is very broad—it includes many subjects that have never appeared on the BMAT. Kaplan recommends using the subjects in the BMAT past papers to drive your revision, as you will notice that the same few topics are tested again and again and again.

x

S T R AT EG Y SE S SI O N 1

Scoring and Triage LEARNING OBJECTIVES By the end of this session, you will be able to:

• Explain how the BMAT is scored. • Identify the different types of questions contained in each section of the BMAT. • Specify different approaches to multiple-choice questions.

Strategy Session 1 Scoring and Triage

BMAT SCORE CONVERSION Your BMAT results for Section 1 and Section 2 correspond to the number of correct answers given in each section. Each correct answer is worth one mark; your total for each section is converted to a scaled score from 1.0 to 9.0. Use the scoring tables below to convert your correct answers on your Kaplan practice tests into BMAT scores. SECTION 1

SECTION 2

Number Correct

BMAT Score

Number Correct

BMAT Score

0 1 2 3 4 5 6 7 8 9 10 11 12 13 14 15 16 17 18 19 20 21 22 23 24 25 26 27 28 29 30 31 32 33 34 35

1.0 1.0 1.0 1.0 1.0 1.6 2.0 2.4 2.7 3.0 3.3 3.5 3.8 4.0 4.3 4.5 4.7 4.9 5.2 5.4 5.6 5.9 6.1 6.3 6.6 6.9 7.1 7.4 7.8 8.1 8.5 9.0 9.0 9.0 9.0 9.0

0 1 2 3 4 5 6 7 8 9 10 11 12 13 14 15 16 17 18 19 20 21 22 23 24 25 26 27

1.0 1.0 1.6 2.1 2.5 2.9 3.2 3.4 3.7 3.9 4.1 4.3 4.5 4.7 4.9 5.1 5.3 5.5 5.7 6.0 6.2 6.5 6.7 7.1 7.5 8.0 8.8 9.0

Section 3 (the Writing Task) is assessed according to a scoring rubric that accounts for the content of your argument, as well as your use of English. Scoring of the essay will be discussed in more detail in Lesson 4.

2

Strategy Session 1 Scoring and Triage

HOW WILL I SCORE ON TEST DAY? The score conversion tables for BMAT Sections 1 and 2 vary slightly from year to year, but their fundamentals are remarkably consistent. The conversion tables on the previous page are the same as those used on last year’s BMAT. Here’s some further information to put your BMAT score in context: ·· The average score in both Section 1 and Section 2 is usually just above or below 5.0, and most testtakers will score relatively close to the average. ·· To score 5.0, you have to answer half of the questions in the section correctly. Thus, answering half of the questions correctly corresponds to an average score. ·· Approximately 10% of test-takers score 7.0 or above in Section 1, and a similar figure score 7.0 or above in Section 2. ·· To score 7.0, you have to answer approximately 80% of the questions in the section correctly. Thus, you can achieve an excellent mark in Section 1 or Section 2 by answering 80% of the questions correctly. ·· Scores of 9.0 in Section 1 or Section 2 are rare, indeed. NB, however, that you can get a ‘perfect’ score of 9.0 even if you miss a few questions. ·· In Section 1, you can get 3 questions wrong and still achieve a perfect score. In Section 2, you can get 1 question wrong and still achieve a perfect score. ·· Each question is worth one mark. You don’t get extra marks for tougher, more time-consuming questions. ·· There is no penalty for wrong answers. Here are the takeaways, as used by past students who achieved excellent BMAT scores: ·· You will maximise your score by marking as many correct answers as possible on your answer sheets. ·· You will earn a score of 7.0 in Section 1 and Section 2 by correctly answering 80% of the questions in each section. Doing so will put you in the top 10% of BMAT test-takers. ·· You should mark an answer for every question—but don’t waste time on the few, very time-consuming questions. Based on the score conversion table, how many questions must you answer correctly in order to score a 7.0? How many more questions would you need to answer correctly to score an 8.0? How many more than that for a 9.0?

Section

No. of Questions

Aptitude and Skills

35

Scientific Knowledge and Applications

27

No. Correct to Score 7.0

How Many More for 8.0?

How Many More for 9.0?

BMAT TEST TIP Attack the BMAT with the goal of maximising your marks in each section. Most students will target a score of 7.0 in Sections 1 and 2. If you find the BMAT easy, then target a score of 8.0 or even 9.0.

3

Strategy Session 1 Scoring and Triage

STANDARDISED TESTING What makes a test ‘standardised’? ·· A ‘standardised’ test is one that presents the same types of questions in the same format for all test-takers. ·· Standardised tests ‘normalise’ the applicant pool, so universities have a fair basis of comparison. ·· Even though test-takers might sit different versions of the test paper, their results are comparable due to the fact that the test is standardised. ·· Standardised tests feature a limited range of tasks and question types in each scored section. ·· Most standardised tests include only questions that are multiple-choice. ·· Writing tasks can also be standardised, so long as all the different essay titles require the same basic approach to structuring a successfully argued essay.

What makes the BMAT ‘standardised’? ·· All questions in Section 1 are either Critical Reasoning or Problem Solving, with the same few basic types of each appearing again and again. ·· All biology and chemistry questions in Section 2 involve one of a few basic question formats. ·· All physics and maths questions in Section 2 involve one of a few basic areas of GCSE content. ·· All essay titles in Section 3 require the same basic approach to structuring a successful essay. ·· Nearly all questions are multiple-choice.

What is the advantage in understanding that the BMAT is a standardised test? ·· You can learn and practise the tasks and question types in each section. ·· You are given the correct answer with every scored question. ·· You can use elimination strategies to ‘zero in’ on the correct answer. ·· You can limit your GCSE revision to the most high-value content for Section 2. ·· You can prepare a systematic approach for every question, and for every section.

What about the claims that you cannot revise for a test like the BMAT? ·· You cannot ‘revise’, in the way you would for A-levels, as the BMAT is not a content-based test. ·· Most tasks and question types require skills that are not commonly used in school. ·· Students who learn and practise these tasks and question types have an advantage on the BMAT. ·· Practising for speed and accuracy gives a further advantage on Test Day. ·· This is not ‘revision’ as commonly understood, but students find it helps in getting higher scores on Test Day.

BMAT TEST TIP Section 2 questions are based on GCSE content, but they are not GCSE questions. Practising for speed and accuracy in answering Section 2 questions is more important than revising your GCSE science notes, though some GCSE revision will likely be helpful.

4

Strategy Session 1 Scoring and Triage

ATTACKING MULTIPLE-CHOICE QUESTIONS One answer is correct; the rest are wrong. ·· Each multiple-choice question has one correct answer. ·· Wrong answers can be wrong for any number of reasons. ·· There are no ‘pretty good’ answers, or ‘close’ answers, or ‘nearly right’ answers. ·· Standardised tests include ‘trap’ wrong answers to lead students down the wrong path. ·· Understanding the task/question type will help you find the correct answers quickly and accurately.

Resist the temptation to rationalise. ·· Intelligent students are prone to rationalising answers. ·· Test-makers use your intelligence to try to catch you out with wrong answer traps. ·· Tough wrong answers separate the ‘wheat from the chaff’—students that overrationalise get lower scores. ·· Don’t get hung up looking for other ways a wrong answer could be correct. You don’t have time. ·· Students who do not take extra steps to rationalise answers will answer faster, and score higher.

Three ways to get the correct answer ·· Work it out (straightforward maths/reasoning) ·· Eliminate the wrong answers. ·· Guess. ·· The BMAT examiners will not know which approach you took to get the correct answer.

Odds of Guessing Correctly Odds of Correctly Guessing After Eliminating . . . No. of Answer Choices

Odds of Correctly Guessing Blindly

1 answer

2 answers

3 answers

4

25%

33%

50%

100%

5

20%

25%

33%

50%

100%

6

17%

20%

25%

33%

50%

4 answers

5 answers

100%

KAPLAN TIPS FOR MULTIPLE-CHOICE QUESTIONS ·· Eliminate as many answer choices as possible ·· If you cannot determine the right answer, guess from the remaining choices ·· Pick a consistent answer choice for guessing (eg, always guess B, or always guess C). Pick a second favourite guessing choice, for those times when you can eliminate your first choice as incorrect.

5

Strategy Session 1 Scoring and Triage

HOW TO ACHIEVE YOUR BEST SCORE Set up a weekly schedule and stick to it. There’s room for flexibility, but we’ve found that students who follow a disciplined preparation schedule are more confident on Test Day. Always use Kaplan’s methods and strategies when you practise. Don’t wait until Test Day to try a strategic approach. Know which strategies yield the greatest results for your performance before Test Day. Know your strengths and weaknesses. Early in your BMAT preparation, focus on your weaknesses. Your focus should be qualitative. It’s not important whether you get one question right, or 63 questions right. At the outset, what matters is determining whether you are approaching the BMAT in the most strategic and productive way. As Test Day approaches, you should spend more of your practice time on your strengths. They will earn you points. You cannot afford to neglect your strongest skills. The only results that count are the ones on Test Day. Skill is acquired through patient effort over time. During your practice you should always concentrate on process, not performance. Getting a question right today is not the most important thing. Here’s what matters: Can you get a similar question right on Test Day? Did you employ a method that you can replicate on Test Day? On the BMAT, timing is everything. It’s essential that you learn the methods and strategies, so they become second nature by Test Day. As you prepare, you must practise for speed as well as accuracy. On a standardised test like the BMAT, it’s not ‘getting the question right’ that gives you high marks; rather, it is getting the question right in the shortest amount of time. Thus, you must prepare to answer quickly and accurately on Test Day. Your five Kaplan BMAT practice tests and the official past papers from the BMAT website will help you practise for pacing.

BMAT SKILLS IN PRACTICE As part of your medical training, you are volunteering several hours a week as an ambulance paramedic. One Saturday night, your crew responds to a car crash in the city centre. The motorist appears to have lost control of his vehicle, which ploughed through a group of young women on a hen do before colliding with a parked car. There are numerous casualties, and you and your crew are seeing to the motorist. You notice that the steering wheel in his car is deformed. On close inspection of the motorist, you smell alcohol on his breath. He keeps going in and out of consciousness, so he cannot answer your questions about where it hurts. He is not able to remain alert for more than a few seconds at a time. Your supervisor on the ambulance crew asks, “Stay and play, or scoop and run?” What does it mean to ‘stay and play’? What does it mean to ‘scoop and run’? Is one course of action necessarily better than the other? What are the implications for the BMAT? What is the third option on the BMAT?

6

LESSON 1

Critical Reasoning Basics LEARNING OBJECTIVES By the end of this session, you will be able to:

• Enumerate the eight types of critical reasoning questions. • Find the components of an argument: Conclusion, Evidence and Assumption. • Use the Kaplan Method for Critical Reasoning to answer Conclusion, Assumption, Flaw and Method of Argument questions.

Lesson 1 Critical Reasoning Basics

INTRODUCING CRITICAL REASONING ·· One-third to one-half of questions in BMAT Section 1 are Critical Reasoning questions. ·· These questions test your ability to analyse arguments and make valid inferences.

Critical Reasoning Basics ·· Approximately 15 to 17 questions (out of 35) ·· Appear throughout the section, including on multi-question sets ·· Include a long paragraph, which usually consists of an argument ·· Most have 4 or 5 answer choices; some will include numbered statements.

Warm-Up Exercise ·· The next page contains two Critical Reasoning questions. ·· Try to complete the exercise, and answer both questions, in 3 minutes.

9

Lesson 1 Critical Reasoning Basics

Scientists at a British university sampled hundreds of cups of tea to determine the exact conditions under which tea tastes the best. They recommend brewing the tea for 2 minutes before adding milk, though their position on the age-old question as to whether to add the milk before or after brewing the tea is not the most surprising or significant result of the scientists’ research. Rather, it’s their finding about the optimum temperature at which tea should be consumed, to deliver optimum taste. The scientists determined that tea tastes best if left to sit for 6 minutes after brewing, and should be drunk within 17 minutes after brewing. This is because tea starts brewing at 100C, but cools to 60C after 6 minutes, and to 45C after 17 minutes; by the time it reaches this tepid temperature, tea will have lost its distinctive, uplifting flavour. 1

Which one of the following is the best statement of the conclusion of the argument above? A

You should not drink tea immediately after brewing it.

B

You should not drink tea more than 15 minutes after brewing it.

C

You should brew tea for 2 minutes, then add milk, to achieve optimum taste.

D

Tea tastes best between temperatures of 45C and 60C

E

Tea loses its special, refreshing flavour 17 minutes after brewing.

Foxes are known to be shy creatures that normally avoid contact with humans. However, there have been several recent cases of foxes attacking small children. One fox entered a house in east London through an open window and attacked nine-month-old twins; another bit a three-year-old who had tried to touch its tail at a Brighton playgroup. Since the only part of the country where these attacks have taken place is in the South-East of England, one can conclude that foxes in the South-East are no longer shy, and pose a great danger to children. 2

Which of the following indicate flaws in the above argument?

1 The possibility that the foxes were frightened when they attacked the children is overlooked.



2 Two foxes are assumed to be representative of all foxes in the South-East.



3 Foxes in other parts of the country are not considered.

A

1 and 2

B

1 and 3

C

2 and 3

D

1, 2 and 3

E None

STOP.  IF YOU FINISH BEFORE TIME IS UP, CHECK ANY QUESTIONS YOU HAVE MARKED FOR REVIEW. YOU MAY GO BACK TO QUESTIONS ON THIS PAGE ONLY.

10

Lesson 1 Critical Reasoning Basics

Critical Reasoning Challenges ·· Very difficult to read the argument and answer choices in 90 seconds ·· Reasoning skills involved are very specific, and not taught in school ·· Wrong answer choices can be very confusing and time-consuming ·· Unprepared test-takers may have to read the argument 2 or 3 times

Attacking Critical Reasoning Questions ·· There are eight types of Critical Reasoning questions. ∘∘ ∘∘ ∘∘ ∘∘ ∘∘ ∘∘ ∘∘ ∘∘

Conclusion Assumption Flaw Method of Argument Weaken Strengthen Inference Explain

·· Each question type requires a slightly different approach to attacking the argument. ∘∘ Four question types require you to find conclusion, evidence and assumption. ∘∘ One requires you to find the conclusion. ∘∘ One requires you to find the answer that makes a correct inference, based on the contents of the argument. ·· Some arguments will use Formal Logic—an advanced skill requiring some basic notation. ·· The skills required to attack these questions can be learnt, and improved for speed. ·· With these skills, you can answer most Critical Reasoning questions in 60 seconds.

BMAT SKILLS IN PRACTICE A new mum comes to see you, her GP, for her baby’s 8-week check-up. Her baby is bright and active, and has a completely normal examination. Near the end of the consultation, you mention that the baby will be due for upcoming appointments. The baby’s first set of injections will include the jab for the measles, mumps and rubella (MMR) vaccine. The mum says, ‘Oh, Doctor, I’m not sure we’ll be wanting the MMR jab. I’ve heard that they’ve done studies in the scientific journals, and there may be a link between the MMR jab and autism. We don’t want to put our healthy baby at risk for autism, so I don’t think the MMR jab is a very good idea.’ What is the mother’s main concern? Why does she think this? How would you, as the doctor, respond?

BMAT TEST TIP Most test-takers make the same mistake on every Critical Reasoning question: they read the argument before reading the question, which means they have to re-read the argument. You will save valuable time by ALWAYS reading the question before you read the argument.

11

Lesson 1 Critical Reasoning Basics

THE KAPLAN METHOD FOR CRITICAL REASONING STEP 1: STEP 2: STEP 3:

Keywords Keywords are sign-posts to an argument’s structure. They highlight the bits that are essential to its main point. Consider these two details:

Dalia got a great BMAT score.



Dalia will attend Wessex Medical School.

The meaning depends on the keyword linking the details.

Dalia got a great BMAT score. Therefore, she will attend Wessex Medical School



Dalia got a great BMAT score. Nevertheless, she will attend Wessex Medical School.

There are three categories of keywords that will help make quick work of BMAT Critical Reasoning: ·· Conclusion keywords: ·· Evidence keywords: ·· Contrast keywords:

BMAT TEST TIP Be very careful with Contrast keywords. They may indicate a contrast that leads to the conclusion, or they may indicate a contrast within the evidence.

12

Lesson 1 Critical Reasoning Basics

Conclusion, Evidence and Assumptions CONCLUSION ·· The conclusion is the main point of the author’s argument. ·· Ways to identify the conclusion: ∘∘ Conclusion keywords or phrases ∘∘ The One-Sentence Test ∘∘ Fact vs. Opinion Find the conclusion in the following argument:

The Second World War had a profound effect on the growth of businesses. The Absolute Packaging Company turned a profit of only £10,000 in the year before the war, yet by 1948 was earning nearly 10 times that figure.

EVIDENCE ·· Evidence is provided to support the conclusion ·· Anything stated in the argument that is not the conclusion is considered evidence ·· Data, in the form of statistics, surveys or historical facts, are typically evidence ·· Sometimes the evidence is merely a conjecture Find the conclusion, and then the evidence, in the following argument:

Recent surveys show that many people who have emigrated from Britain to other countries have learnt a foreign language. Clearly, learning a foreign language is likely to cause emigration from Britain.

ASSUMPTIONS ·· The assumption is the unstated link between the evidence and the conclusion ·· There is always a gap between the terms of the evidence and the terms of the conclusion ·· Without the assumption, the argument’s conclusion could not follow from its evidence ·· Evidence + Assumption → Conclusion

At the start of the last school year, a local educational authority implemented a new scheme designed to reduce the incidence of teen pregnancy. Unfortunately, the scheme failed to produce the desired result. If the scheme had succeeded, the dropout rate for female students would not have increased substantially in the last school year.

BMAT TEST TIP An argument’s assumption bridges the gap between the terms of the evidence and the terms of the conclusion. Connect the mismatched terms to find the assumption.

13

Lesson 1 Critical Reasoning Basics

Conclusion Questions Conclusion questions are the quickest of the Critical Reasoning question types. ·· You need simply find the conclusion, and then find the answer choice that matches. ·· Use keywords to help find the conclusion. ·· If the conclusion is tough to spot, then look for one of the following: ∘∘ The author’s main point ∘∘ The most strongly-stated opinion ∘∘ The one sentence in the argument that is most essential ·· Always identify the conclusion in the argument before looking at the answers.

GP surgeries do not keep hours that are convenient for most people, and as such it can be difficult to get an appointment. Too many people find themselves foregoing treatment, or self-prescribing ‘home remedies’ rather than taking medical advice. The NHS can do better. 3

Which one of the following best expresses the main conclusion of the argument above? A

More people would attend GP surgeries if they kept more convenient hours.

B

The NHS should make it easier to see a doctor.

C

The NHS should discourage home remedies.

D

Doctors should keep more convenient hours.

E

Fewer people would take home remedies if GPs worked different hours.

BMAT TEST TIP A scope shift occurs whenever an argument changes the subject, even slightly. The correct answer will usually be based on the scope shift.

14

Lesson 1 Critical Reasoning Basics

Astronomers have confirmed the existence of a distant planet, whose mass is likely to consist mostly of water. The planet is very similar to Earth, in that it orbits a nearby star, though it has 7 times Earth’s mass, and 2.7 times its diameter. The discovery of this new planet, which has been given the entirely new classification of ‘waterworld’, goes to show that advances in telescope technology are yet to continue revealing previously unknown truths of the universe. This new ‘waterworld’ orbits much nearer to its sun than the Earth does to ours, meaning that much of its water exists in vapour form, likely at temperatures in excess of 200C—another aspect of its discovery that has confounded and delighted astronomers. 4

Which of the following is the best statement of the conclusion of the argument above? A

A ‘waterworld’ that orbits very near its sun retains most of its water in vapour form.

B

The new ‘waterworld’ could not have been found without a very new telescope.

C

New telescope technologies will keep uncovering the universe’s hidden secrets.

D

A ‘waterworld’ is a planet of similar mass and diameter to Earth, which consists mostly of water.

E

The discovery of a distant ‘waterworld’ has delighted and confounded astronomers.

BMAT TEST TIP Sometimes, an argument’s conclusion may be difficult to find. If this happens, try to ‘boil down’ the argument to the author’s main point. Do this BEFORE looking at the answers, to avoid being led astray by a tempting wrong choice.

15

Lesson 1 Critical Reasoning Basics

Assumption Questions Assumption questions require you to first find the conclusion and the evidence. ·· The conclusion is the argument’s main point—everything else stated in the argument is evidence ·· The assumption is always unstated ·· Predicting the assumption will make quick work of the answer choices

Nurseries across England have been encouraged to give children less fruit and vegetables, and more cake. The advice may seem paradoxical, but children under five have growing bodies, requiring a diet with a higher proportion of fat and carbohydrates. Because children burn fat much more quickly than adults, the under-fives can enjoy daily portions of sponge, custard or biscuits, along with their ‘five-a-day’ of fruit and vegetables; whole milk and red meat also have a place on the menu, which may seem surprising, given that a low-fat diet is generally thought to be best for everyone. The simple, if obvious, explanation is that children under five have dietary needs that are fundamentally different from those of adults. 5

What is the central assumption in the argument above? A

Children and adults have different dietary needs.

B

Children under five should eat more cake and less fruit.

C

Nurseries in England are not giving children enough carbohydrates.

D

Adults need a diet that is low in fat.

E

Nurseries in England do not give children enough cake.

BMAT TEST TIPS An assumption is an unstated yet necessary link between evidence and conclusion. If it’s stated in the argument, it’s not an assumption.

16

Lesson 1 Critical Reasoning Basics

6

In a survey of 1,000 children, 50% could not identify a crocodile, and 10% could not recognise a gorilla. Ten per cent of children were also unable to distinguish a male lion from a female. Most children reported learning about animals from wildlife television programmes, but clearly these programmes are not sufficient. All schools should therefore take children to visit zoos.

Which of the following are assumptions of the argument in the passage above?

1 Visiting zoos will improve children’s ability to identify animals.



2 Most children can distinguish a crocodile from a gorilla.



3 Schools should improve children’s ability to identify animals.



4 Most children learn about animals at school.

A

1 and 2 only

B

1 and 3 only

C

1 and 4 only

D

2 and 3 only

E

2 and 4 only

F

3 and 4 only

BMAT TEST TIP If you’re having trouble finding the assumption, try the Negative Test. Negate each of the answer choices, and see if the argument still holds true. If negating an answer means the conclusion no longer follows from the evidence, then that answer is a valid and necessary assumption—and, therefore, correct.

17

Lesson 1 Critical Reasoning Basics

Flaw Questions Flaw questions ask you to describe what’s wrong with the argument as it stands. ·· Sometimes the flaw is extremely obvious, and these questions are quick work ·· Sometimes there is more than one flaw, or the flaw is a bit complicated to specify ·· Finding the assumption will help identify the flaw, as it’s always the argument’s reasoning that is flawed

Scientists in Russia have successfully regenerated a flower that had been extinct for thousands of years from ancient seeds. The seeds were buried by squirrels in burrows dug in frozen earth and lined with hay and fur; the burrows were so well lined as to be fully insulated from any water penetration. As a result, the scientists were able to plant the seeds and, despite being 30,000 years old, the seeds were viable. The flower that grew from them, known as Silene stenophylla, has borne viable seeds, bringing this long-lost species back to life. Silene stenophylla is by far the oldest flower that has ever been successfully regenerated from ancient fossilised seeds, but its example proves that any species of flower can be regenerated by scientists with the proper experience and knowledge of ancient flowers. 7

Which one of the following illustrates a flaw in the above argument? A

It claims that waterproof burrows caused the seeds to be viable, but this is only a correlation.

B

It fails to present evidence as to whether any older seeds have ever been regenerated.

C

It makes a faulty assumption about the behaviour of ancient squirrels.

D

It makes a generalisation about Russian scientists based on the experience of a few scientists.

E

It makes a generalisation about flowers based on the experience of a single species of flower.

BMAT TEST TIP Common flaws include (but are not limited to): ·· Causation vs. correlation ·· Value vs. percentage ·· Scope shift between evidence and conclusion ·· Overlooked alternatives ·· Inappropriate conflation/distinction

18

Lesson 1 Critical Reasoning Basics

The Large Hadron Collider (LHC) malfunctioned during initial testing and was disabled for over a year before resuming operation in late 2009. The breakdown was attributed to faulty electricals that damaged a series of magnets inside the LHC, which is attempting to detect the elusive Higgsboson particle, believed to be one of the fundamental building blocks of the universe. There may, however, be another reason for the failure of the LHC: two scientists have theorised that that malfunction may have been caused by a Higgs-boson particle from some future experiment that travelled back in time to damage the machine in the present. Thus, the LHC breakdown is in fact a watershed moment in the history of science, proving the existence of the Higgs-boson, and also that time travel is possible. 8

What is the best statement of the flaw in the argument above? A

It concludes that time travel is possible without providing evidence about time travel.

B

It overlooks the real reason for the failure of the Large Hadron Collider.

C

It does not explain the role of magnets in time travel.

D

It assumes that the Higgs-boson particle is a fundamental building block of the universe.

E

It concludes that time travel is possible based on a theory that it may be possible.

BMAT TEST TIP Absence of evidence is not evidence of absence.

19

Lesson 1 Critical Reasoning Basics

Method of Argument Questions Method of Argument questions ask you to identify the approach used to make an argument. ·· You may be asked to describe the argument, its organisation or the type of evidence it relies on. ·· These questions will often identify the conclusion, or part of the evidence. ·· Your knowledge of conclusion, evidence and assumption will help in attacking these questions.

Mick: It seems like no one goes to the library anymore. You can get all the news and information you need online, and nowadays it seems virtually any book you want is available to read on a computer or tablet. So the government cuts to library funding are completely justified. Nessa: Actually, it is clearly the case that the government’s cuts to library funding must be reversed. Communities depend on libraries to have a place where everyone can access the internet without having to pay, and not everyone can afford to buy electronic versions of books. Students and pensioners, in particular, are severely affected by the cuts to library funding. And, anyway, the only reason there are so many online versions of books is that libraries have invested the resources in scanning out-of-print editions. Further scanning by libraries has had to be curtailed because of these cuts. 9

Which one of the following best describes how Nessa makes her main point in response to Mick’s claim? A

She advises a course of action in support of his claim.

B

She makes a comparison to another relevant case.

C

She makes a conditional claim.

D

She predicts a future event that will occur because of his claim.

E

She asserts a fact that is contrary to his claim.

BMAT TEST TIP Learn the keywords that indicate the common types of conclusions that come up again and again: Prediction: will; shall; is going to Recommendation: should; ought to

Conditional: if Assertion of fact: must be; is certainly/definitely

Comparison: more; less

Value judgement: any evaluative statement

Assertions of fact can be a bit tough to spot—these conclusions assert something that must be true, though this ‘truth’ may not necessarily follow from the evidence.

20

Lesson 1 Critical Reasoning Basics

Following the disclosure that the Swiss branch of one major international bank helped customers to hide as much as $100 billion in secret accounts—nearly a fifth of it belonging to UK citizens— there has been much confusion about the difference between tax avoidance, which is legal, and tax evasion. Tax evasion is always wrong, and should remain illegal, but the relevant distinction is that tax avoidance is perfectly legal and should remain so. If someone is not legally required to disclose their foreign holdings to HM Revenue & Customs, then it is not a crime for them not to have done so. You could say that it’s much the same as the situation regarding hunger. Arguably, it’s wrong that so many of us throw away food that is perfectly edible when so many others in our own country are going hungry and having to depend on food banks. But it is not illegal to waste food, and why should it be? Who would pay for the cost of policing it, and why should it be a priority? 10

Which one of the following best describes the evidence that is offered in the remainder of the argument for the statement below? ‘If someone is not legally required to disclose their foreign holdings to HM Revenue & Customs, then it is not a crime for them not to have done so.’ A analogy B

appeal to authority

C prediction D statistics E testimony

BMAT TEST TIPS Start by identifying the conclusion. This will help you to think about, and describe, the evidence. If you are asked to describe a specific sentence from the evidence, check that your answer describes only that sentence—not the sentence that comes just before or just after.

21

Lesson 1 Critical Reasoning Basics

LESSON 1 QUIZ (6 minutes)

Please read this page carefully, but do not start the quiz until you are told to do so. Speed as well as accuracy is important in this section. Work quickly, or you may not finish the quiz. There are no penalties for incorrect responses, only points for correct answers, so you should attempt all questions. All questions are worth one mark. You will have 6 minutes to answer the 4 questions in this quiz. Record your answers in this book. Show your choice clearly by circling the corresponding letter in your book. If you make a mistake, erase thoroughly and try again. Any rough work should be done in this book. At the invigilator’s prompt, turn the page and begin the practice quiz.

23

Lesson 1 Critical Reasoning Basics

As companies create virtual maps of Britain for the internet, they are changing our fundamental understanding of the landscapes in which we live. Internet maps are primarily used for driving, and as such are designed to focus on roads and intersections. Most internet maps omit pedestrian walkways, and leave out landmarks such as churches, museums, woodlands and stately homes. Britain’s landscapes are a geography rich in history; sadly, all of this history disappears in the driver-focused maps made for the internet. 11

Which one of the following underlies the argument above? A

Drivers do not need to know the locations of museums and stately homes.

B

People who use internet maps are not aware of history.

C

People who use pedestrian walkways do not use the internet.

D

Pedestrians do not need to use internet maps.

E

Drivers need to know the locations of churches and woodlands.

The percentage of people who believe that human activity does not contribute to climate change has increased over the past five years. Climate change sceptics often mention the extreme cold and snow of the last few winters as justification of their view. However, the last few summers have been among the hottest in recent decades, and evidence shows that human activity contributed to the extreme heat. Super-hot summers led to unprecedented melting of polar ice-caps, making for colder winters. 12

24

Which one of the following best expresses the conclusion of the passage above? A

Climate change sceptics are wrong.

B

Hotter summers make for colder winters.

C

Climate change makes for hotter summers.

D

Winter weather influences people’s opinions.

E

Humans have not contributed to the extreme weather of recent years.

Lesson 1 Critical Reasoning Basics

If the Government is serious about cutting spending, they should eliminate the military and all other spending on national defence. High-tech and high-cost programmes, such as the Trident nuclear deterrent, make up a disproportionate share of the national budget, and are very difficult to keep up-to-date and properly functioning. The odds of a war are even less than those of a nuclear attack, and in the event of any attack, we can always rely on our military alliances to defend us. 13

Which one of the following identifies a weakness in the above argument? A

Defence spending is only 5% of the budget.

B

We have had wars, but never a nuclear attack.

C

Government computers are difficult to keep up-to-date.

D

There is a cost to maintaining our military alliances.

E

The Government spends more on office supplies than on national defence.

It’s commonly believed that high-speed trains are essential to the future of European travel, and they are often heralded for their low cost to consumers and their relatively low impact on the environment, as compared to aeroplanes. It’s also commonly believed that, as European governments build more high-speed lines and improve connections and travel times, there will be a reduction in air travel. However, this reduction has not been borne out in the evidence, and it is significant to note that the increase in high-speed rail travel has led to higher energy use and carbon emissions. It’s true that high-speed rail has eliminated the market for short-haul flights on such routes as Brussels-Paris and Frankfurt-Cologne. Unfortunately, though, these are exceptions to the trend, which is that the introduction of high-speed rail serves in the main to increase travel undertaken by affluent customers, who otherwise would not have travelled by car or air; over medium to longer distances, high-speed train journeys remain too expensive for the vast proportion of Europeans, who are taking advantage of the cheaper airfares for these journeys. As a result, energy use and carbon emissions have in fact increased. 14

The statement below plays which one of the following roles in the argument? ‘It’s true that high-speed rail has eliminated the market for short-haul flights on such routes as Brussels-Paris and Frankfurt-Cologne.’ A

It states the main point of the argument.

B

It introduces information that makes the argument’s conclusion more likely to be true.

C

It concedes a point that would seem to undermine the argument’s conclusion.

D

It explains that there are limited exceptions to an overall negative trend.

E

It is the key justification for the argument’s conclusion.

STOP. IF YOU FINISH BEFORE TIME IS UP, CHECK ANY QUESTIONS YOU HAVE MARKED FOR REVIEW. YOU MAY GO BACK TO QUESTIONS IN THIS QUIZ ONLY.

25

Lesson 1 Critical Reasoning Basics

THE KAPLAN METHOD FOR CRITICAL REASONING

STEP 1: ID the question type.



STEP 2: Attack the argument.



STEP 3: Eliminate and select.

ATTACKING CRITICAL REASONING QUESTIONS Conclusion: Find by using keywords or the One Sentence test Assumption: Look for the missing link between evidence and conclusion Flaw: Look for what’s gone wrong between evidence and conclusion

Method of Argument: Use your knowledge of conclusion, evidence and assumption

CRITICAL REASONING CHALLENGES ·· Weaken questions ·· Strengthen questions ·· Inference questions ·· Formal Logic

26

Lesson 1 Critical Reasoning Basics

ANSWERS AND EXPLANATIONS Answer Key 1. (D)

8. (E)

2. (A)

9. (E)

3. (B)

10. (A)

4. (C)

11. (A)

5. (D)

12. (A)

6. (B)

13. (D)

7. (E)

14. (C)

Explanations 1. (D) This is a Conclusion question, so look through the argument to find the sentence that expresses its main point, or its strongest opinion. The strongest opinion is the scientists’ ‘surprising’ finding that tea will deliver optimum taste if drunk within 6 to 17 minutes after brewing, because tea tastes best between temperatures of 45C and 60C. Answer (D) identifies this conclusion, and is therefore correct. As for the wrong answers: Don’t assume that the conclusion is necessarily the argument’s last sentence or clause; in this instance, the final clause starts with ‘This is because’, which are keywords indicating that what follows is evidence. (E) is therefore evidence, and cannot be the conclusion. (C) similarly repeats evidence, this time from the argument’s second sentence, which gives part of the background of the scientists’ research into tea. (A) and (B) give strong statements of opinion, but they do not match what is stated in the argument, so they cannot be correct. 2. (A) This is a Flaw question, so identify conclusion, evidence and assumption to find the flaw. The conclusion is the last sentence: foxes in the South-East are no longer shy, and threaten children. The evidence includes two instances in which foxes in the SouthEast of England attacked small children, as well as the fact that foxes are normally shy and avoid contact with humans. The assumption here is that foxes in the South-East are fundamentally different from foxes in the rest of England. Obviously, there are some logical flaws at work in this reasoning. The first flaw is nicely stated in Statement 1: the argument assumes that the foxes were not shy when they attacked the children,

but what if they attacked because they were frightened? This would mean that there had been no fundamental change in the nature of the foxes. Eliminate answers (C) and (E), which do not include Statement 1 as a flaw in the argument. Statement 2 captures another flaw: we are given evidence about two individual foxes, as the basis for a sweeping conclusion about all foxes in the South-East. These two foxes are hardly representative, so Statement 2 is a flaw in the argument. Eliminate (B), which does not include it. You may think at first glance that Statement 3 is also a flaw in the argument, but note that the final sentence says that the South-East of England is the only part of the country where foxes have attacked children. Thus, foxes in other parts of the country have been considered. Statement 3 is not a flaw, so eliminate (D). The correct answer is therefore (A). 3. (B) This is a Conclusion question, so find the sentence that best expresses the argument’s main point. In this case, it’s the final sentence: the NHS can do better. The answer choices are all a bit wordier, but you can quickly eliminate any that fail to mention the NHS: (A), (D) and (E) are out. The evidence is all about the problem of people not being able to see GPs, and either foregoing treatment or pursuing home remedies. Thus, the specific way the NHS can do better is by making it easier to see a GP. Answer (B) is therefore correct. 4. (C) This is a Conclusion question, so check quickly to see whether there is a conclusion keyword. The argument’s third sentence states that ‘The discovery of this new planet . . . goes to show.’ In this argument, the discovery of the new planet is the evidence, and ‘goes to show’ are the keywords indicating that what follows is the conclusion: advances in telescope technology are yet to continue revealing previously unknown truths of the universe. The correct answer, (C), is the only close match. 5. (D) This is an Assumption question, so find the conclusion and the evidence, and use these to work out the assumption. The conclusion is the last sentence: children under five and adults have very different

27

Lesson 1 Critical Reasoning Basics

dietary needs. There is a lot of evidence about these dietary needs, so let’s see if we can eliminate based on the terms in the conclusion. The correct assumption will link some piece of the evidence to something in the conclusion, so it must include either adults or children under five. On this basis, eliminate (A), (C) and (E), which mention children rather than children under five, as children are a much larger group and not included in the conclusion. Answer (B) may be tempting, but note that it restates the first sentence of the argument. This sentence is evidence; remember, the assumption is unstated, so anything stated directly in the argument cannot be the assumption. Eliminate (B), and the correct answer must therefore be (D). The second-to-last sentence says that a lowfat diet is best for everyone, and linking this piece of evidence to our conclusion leads to the assumption: adults need a diet that is low in fat. 6. (B) This argument includes multiple assumptions, but we can handle it just like any other Assumption question. First, find the conclusion (much simpler this time around, thanks to the keyword therefore): all schools should take children to visit zoos. The evidence is all about children’s difficulties at identifying animals, which they mostly learn about from wildlife TV programmes. The two terms in the conclusion that do not appear in the evidence are zoos and schools. So the assumptions must link each of these back to the evidence. Statement 1 links zoos to children’s ability to identify animals, so it is a correct assumption. Eliminate answers (D), (E) and (F), as they do not include Statement 1. Statement 2 makes a comparison based on statistics in the evidence, so this cannot be an assumption; eliminate (B). Statements 3 and 4 both mention school; Statement 4 says that children learn about animals at school, but this contradicts the argument; if children learnt about animals at school, they would not need to visit zoos. Statement 3, on the other hand, says that schools should help students to better identify animals. This fits with the conclusion and evidence in the argument, so it is correct. The answer is therefore (B). 7. (E) On the BMAT, most flaws will involve disconnects between evidence and conclusion. The first step to finding a flaw, then, is to find the conclusion, and consider how it differs from the evidence. In this case, the conclusion is the final clause of the argument’s final

28

sentence, introduced by the keywords ‘its example proves’: the conclusion is that any species of flower can be regenerated by scientists who know what they’re doing with ancient flowers. Take a moment to scan the evidence, and you’ll see it’s all about a single species of ancient flower, Silene stenophylla, which has been successfully regenerated from 30,000 year old seeds. No other flowers are mentioned in the evidence. You can’t necessarily assume that scientists who know what they’re doing could regenerate any flower based on the results from a single ancient flower. This is a major flaw in the argument, and it’s well described in answer (E), which is correct. 8. (E) With a Flaw question, start by seeking out the conclusion. Here, it’s in the last sentence, signalled by the keywords thus and proving: the LHC proves that the Higgs-boson exists, and that time travel is possible. The evidence is that some scientists have theorised that the LHC breakdown was caused by a Higgs-boson particle travelling back in time from a future experiment. There is no evidence for this theory; it’s simply given as an alternate explanation. Thus, our evidence says that time travel may be possible, and could explain the LHC malfunction; the conclusion states that this theory proves that time travel is possible. Answer (E) matches this precisely, and is correct. Answer (A) may be tempting, but note that the argument does provide evidence about time travel, that is, the claim by the two scientists that a time-travelling Higgs-boson may have caused the damage to the LHC. These scientists do not give evidence for their claim, but their claim is evidence in the argument. 9. (E) This is a Method of Argument question, because it asks you to describe how Nessa’s main point responds to Mick’s claim. Her main point is her conclusion, which is the first sentence of her argument. This sentence is the strongest opinion in her argument, indicated by the keywords ‘Actually’ and ‘clearly’; also, all the other sentences in her argument are facts that support the claim in the first sentence—the first sentence is the conclusion, and the rest is evidence. Nessa’s claim is that the government’s cuts to library funding must be reversed. Her main point directly contradicts Mick’s—his conclusion (indicated by the keyword ‘So’) is that the library funding cuts are completely justified. Their points of view are in direct contention, so eliminate (A). She does not make a comparison to

Lesson 1 Critical Reasoning Basics

another case—she directly contradicts him—so eliminate (B). There is no conditional in her conclusion—no if/then—so eliminate (C). Nessa says that the library cuts must be reversed, but she does not predict that they will be reversed. Eliminate (D). Nessa’s claim is best described as an assertion of fact, indicated by ‘clearly’ and ‘must be,’ and her assertion is certainly contrary to Mick’s claim. Answer (E) is correct. 10. (A) This question asks you to describe the evidence that supports a particular statement in the argument, which indicates that it is a Method of Argument question. The conclusion of the argument follows the ‘but’ in its second sentence: the key distinction is that tax avoidance is perfectly legal and should remain so. This is a mix of an assertion of fact (tax avoidance is perfectly legal) and a recommendation (tax avoidance should remain legal). The next sentence is a conditional claim that it is not a crime if you don’t do something that is not specifically illegal. It’s this sentence that the question is asking about. The following sentence says that the situation with tax avoidance is much the same as the situation regarding hunger, and the rest of the argument fills out this comparison. A comparison is an analogy, so the correct answer is (A). 11. (A) Another Assumption question, so start by finding the conclusion. The conclusion is the statement of the author’s main point or opinion, and the strongest statement of that in this argument comes in the final sentence; the keyword ‘sadly’ is a strong indicator of opinion. The conclusion, therefore, is that Britain’s historical geography disappears in the driver-focused internet maps. The evidence mentions some of the things that are omitted from these maps, including churches, museums and stately homes. Thus, the assumption must link drivers to the things omitted from the maps. The correct answer is therefore (A), which states that drivers do not need to know the locations of museums and stately homes, which are left off internet maps. You may be tempted by (E), but note that this contradicts the argument: if drivers needed to know the locations of churches and woodlands, the internet maps would not be very driver-focused! Answers (B), (C) and (D) are clearly wrong, as they do not mention drivers.

12. (A) This Conclusion question is a bit tricky, as none of the sentences in the argument seems to be a statement of its main point. The first part of the argument talks about climate change sceptics, who use extreme weather from recent years to justify their view; the second part of the argument shows why these sceptics are wrong. Answer (A) seems a good fit, as disproving climate change sceptics appears to be the argument’s main point. If you’re not sure about this, then check the other answers to eliminate. (B) paraphrases the argument’s last sentence, and (C) paraphrases its next-to-last sentence; both sentences sound like facts, or evidence, so they cannot be the conclusion. Eliminate (B) and (C). (D) follows logically from the argument, but is too neutrally stated to be correct. (E) contradicts the argument, which makes the case that humans have influenced the extreme weather. Thus, the correct answer must be (A). 13. (D) This is a Flaw question. The conclusion is that the Government should eliminate the military and all defence spending. The evidence for this is that much defence spending is wasteful, that wars are unlikely, and that our military alliances will protect us in case of a war. The assumption is that we will be safe without spending anything on defence. Answer (D) addresses the flaw in this logic: the argument overlooks the cost in maintaining our military alliances, which we would not be able to support if we eliminated the military and all defence spending. Without these alliances, we would be defenceless in the face of attack, so the argument is weak on this point. None of the other answers address flaws in the argument. Answers (A) and (C), if true, would weaken the argument, but both bring in new information, rather than pointing to flaws in the existing argument. Answer (E) makes an irrelevant comparison involving office supplies, which are not mentioned in the argument. Answer (B) addresses past attacks, whilst the argument addresses future attacks, so it is not clearly relevant. The correct answer is therefore (D). 14. (C) This Method of Argument question asks you to describe the role played by a statement in the argument. The argument’s conclusion is the second part of the third sentence: the increase in high-speed rail travel has led to higher energy use and increased

29

Lesson 1 Critical Reasoning Basics

c­ arbon emissions. Eliminate (A). The sentence following the conclusion is the statement that the question is asking about. This statement makes a point that would seem to support the opposite view—that is, that high-speed rail travel has in fact reduced energy use and carbon emissions by eliminating the market for air travel on two routes. However, the next sentence indicates that these are limited exceptions to an overall negative trend. Thus, there’s a wrong trap answer, (D)—the clarification that Brussels-Paris and Frankfurt-Cologne are exceptions to the trend comes not in the statement in question, but in the following sentence. Eliminate (D). Our statement does not make the conclusion more likely to be true, and it is not the key justification for the conclusion—that comes after our statement. The correct answer is therefore (C).

30

LESSON 2

Problem Solving Basics LEARNING OBJECTIVES By the end of this session, you will be able to:

• Identify Word Problems, Repeating Questions and Spatial Questions. • Use the Kaplan Methods to solve Word Problems, Repeating Questions and Spatial Questions.

• Identify opportunities to estimate, eyeball and eliminate (‘the three Es’) to save time.

Lesson 2 Problem Solving Basics

INTRODUCING PROBLEM SOLVING ·· At least one-half of questions in BMAT Section 1 are Problem Solving questions. ·· These questions test your ability to analyse data and make calculations quickly and accurately.

Problem Solving Basics ·· Approximately 18 to 23 questions (out of 35) ·· Appear throughout the section, including on multi-question sets ·· Many include data, in the form of charts, graphs or tables ·· Most have 4 or 5 answer choices; some will include numbered statements.

Warm-Up Exercise ·· The next page contains two Problem Solving questions. ·· Try to complete the exercise, and answer both questions, in 3 minutes.

33

Lesson 2 Problem Solving Basics

Over the centuries, there have been many ideas for new systems of tracking time. One such system is known as the ‘hexadecimal clock’, in which each day would be divided into 16 hexahours, with each hexahour divided into 128 hexaminutes and each hexaminute divided into 16 hexaseconds. Clocks would indicate the time in hexahours and hexaminutes instead of hours and minutes. Midnight would be 00:00 and midday would be 08:00. 1

If a digital hexadecimal clock were compared with a standard 24-hour digital clock, what would the standard clock read when the time on the hexadecimal clock is 12:64? A 17:50 B 18:20 C 18:45 D 19:15 E 19:30

The chart below shows the cumulative number of countries that were members of the Commonwealth in various years. The vertical axis indicates the number of countries that were members of the Commonwealth in the first year of each decade, as plotted on the horizontal axis. 60 50 40 30 20 10 0 1940

2.

1950

1960

1970

1980

1990

2000

2010

How many countries were members of the Commonwealth in 1990, but were not members of the Commonwealth in 1960? A 11 B 30 C 34 D 38 E 45 F 49

STOP. IF YOU FINISH BEFORE TIME IS UP, CHECK ANY QUESTIONS YOU HAVE MARKED FOR REVIEW. YOU MAY GO BACK TO QUESTIONS ON THIS PAGE ONLY.

34

Lesson 2 Problem Solving Basics

Problem Solving Challenges ·· Can appear in almost any format, many of which appear to be time-consuming and ‘scary’ ·· Challenging to do the straightforward maths without a calculator ·· Some formats require specific skills that are not taught in school ·· Very difficult to answer in 2 minutes or less, without shortcuts

Attacking Problem Solving Questions ·· There are four types of Problem Solving questions. ∘∘ ∘∘ ∘∘ ∘∘

Word Problems: usually involve algebra, geometry or probability Repeating Questions: involve a sequence that repeats or an overlapping element Spatial Questions: involve ‘flipping’ and rearranging shapes or figures in 2 or 3 dimensions Data Questions: involve making calculations or deductions based on a chart, graph or table

·· All Problem Solving questions will involve maths, and all will provide one or more shortcuts. ∘∘ Repeating Questions usually have the same shortcut. ∘∘ Spatial Questions usually have the same type of shortcut. ∘∘ Word Problems and Data Questions are usually as much about reasoning as they are about maths. ·· Whenever a question involves calculations, the BMAT will almost always provide figures in a format that allows you to simplify calculations by minimising maths. ·· The skills required to attack these questions can be learnt, and improved for speed. ·· With these skills, you can answer most Problem Solving questions in 2 minutes.

BMAT SKILLS IN PRACTICE You are a veterinarian and determine that one of your patients, a bloodhound called Columbo, requires daily medication for a heart condition. Columbo weighs 75 lbs. The medication is a liquid to be taken orally, and the indicated dosage is 2 cc per 5 lbs of body weight per day. What daily dosage of the medication would you prescribe for Columbo? How would you calculate the correct dosage for Columbo? Do you need a calculator to determine the dosage? What are the implications if you make an error? Columbo’s owner does not have a device to measure cubic centimetres. If one teaspoon equals 5 cc, how many teaspoons of the medication would you advise her to administer to Columbo per day? How is this calculation similar to the first calculation? What would be the danger if a BMAT question involved both calculations?

35

Lesson 2 Problem Solving Basics

THE KAPLAN METHOD FOR CALCULATE QUESTIONS STEP 1: STEP 2: STEP 3: STEP 4:

THE KAPLAN METHOD FOR SPATIAL AND REPEATING QUESTIONS STEP 1: STEP 2: STEP 3: STEP 4:

Problem Solving Triage Under certain circumstances, it’s best to triage a Problem Solving question. You should always triage a P ­ roblem Solving question when:   , or

In either instance, you should always

 .

 ,

and, depending on how much time is left, you might also before moving on to the next question.

BMAT TEST TIPS With practice, you will improve your basic numeracy skills between now and Test Day. The first step to doing so is to master the ‘Three Es’—Estimate, Eyeball and Eliminate. Working with fractions is an essential step on almost all calculations on the BMAT. Practise with fractions, and be ready to use them early and often.

36



Lesson 2 Problem Solving Basics

Minimise Maths: Estimate ·· Estimate by rounding up or down to make calculations quick and manageable. ·· The BMAT likes to provide figures in formats that encourage rounding. ·· It’s often easiest to complete calculations by working with figures in fraction format, as fractions allow you to reduce before you multiply or divide.

Siobhan and Zubaida bake a total of 1,518 cookies for the Volunteer Society Bake Sale. On the first day of the sale, Siobhan sells 417 cookies, and Zubaida sells 493 cookies.

3

On the first day of the sale, Zubaida sold what percentage of all cookies baked? A 26% B 32% C 39% D 48%

4

On the first day of the sale, Siobhan and Zubaida sold what percentage of all cookies baked? A 50% B 55% C 60% D 65%

ANNUAL BUDGET Rent and Utilities 15%

ADVERTISING BUDGET

Advertising 10% Other 15%

Internet 15%

TV 35%

Production 25%

Print 30% Personnel 35%

Radio 20%

TOTAL: £1,945,000

A company’s annual budget is broken down by category in the chart on the left. The advertising category is broken down by subcategory in the chart on the right. 5

How much has the company budgeted for internet advertising? A £28,895 B £29,175 C £30,225 D £31,045

37

Lesson 2 Problem Solving Basics

Minimise Maths: Eyeball ·· Eyeball by making a quick visual estimate of rough percentages and fractions from data in charts, graphs and tables. ·· Eyeballing requires you to make a rough approximation, but this is often enough to answer a question correctly.

A company’s annual budget is broken down by category in the chart on the left. The advertising category is broken down by subcategory in the chart on the right. ANNUAL BUDGET Rent and Utilities 15%

ADVERTISING BUDGET

Advertising 10% Other 15%

Internet 15%

TV 35%

Production 25%

Print 30% Personnel 35%

Radio 20%

TOTAL: £1,945,000

One of the categories has been omitted from the annual budget, resulting in the chart below. 6

Which category has been omitted? A Advertising B Other C Personnel D Production E

Rent and Utilities

BMAT TEST TIPS Eyeballing makes quick work of questions where the answer choices are words, rather than numbers. When the answers are numbers, you can eyeball and eliminate any answers that are clearly too large or clearly too small, based on the data.

38

Lesson 2 Problem Solving Basics

Minimise Maths: Eliminate ·· Eliminate any answers that are too large, too small or otherwise just not right. ·· Be sure to eliminate each step of the way as you estimate or eyeball—sometimes, you can ‘zero in’ on the correct answer with partial work.

A standard sheet of A4 (210 mm × 297 mm) has been folded into a paper aeroplane. The sheet was folded lengthwise into quarters, forming the wings and the body of the aeroplane. The top of each side was folded back, to increase aerodynamic performance, and form the wings shown below. PAPER AEROPLANE, TOP VIEW 52.5 mm 100 mm

7

50 mm

What is the area of the top of the left wing? A

9405 mm2

B

11655 mm2

C

13875 mm2

D

15295 mm2

BMAT TEST TIPS Memorise the fractions equivalent to common percentages: 1 12.5% = 8

1 20% = 5

1 33.3% = 3

1 10% = 10

1 14.3% = 7

1 25% = 4

2 40% = 5

1 11.1% = 9

1 16.7% = 6

3 30% = 10

1 50% = 2

5% =

1 20

Estimating by rounding to these percentages will save loads of time on Test Day.

39

Lesson 2 Problem Solving Basics

Word Problems Word problems on the BMAT are designed to seem like they require far more than 2 minutes each. ·· Make sure you understand the basic situation before beginning calculations ·· Estimate, eyeball and eliminate wherever possible ·· When a problem appears to involve unreasonable calculations, there must be a shortcut

8.

A bag contains exactly eight marbles—four are red, and four are blue. I select a marble at random from the bag, and then another, and then another, and so on, until the bag is empty.

What is the probability that all the marbles will be in chosen in alternating colour sequence (eg red-bluered-blue or blue-red-blue-red)? A

1 48

B

1 35

C

1 21

D

1 14

E

1 8

BMAT TEST TIPS desired outcomes Probability = possible outcomes On the BMAT, fractions are your friends.

40

Lesson 2 Problem Solving Basics

9

Estimated ideal body weight (IBW) is calculated using the following formulae:



For males: IBW = 50 kg + 2.3 kg for each inch in height over 5 feet For females: IBW = 45.5 kg + 2.3 kg for each inch in height over 5 feet

If height is 5 feet or less, the formulae cannot be used. If actual weight is greater than estimated IBW + 30%, calculate the estimated adjusted body weight (ABW) using this formula, for males and females alike:

ABW = IBW + 0.4 (actual weight – IBW)

Which of the following individuals has an estimated adjusted body weight of 73.7 kg? A

A man with height of 6 feet, 5 inches, and actual weight of 84.5 kg

B

A man with height of 6 feet, 1 inch, and actual weight of 95.8 kg

C

A man with height of 5 feet, and actual weight of 67.3 kg

D

A woman with height of 4 feet, 11 inches, and actual weight of 62.4 kg

E

A woman with height of 5 feet, 4 inches, and actual weight of 60.9 kg

F

A woman with height of 5 feet, 6 inches, and actual weight of 95.4 kg

BMAT TEST TIPS If a Word Problem requires a time-consuming calculation for each answer choice, then check the very large or very small answers first. These will often give you a sense of the parameters of the maths involved, and help you eyeball the remaining answers. Sometimes partial calculations provide enough information to eliminate two or three answers.

41

Lesson 2 Problem Solving Basics

Spatial Questions Spatial questions require you to mentally ‘flip’ or rearrange one or more diagrams, shapes or other multidimensional objects. You may be required to reason in two or three dimensions. ·· The correct answer is likely to be an ‘odd-man-out’: a particular arrangement or position. ·· The odd-man-out may be difficult to spot; check closely so you don’t miss it. ·· If you have trouble mentally flipping an two-dimensional object, try rotating the test paper. ·· If you have trouble mentally flipping a three-dimensional object, use your rubber as a visual aid.

10

The floor of my conservatory is covered with 81 tiles to form a 9 by 9 square. How many different types of individual tiles are there on my floor?

A 5 B 6 C 7 D 8 E 9

BMAT TEST TIPS If a Spatial question includes a diagram, don’t hesitate to draw on it as you work out your answer. Just be sure to use pencil, so you can rub it out if you need to.

42

Lesson 2 Problem Solving Basics

11

If folded up, the pattern below will form a cube.

Which one of the following patterns could fold up to form an identical cube?

A

B

C

D

E

BMAT TEST TIP If a Spatial question involves unfolding the sides of a three-dimensional cube, use your rubber as a visual aid: once you work out the relative positions of a couple of sides of the cube, assign these to sides of your rubber—rotate the rubber to work out the rest.

43

Lesson 2 Problem Solving Basics

Repeating Questions Repeating questions appear to ask you to count out something unreasonably large. Instead, follow this shortcut: ·· Work out the sequence ·· Work out how often the sequence repeats ·· Find the spot in the sequence where the correct answer will fall Sometimes, you will have to count out the number of various items to find the total. Watch out for overlaps— don’t overcount!

12.

Construction crews building a 104-floor skyscraper work Monday to Friday, and will take 9 working days to complete each floor once they complete the 20th floor, which happens on a Wednesday. They start work on the 21st floor on the next day.

If work continues on schedule, with no disruptions due to weather or Bank Holidays, on what day of the week will they complete the 104th floor? A Monday B Tuesday C Wednesday D Thursday E Friday

BMAT TEST TIP If a problem contains a repeating element, solve by finding the sequence. Test-takers who don’t find the sequence will waste time, and miss out marks.

44

Lesson 2 Problem Solving Basics

13

The digital display at a train station is broken, so the stationmaster devises a system that will allow her to display the current time and date using a set of transparent paddles, each representing a number from 0 to 9. The new system would display the time at 16:49 on the 25th of July as shown below.

1 6:4 9 2 5/0 7 To save on costs, the stationmaster will use the same paddles to represent both 6 and 9 (by turning the paddles 180 degrees). Since the paddles are transparent, the same paddles will be used to represent both 2 and 5 (by flipping the paddles from front to back). All times are displayed using the 24-hour clock (from 00:00 to 23:59). What is the minimum number of paddles required to represent all possible dates and times from 1st January to 31st December? A 43 B 47 C 48 D 51 E 52 F 54

BMAT TEST TIP If you notice an overlap in one or more elements of a repeating sequence, you may be able to solve most quickly by listing out the elements. This will allow you to avoid making an error in counting ­elements twice. Simple, but very effective!

45

Lesson 2 Problem Solving Basics

LESSON 2 QUIZ (5 minutes)

Please read this page carefully, but do not start the quiz until you are told to do so. Speed as well as accuracy is important in this section. Work quickly, or you may not finish the quiz. There are no penalties for incorrect responses, only points for correct answers, so you should attempt all questions. All questions are worth one mark. You will have 5 minutes to answer the 3 questions in this quiz. Record your answers in this book. Show your choice clearly by circling the corresponding letter in your book. If you make a mistake, erase thoroughly and try again. Any rough work should be done in this book. At the invigilator’s prompt, turn the page and begin the practice quiz.

47

Lesson 2 Problem Solving Basics

14

The ratio of gazelles to lions in a nature preserve is 91:1. During the last quarter-century, the gazelle population has fallen by 30%, whilst the lion population has doubled. What was the approximate ratio of gazelles to lions in the nature preserve 25 years ago? A 65:1 B 182:1 C 260:1 D 650:1

48

Lesson 2 Problem Solving Basics

15

The pentagon shown below has been carved up into five quadrilaterals, which are shown in the answers, along with one quadrilateral that is not part of the completed pentagon.

Which quadrilateral is not part of the completed pentagon?

A. A.

B. B.

C. C.

D. D.

E. E.

F.

F.

49

Lesson 2 Problem Solving Basics

16

Two runners are racing on an oval track. The quicker runner has an average lap time of 2 minutes, 28 seconds, and the slower runner has an average lap time of 2 minutes, 36 seconds. If they run at constant speed and started at the same time, how long will it take for the quicker runner to overtake the slower runner? A

37 minutes, 54 seconds

B

39 minutes, 36 seconds

C

47 minutes, 18 seconds

D

48 minutes, 6 seconds

E

49 minutes, 12 seconds

STOP. IF YOU FINISH BEFORE TIME IS UP, CHECK ANY QUESTIONS YOU HAVE MARKED FOR REVIEW. YOU MAY GO BACK TO QUESTIONS IN THIS QUIZ ONLY.

50

Lesson 2 Problem Solving Basics

THE KAPLAN METHOD FOR CALCULATE QUESTIONS STEP 1: ID the target. STEP 2: Research relevant data. STEP 3: Set up and solve. STEP 4: Eliminate and select.

THE KAPLAN METHOD FOR SPATIAL AND REPEATING QUESTIONS STEP 1: ID the target. STEP 2: Check components. STEP 3: Count/Match/Rotate. STEP 4: Eliminate and select.

ATTACKING PROBLEM SOLVING QUESTIONS Estimate, Eyeball and Eliminate to minimise maths Spatial: Look for the odd-man-out (often the largest/smallest is key) Repeating: Work out the sequence, then solve mathematically or by counting/listing

PROBLEM SOLVING CHALLENGES ·· Data questions ·· Multi-question sets ·· Complex calculations

51

Lesson 2 Problem Solving Basics

ANSWERS AND EXPLANATIONS Answer Key

2. (D)

1. (C)

9. (F)

2. (D)

10. (D)

3. (B)

11. (C)

4. (C)

12. (D)

5. (B)

13. (A)

6. (D)

14. (C)

7. (B)

15. (E)

8. (B)

16. (D)

3. (B)

Explanations 1. (C) This question asks you to convert the time from an unusual new ‘hexadecimal’ clock to the standard 24-hour clock. Since midnight is 00:00 in the new clock and midday is 08:00, you know that 8 hexahours = 12 standard hours. The time in question, 12:64, includes 12 hexahours, which must equal a standard hours. Set up a ratio to solve:

On the first day of the sale, Zubaida sold 493 of the total 1518 cookies baked. The maths are a lot easier 500

if you estimate 493 as 500, and 1518 as 1500: 1500 1 1 = 3 = 33%. Think of 3 as the number of pieces (the numerator) from the total pieces in a pie (the denominator). Since 1500 is slightly less than 1518 and 493 is slightly less than 500, the actual ‘pie’ (the actual number of cookies Zubaida sold) is slightly larger than in our estimated fraction, and the actual number of pieces is slightly smaller. Thus, Zubaida sold slightly fewer cookies from a slightly larger total, and the actual percentage must be just slightly less than 33%. The correct answer must be (B), 32%. 4. (C)

8 hexahours 12 = a 12 hr 12 × 12 = 8a 8a = 144 a = 18

Siobhan and Zubaida sold a total of 417 + 493 = 910 cookies out of the total 1518 cookies baked. Estimate 1518 as 1500 and 910 as 900 to make quick work of 900 9 3 the maths: 1500 = 15 = 5 = 60%. The answer is (C).

Thus, there are 18 standard hours in the time that will display on the standard 24-hour clock. Next, convert the 64 hexaminutes to standard minutes. The first step is to convert the 64 hexaminutes to hexahours. Since there are 128 minutes in a hexahour, then 64 64 1 hexaminutes equals 0.5 hexahours, since 128 = 2 . Set up another ratio, with b as the unknown number of standard hours that equals 0.5 hexahours:

5. (B)

8 hexahours 0.5 = b 12 hr 12 × 0.5 = 8b 8b = 6 6

The annual budget is £1,945,000, and 10% of this is advertising, which is £194,500. Fifteen per cent of the advertising budget is for internet advertising, and we can find this quickly by taking 10% of £194,500, which is £19,450, and then half of this amount, £9,725, as 5% is half of 10%, and 5% + 10% = 15%. Thus, 15% of £194,500 is £19,450 + £9,725 = £29,175. The answer is (B). 6. (D)

3

b = 8 = 4 = 0.75 Hence, there are 0.75 standard hours in 0.5 hexahours, or 45 standard minutes. The time is therefore 18:45 on the standard 24-hour clock when it is 12:64 on the hexadecimal clock. Answer (C) is correct.

52

According to the graph, the Commonwealth had 11 member countries in 1960, and 49 member countries in 1990. This means that there were 49 – 11 = 38 countries that were members in 1990, but not 1960. Answer (D) is correct.

Eyeball the original and adjusted charts, and you will notice that each contains two portions that are the same size. These must be Other and Rent and Utilities. The portion between them is smaller in both charts, so that portion must be Advertising in both charts. Eliminate (A), (B) and (E). The largest portion in the adjusted chart must be Personnel or Production. If 35 25 it’s Personnel, it’s 75 ; if it’s Production, it’s 65 . Since the largest portion is nearly half of the chart, it must correspond to Personnel, so the correct answer is (D).

Lesson 2 Problem Solving Basics

7. (B)

9. (F)

Divide the left wing into two portions: the triangle at the top, and a long rectangle. The rectangle has a width of 52.5 mm, shown in the diagram, and its length is 297 mm – 100 mm = 197 mm. Estimate the area of the rectangle as 50 mm × 200 mm = 10,000 mm2. Eliminate answer (A), which is clearly too small, as we have not yet added in the triangle portion of the wing. The triangle has a base of 52.5 mm, and a height of 100 mm – 50 mm = 50 mm. Estimate the area of 1 the triangle as 2 × 50 mm × 50 mm = 1250 mm2. The total estimated area of the wing is thus 10,000 mm2 + 1250 mm2 = 11,250 mm2. Answers (C) and (D) are clearly too large, so the correct answer is (B).

Read the information carefully, and try to eliminate before doing any maths. IBW is calculated differently for males and females, but ABW is only calculated if someone’s actual weight is greater than IBW + 30%. Also, the IBW formulae cannot be used if someone’s height is 5 feet or less. Eliminate (C) and (D), as they involve people whose height is 5 feet or less. Check each of the remaining answers to see whether the ABW formula will be required. The first man is 17 inches taller than 5 feet, and 17 × 2.3 kg = 39.1 kg. His IBW is 50 kg + 39.1 kg = 89.1 kg, which is greater than his actual weight, so the ABW formula is not required. Eliminate (A). The first man would have to have an actual weight nearly 30 kg more than his IBW (and 35 kg more than his actual weight) for the ABW formula to be required. Thus, it’s safe to infer that the second man also will not require the ABW formula, as he is 4 inches shorter than the first man, but only weighs 11 kg more. Eliminate (B). With this knowledge of when the ABW formula is required, it’s clear that of the two remaining answers, women of nearly the same height, only the final woman will weigh enough to require the ABW formula. The correct answer must be (F).

8. (B) For each choice in the sequence, calculate the probability as a fraction, with the number of marbles of the colour left in the bag as the numerator, and the total number of marbles left in the bag as the denominator. Pick any marble from the bag on the first pick, as it does not matter whether the sequence starts with red or blue. Thus, on the first pick there are 8 marbles of the colour we want, and 8 marbles in the bag. Let’s say a red marble is chosen. On the second pick, a blue marble is then required. There are 4 blue marbles, and 7 marbles total, in the bag. The odds of picking 4 a blue marble on the second pick are 7 . On the third pick, a red marble is desired; there are 3 red marbles and 6 total marbles in the bag; the odds of picking 3 1 red are 6 , or 2 . On the fourth pick, it’s a blue marble again; there are 3 blue marbles and 5 total marbles in the bag, so the odds of picking a blue marble are 3 . On the fifth pick, it’s back to red. There are 2 red 5 marbles and 4 total marbles in the bag, so the odds of 1 picking red are 2 . On the sixth pick, it’s another blue; there are 2 blue and 3 total marbles in the bag, so the 2 odds of picking blue are 3 . On the seventh pick, it’s the final red; there’s only 1 red and 2 total marbles 1 in the bag, so the odds of picking red are 2 . On the final pick, the only marble left in the bag is blue; the odds of this are 1. Multiply the odds of all the picks to find the probability of the overall sequence: 4

1

3

1

2

1

1

1 × 7 × 2 × 5 × 2 × 3 × 2 × 1 = 35 The probability of an alternating colour sequence is 1 , answer (B). NB If you reverse the order, and start 35 with a blue marble and finish with a red, the probability will work out to be exactly the same, as the question is about alternating the sequence of choices, rather than the particular colours involved.

10. (D) When a Spatial question involves repeating tiles, the quickest way to distinguish them is to make a note on your test paper—either draw each distinct tile, or circle an example of each. The central type of tile is a large white square (1); in all the other types, the white square is divided into as many as 4 square segments, which are black or white; sometimes, the white squares are outlined by a solid line across the middle of the large square. Thus, there are other types that have a black square in one corner (2), and one with a white square outlined in one corner (3). There’s another type that’s half-black and half-white (4), and one that’s white on both sides with a line down the middle (5). There’s a further type that has one white square, with the other three black squares forming an L (6). Then, there are the tiles such as the second from the left in the top row and the second from the left in the third row. These may initially appear to be the same, but if you rotate your book, you will see that if you rotate the second tile in the top row 180°, then the black and white squares will be in the top half of the box, but the black square will be in the top left and the white square in the top right. This is sufficient to see that these are two distinct tiles. Thus, there are a total of 8 different types of tile. The answer is (D).

53

Lesson 2 Problem Solving Basics

11. (C)

13. (A)

When a Spatial question requires you to think in three dimensions, find a way to describe the arrangement so that you can compare and eliminate the answer choices efficiently. This question involves folding a pattern into a cube. Imagine that the cube is folded up, and pick a side to be the front, and another to be the top. Let’s say that the crossed circle is the front, and the black X is the top. Starting with the crossed circle and going round the sides of the cube, then, the clockwise order is crossed circle, black donut, black octagon, white arrows. Starting with the top and going clockwise around the bottom of the cube and back up again, while skipping the front with the crossed circle, the order is black X, white arrows, white octagon, black donut. Compare this sequence to the clockwise order in each answer choice, if the crossed circle is the front of the cube:

This question contains two repeating elements: the same paddles will be used for 2 and 5, and the same paddles will be used for 6 and 9. You must monitor the repeating elements carefully, or you will select an incorrect answer. The display includes the time from 00:00 to 23:59 and the date, with two digits for the day and the month. The months can range from 01–12, and the days from 01–31. The only way to work out the correct number of paddles is to count how many are required for each digit, although there are two general approaches: you could count them in order—starting with the 0’s, and working up to the 9’s—or you could count them in decreasing frequency. If you took the latter approach, the first step is to consider which digit would be used most frequently: it’s possible to have a 1 in all 8 slots, so you would need eight 1’s. There is no other digit that could fill all 8 slots, though you could have a 2 in seven of the slots—all but the first digit of the month. There is no other digit that could fill 7 slots, though you could have a 0 in six of the slots—all 4 time slots, and one on each side of the slash in the date. So far, that means we have 8 + 7 + 6 = 21 paddles, including the digits 0, 1, 2 and 5 (since the 2 paddles would also be used for 5).

A: Clockwise from the top: black X, white arrows, black donut, white octagon. Eliminate (A). B: Clockwise from the top: black X, black donut, white octagon, white arrows. Eliminate (B). C: Clockwise from the top: black X, white arrows, white octagon, black donut. Answer (C) is correct. For the record, the other two incorrect sequences are: D: Clockwise from the top: black X, black donut, white octagon, white arrows. E: Clockwise from the top: black X, white arrows, black octagon, black donut. 12. (D) Work out the pattern, starting from the 21st floor, until we see how often it repeats: Floor Starting Day 21st Thursday 22nd Wednesday 23rd Tuesday 24th Monday 25th Friday 26th Thursday

Finishing Day Tuesday Monday Friday Thursday Wednesday Tuesday

The pattern repeats every five floors, so we can see that the 26th, 31st, 36th and so forth will start on a Thursday and finish on a Tuesday, the 27th, 32nd, 37th and so forth will start on a Wednesday and finish on a Monday, and so on. We need the finishing day for the 104th floor, which comes 4th in our repeating pattern. Thus, we know that the 104th floor would be completed on a Thursday, and the correct answer is (D).

54

If the time were 03:33 or 04:44, then you would need five 3 or five 4 paddles, since you could also have one on either side of the slash in the date. Thus, you need five 3 paddles and five 4 paddles. Be careful not to add five 5 paddles—that is the maximum possible number of 5’s, but we can use the 2 paddles to represent 5. We have added 10 more paddles, taking the total to 21 + 10 = 31. If the time were 06:06, then you would need four 6 paddles, since you could have one each in the day and the month. The same would apply for 7 and 8—if the time were 07:07 or 08:08, you would need four paddles for each digit. This adds 4 × 3 = 12 more paddles, for a total of 31 + 12 = 43. The maximum possible number of 9’s is the same as for 6: if the time were 09:09, you would need four 9 paddles on the 9th September. However, these are already covered by the 6 paddles. Thus, the minimum number of paddles required is 43. Answer (A) is correct.

Lesson 2 Problem Solving Basics

14. (C) The current population of gazelles and lions is in a ratio of 91:1. So the actual numbers could be 9100 gazelles and 100 lions (or any numbers in a 91:1 ratio). The gazelle population has fallen 30% over the past 25 years, so the current number of gazelles is 70% 7 of the total, G, from 25 years ago. G × 10 = 9100, so 10 G = 9100 × 7 , which reduces to G = 13,000. The number of lions has doubled over the last 25 years, so the number of lions 25 years ago, L, is simply half the number of lions today; therefore, L = 50. Thus, the ratio of gazelles to lions 25 years is 13,000:50. 13,000 ÷ 50 = 260, so the ratio in reduced form is 260:1, and the correct answer is (C).

× 2 min/lap = 39 min. Multiplying 19.5 laps × 28 sec/ lap could take a bit of time, so instead multiply 19.5 laps × 0.5 min/lap = 9.75 min (as 30 seconds is only slightly more than 28, and 30 sec = 0.5 min). The total time is slightly less than 39 min + 9.75 min = 48.75 min, so eliminate (A) and (B), which are clearly too small, and also (E), which is clearly too large. 48.75 min is 48 minutes, 45 seconds, and we can calculate the extra seconds as 30 sec – 28 sec = 2 sec, and then multiply 2 sec × 19.5 = 39 sec. Subtract for the exact total: 48 min, 45 sec – 39 sec = 48 min, 6 sec. The answer is therefore (D).

15. (E) This Spatial question may seem a bit scary, as it can be tricky to turn the pieces mentally to form the original image. However, the twist here—one of the six answers is a piece we don’t need to form the original figure—makes it more straightforward than expected. Scanning the pieces, you will notice that some are larger than others. Piece (C) is the smallest, so consider what happens if we eliminate (C), eg if it is the piece that does not belong. It’s fairly clear that the remaining five pieces have a combined area that is much greater than the pentagon they are meant to form. Thus, we can conclude that (C) must be part of the pentagon, and we can further deduce that one of the larger pieces is likely to be the odd-man-out. Simply by eyeballing the combined area of (A) and (E) each with the other four pieces, it’s fairly clear that (E) is the odd-man-out, as eliminating (A) and keeping (E) would result in a total area that is slightly larger than the pentagon. The correct answer is therefore (E). 16. (D) The quicker runner is 8 seconds per lap faster than the slower runner. He will need to make up a total of 2 minutes, 36 seconds (the time of the slower runner) in order to overtake the slower runner. In seconds, this is 2 min × 60 sec/min = 120 sec, and 120 sec + 36 sec = 156 sec. Divide to find the number of laps the quicker runner will need to overtake the slower runner: 156 sec ÷ 8 sec/lap = 19.5 laps. Multiply 19.5 laps by the quicker runner’s speed to find the total time he will require to overtake the slower runner. The easiest way to do this without a calculator is to break the speed into sections: first multiply 19.5 laps

55

LESSON 3

Applied Science Basics LEARNING OBJECTIVES By the end of this session, you will be able to:

• Explain the breakdown of Section 2 of the BMAT. • Use the Kaplan Method for Applied Science to solve Biology and Chemistry Questions. • Apply the appropriate strategy to Find the Fact, Matching Table, Compound and Reaction Questions.

• Apply the appropriate strategy to Numbered Statements and Illustration Questions.

Lesson 3 Applied Science Basics

INTRODUCING APPLIED SCIENCE ·· All the questions in BMAT Section 2 are based on GCSE Biology, Chemistry, Physics and Maths content. ·· Section 2 questions require you to apply science and maths knowledge in new contexts and work quickly to find the correct answers.

Applied Science Basics ·· 27 questions in 30 minutes (1 minute per question) ·· 7 biology questions; 7 chemistry questions; 7 physics questions; 6 maths questions ·· Questions usually in the same repeating sequence: one each from biology, chemistry, physics, maths ·· Most questions have 4, 5 or 6 answer choices; a few may be fill-in questions, though none of these have appeared in the last few years.

Warm-Up Exercise ·· The next page contains two Applied Science questions. ·· Try to complete the exercise, and answer both questions, in 2 minutes.

59

Lesson 3 Applied Science Basics

A family tree for cystic fibrosis (CF) is shown below:

A

B

C

D

E

1

2

Which one of the following statements must be true? A

Individuals A and C are both heterozygous for CF

B

The offspring of B and a female heterozygous for CF will have a 50% probability of being homozygous recessive for CF

C

The next offspring of C and D has a 50% probability of being heterozygous for CF

D

The offspring of E and a male homozygous dominant for CF can not be heterozygous for CF

E

Individuals B and E are both heterozygous for CF

Fool’s gold (pyrite) changes colour when it comes into contact with oxygen. The equation for this reaction is:

x FeS2 (s) + y O2 → z SO2 (g) + 2 Fe2O3 (s)

What is the value of y? A 2 B 3 C 4 D 6 E 8 F 11

STOP. IF YOU FINISH BEFORE TIME IS UP, CHECK ANY QUESTIONS YOU HAVE MARKED FOR REVIEW. YOU MAY GO BACK TO QUESTIONS ON THIS PAGE ONLY.

60

Lesson 3 Applied Science Basics

Applied Science Challenges ·· Questions draw on a wide range of knowledge ·· Most BMAT test-takers have not studied physics since they sat their GCSEs ·· Maths questions may draw on relatively obscure formulae (eg, area of a trapezium) ·· Timing is brutal—many questions are very tough to answer in 1 minute ·· Triage is almost impossible—you are unlikely to have time for a second pass

Attacking Applied Science Questions ·· Biology and chemistry questions are more straightforward, because: ∘∘ Nearly all BMAT test-takers are sitting A-levels in Biology and Chemistry. ∘∘ In Section 2, there are 4 types of biology questions and 4 types of chemistry questions—and 2 of these are common to both. ∘∘ Most Biology questions involve genetics or the human body (e.g. homeostasis). ∘∘ Most Chemistry questions involve stoichiometry or the periodic table. Most are Calculate questions. ·· Physics and maths questions are generally more challenging, because: ∘∘ They focus on specific topics from physics and maths that BMAT test-takers will likely not have studied in the last year or two. ∘∘ In Section 2, physics questions usually cover the same 6 topics, and maths questions usually cover the same 3 topics. ∘∘ You should plan to revise the pages dealing with these topics in the Assumed Subject Knowledge Guide most thoroughly. ·· Speed is a key consideration—a few questions may be impossible to answer in 1 minute. ·· Use the Assumed Subject Knowledge Guide judiciously to drive your revision.

BMAT SKILLS IN PRACTICE You are a medical student on your first full time hospital placement. On the ward round, the team reviews two of the patients you helped admit that day. The first patient came to A&E feeling very unwell, with a high fever and a cough that produced rustycoloured phlegm. Although his cough was painful, he didn’t feel out of breath. You recorded his capillary oxygen saturation as 92%. Together with the team, you make a diagnosis of pneumonia. The second patient you saw told you that her GP had sent her to hospital because she was ‘having an asthma attack’. While you were with her in A&E you noticed she was very breathless, and you checked her oxygen saturation. It was 99%. The consultant asks you: ‘Why was the second patient so breathless, but the first patient was not?’ What is the essence of question the consultant has asked you? What is the relevant information you gathered from seeing these patients? What scientific principles will help you answer this question?

61

Lesson 3 Applied Science Basics

THE KAPLAN METHOD FOR APPLIED SCIENCE STEP 1: STEP 2: STEP 3:

What is the key difference in a Calculate question?

Applied Science Triage You are unlikely to have time to return to any questions in Section 2. So it’s essential that you ____________________________________ for each question, before moving on to the next one. If you decide to skip a question, then you should ____________________________________ , but still ____________________________________ before moving to the next question.

BMAT TEST TIP Minimising maths and working with fractions will be even more important in Section 2 than in Section 1. Practise these skills, so they are second nature by Test Day.

62

Lesson 3 Applied Science Basics

Biology: Find the Fact Questions ·· On a Find the Fact question, you simply choose the answer that gives a correct fact. ·· This is the most basic question type in Section 2, and usually covers only biology content. ·· Most Find the Fact questions will involve genetics or the human body. ·· If you know the fact, great! If you don’t know or are unsure, then: ∘∘ Eliminate answer choices one at a time ∘∘ Don’t tackle tricky or uncertain answer choices unless absolutely necessary

When developing new antibiotics, scientists try to drugs create that are toxic to bacterial cells, but not to human cells. 3

Which of the following would NOT be an appropriate target for a new antibiotic? A

A drug that inhibits an enzyme needed for the synthesis of a key cell wall component.

B

A drug that can cross a cell membrane, but not a cell wall.

C

A drug that attacks DNA, but cannot cross a nuclear membrane.

D

A drug that stimulates a ‘self-destruct’ enzyme specific to bacterial cells.

The following four statements relate to sex-linked inheritance:

4



1. If a male expresses a Y-linked trait, all of his male offspring will express the trait.



2. If a male expresses an X-linked trait, none of his male offspring will express the trait.



3. If a female expresses an X-linked recessive trait, all of her male offspring will express the trait.



4. If a female does not express an X-linked recessive trait, none of her male offspring will express the trait.

Which statements are always correct? A None B

1 only

C

1 and 2

D

1 and 3

E

1, 2 and 3

F

1, 3 and 4

G All

BMAT TEST TIP If there are multiple statements, you can attack them in any order. Eliminate answers as you evaluate each statement.

63

Lesson 3 Applied Science Basics

Biology: Matching Table Questions ·· On a Matching Table question, the answer choices appear in a table format. You must match several pieces of information to a diagram or sequence. ·· These questions tend to look complicated, but are usually rather straightforward: ∘∘ Start with the column in the table you can deal with most quickly. ∘∘ Eliminate the answers that do not have the correct information for this column. ∘∘ Repeat until only one answer remains.

The following family tree shows that individual 8 is the only family member affected by an inherited condition.

1

2

4

3

5

6

8

9

7

5

What are the genotypes of individuals 1, 5, 6 and 8? (D = normal allele, d = disease allele) Answer

Individual 1

Individual 5

Individual 6

Individual 8

A

Dd

DD or Dd

DD or Dd

Dd

B

Dd

Dd

Dd

dd

C

DD or Dd

DD or Dd

dd

dd

D

DD or Dd

Dd

Dd

dd

E

DD or Dd

DD or Dd

DD or Dd

Dd

BMAT TEST TIP You may encounter a Matching Table question with one or more columns that you cannot narrow down. Don’t fret; eliminate based on the other columns, and then guess from the remaining answers.

64

Lesson 3 Applied Science Basics

6

Which of the following represents the flow of oxygen from the lungs to the kidneys? A

pulmonary vein

atria

ventricles

aorta

renal artery

B

pulmonary artery

ventricles

atria

vena cava

renal vein

C

pulmonary vein

ventricles

atria

aorta

renal vein

D

pulmonary artery

atria

ventricles

aorta

renal artery

E

pulmonary vein

atria

ventricles

vena cava

renal artery

BMAT TEST TIP Never spend more than a minute on a Matching Table question. That should be enough time to get it down to 2 or 3 answer choices, at which point you should make your best guess and move on.

65

Lesson 3 Applied Science Basics

Chemistry: Compound Questions ·· Compound questions ask you to work with molecular formulae. ·· You may have to solve for a formula, or use a formula to find the number of atoms/moles of a particular element in the compound. ·· Some Compound questions may appear to involve more complex calculations, but you can usually make quick work of these by using your knowledge of fractions and percentages.

7

An unknown compound consists of 84.71% carbon and 15.29% hydrogen. What is its molecular formula? (Ar: H = 1, C = 12) A C6H8 B C8H8 C C12H16 D C12H26

BMAT TEST TIP When a question asks you to find the formula for a compound, work backwards from the answer choices.

66

Lesson 3 Applied Science Basics

8

Hydrogen makes up 13.13% of the mass of one mole of ethanol. The molecular formula for ethanol includes hydrogen, carbon and oxygen; there are a total 6 hydrogen atoms in one molecule of ethanol. What is the total mass of one mole of ethanol? (Ar: H = 1, C = 12, O = 16) A

46.07 g

B

48.29 g

C

49.63 g

D

52.14 g

BMAT TEST TIPS Chemistry questions will often include percentages that are meant to look scary. Don’t fret; rewrite the percentage as an easy-to-work-with fraction. Ar × No of atoms (of element) Percentage mass (of an element in a compound) = Mr (of whole compound)

67

Lesson 3 Applied Science Basics

Chemistry: Reaction Questions ·· Reaction questions give you the equation for a chemical reaction, and usually ask you to: ·· Work out the coefficients for one or more of the reactants/products. ·· Solve for the mass or number of moles of one of the reactants/products, based on information about the mass or number of moles of one of the other reactants/products.

9

An explosion of TNT (trinitrotoluene) results in a number of combustive products. The equation for TNT combustion is:

w C6H2CH3(NO2)3 → x CO + x C + y H2O + z N2

What is the value of x? A 2 B 3 C 5 D 6 E 7 F 8

BMAT TEST TIPS If a Reaction question gives you the value for one of the coefficients, start by balancing out the element involved on both sides of the reaction. Only do as much work on a reaction question as required to find the answer. Don’t get carried away trying to work out all the coefficients, if you don’t have to.

68

Lesson 3 Applied Science Basics

10

Burning propane (C3H8) produces carbon dioxide and liquid water. The equation for the reaction is: C3H8 (g) + 5 O2 → 3 CO2 (g) + 4 H2O (l) How much propane was burnt in a reaction that produced 360 g of water? (Ar: H = 1, C = 12, O = 16) A

180 g

B

220 g

C

240 g

D

260 g

BMAT TEST TIPS Number of moles =

Mass in g (of element/compound) Mr (of element/compound)

If a Reaction question involves the mass of a reactant/product, you will need to find the mass of a mole of the reactant/product (Mr), then multiply by the number of moles; eg: Number of moles × Mr (of element/compound) = Mass in g (of element/compound)

69

Lesson 3 Applied Science Basics

Biology and Chemistry: Numbered Statement Questions ·· You should attack all Numbered Statement questions with the same basic approach: ·· Pick a statement you know is correct (or incorrect). ∘∘ If it’s correct, eliminate all the answers that don’t include it. ∘∘ If it’s incorrect, eliminate all the answers that include it. ·· Pick another statement, and repeat, until you are left with the correct answer.

The following four statements relate to halogens (Group VII):

11



1. A halogen can react with an alkali metal to form a salt



2. Reduction is the process in which a halogen loses an electron to form a stable electronic structure



3. Chlorine is more reactive than iodine



4. Astatine is more reactive than fluorine

Which statements are correct? A

1 only

B

1 and 2

C

1 and 3

D

3 and 4

E

1, 2 and 3

F

1, 3 and 4

G

2, 3 and 4

BMAT TEST TIPS Most Numbered Statement questions will require you to check all the statements. However, you may encounter a question in which one statement appears in more answer choices than any other statement. If you notice this, check that statement first. OILRIG.

70

Lesson 3 Applied Science Basics

The five statements below are about the role of the kidneys in homeostasis.

12



1. All sodium ions are removed from the blood by the kidneys.



2. The kidneys convert any extra amino acids into carbohydrates or fats.



3. Membranes in the kidneys filter proteins and blood cells, so they do not pass through to the bladder.



4. The kidneys filter urea from the bloodstream, storing it in the bladder.



5. Some water may be reabsorbed into the bloodstream as it passes through the kidneys.

Which statements are correct? A

1, 2 and 3

B

1, 3 and 4

C

1, 4 and 5

D

2, 3 and 5

E

2, 4 and 5

F

3, 4 and 5

BMAT TEST TIP Some statements may involve facts that you simply can’t remember. Don’t fret; eliminate based on the facts you can recall. On a Numbered Statement question, this is usually sufficient to eliminate most of the answers, at which point you should make your best guess and move on.

71

Lesson 3 Applied Science Basics

Biology and Chemistry: Illustration Questions ·· Illustration questions feature a diagram or other visual image. ·· The illustration may be very elaborate or complicated. ·· Read the question before looking at the illustration. It’s possible that you won’t need to use much of the information shown, so don’t let it distract or frighten you, or make you lose time.

Signals are transmitted between neurons at synapses. Below is a diagram of a synapse where the arrow shows the direction of transmission.

2 4

1

5

3

13

Which row correctly identifies the receptor molecules, the release of transmitter chemicals and where MDMA interferes in serotonin removal? receptor molecules

release of transmitter chemicals

MDMA interferes in serotonin removal

A

2

4

3

B

3

2

4

C

4

2

5

D

3

4

5

E

4

2

3

F

5

3

4

BMAT TEST TIP Don’t look at the illustration until you have read the question. If you can’t resist, cover the illustration with your hand.

72

Lesson 3 Applied Science Basics

LESSON 3 QUIZ (4 minutes)

Please read this page carefully, but do not start the quiz until you are told to do so. Speed as well as accuracy is important in this section. Work quickly, or you may not finish the quiz. There are no penalties for incorrect responses, only points for correct answers, so you should attempt all questions. All questions are worth one mark. You will have 4 minutes to answer the 4 questions in this quiz. Record your answers in this book. Show your choice clearly by circling the corresponding letter in your book. If you make a mistake, erase thoroughly and try again. Any rough work should be done in this book. At the invigilator’s prompt, turn the page and begin the practice quiz.

73

Lesson 3 Applied Science Basics

14

The following three statements are about glucose:

1. Oxygen is required to produce energy from glucose in the cell



2. Glucose can be synthesised from amino acids in the liver



3. Glucose contains carbon, hydrogen, oxygen and nitrogen

Which statements are correct?

15

A

1 only

B

2 only

C

3 only

D

1 and 2

E

1 and 3

F

2 and 3

Sodium cyclopentadienide can be produced by mixing sodium with cyclopentadiene (C5H6):

2 Na + C5H6 → 2 NaC5H5 + H2

In an experiment, all the hydrogen gas produced by the reaction was captured in a hood, and measured a total of 7 g. (Ar: H = 1, C = 12, Na = 23) What was the total mass of sodium used in the experiment?

74

A

46 g

B

92 g

C

161 g

D

207 g

Lesson 3 Applied Science Basics

16

The following family tree indicates the children and grandchildren of Couple X (the first generation). The male partner in Couple X has a genetic disorder known as polydactyly, meaning that he was born with extra fingers and toes. The female partner in Couple X is not affected by polydactyly.

?

?

?

?

?

How many of the grandchildren of Couple X could have polydactyly? A 20% B 40% C 60% D 80%

17

Halon 2402 is a compound made of carbon, fluorine and bromine. It contains 61.5% bromine and 9.2% carbon, and has an Mr value of 260. What is the molecular formula of Halon 2402? (Ar: C = 12, F = 19, Br = 80) A C2F4Br2 B C4F2Br2 C C4F4Br3 D C6F4Br3

STOP. IF YOU FINISH BEFORE TIME IS UP, CHECK ANY QUESTIONS YOU HAVE MARKED FOR REVIEW. YOU MAY GO BACK TO QUESTIONS IN THIS QUIZ ONLY.

75

Lesson 3 Applied Science Basics

THE KAPLAN METHOD FOR APPLIED SCIENCE STEP 1: ID the subject. STEP 2: Work the data/answers. STEP 3: Eliminate and select.

THE KAPLAN METHOD FOR CALCULATE QUESTIONS STEP 1: ID the target. STEP 2: Research relevant data. STEP 3: Set up and solve. STEP 4: Eliminate and select.

ATTACKING APPLIED SCIENCE QUESTIONS Use the question format to your advantage in biology and chemistry questions. Eliminate as you work, using partial knowledge where necessary. NEVER leave a question without marking an answer on your answer sheet. Revise judiciously with the Assumed Subject Knowledge Guide.

APPLIED SCIENCE CHALLENGES ·· Physics ·· Maths ·· Complex calculations

76

Lesson 3 Applied Science Basics

ANSWERS AND EXPLANATIONS Answer Key

human cells have just a cell membrane. As a result, (A) would selectively attack bacteria, whereas (B) would not be effective because the drug could not attack a bacteria cell if it cannot cross a cell wall. Answer (B) is correct.

1. (C)

10. (B)

2. (F)

11. (C)

3. (B)

12. (F)

4. (D)

13. (E)

5. (D)

14. (B)

4. (D)

6. (A)

15. (C)

7. (D)

16. (B)

8. (A)

17. (A)

There are four statements, and any of them could be correct. The quickest approach is to check the statements in order, and eliminate answers after checking each statement. The first statement is always correct, as a Y-linked trait would be passed from a father to all his sons, and they would all express the trait. Eliminate (A). The second statement is not correct, as a son inherits his X chromosome from his mother; whether or not his father expresses an X-linked trait is not relevant to whether the son inherits and expresses it. Eliminate (C), (E) and (G). The third statement is always correct, as male offspring will receive an X chromosome from their mother—and thus would express a recessive trait, such as red-green colour-blindness, because a male only has one X chromosome. Eliminate (B). The fourth statement is a bit tricky. If a female does not express an X-linked trait, then there are two possibilities: she could be a carrier (X T X), with one allele with the recessive gene for the trait and the other without it, or she could lack the allele for the trait on either chromosome (XX). In the latter case, then none of her male offspring could express the trait. However, in the former case, 50% of male offspring would have the unaffected X chromosome, and 50% would express the trait (X T ). Thus, statement 4 is not always correct. Answer (D) is correct.

9. (E)

Explanations 1. (C) This question is about individuals in a family tree for cystic fibrosis (CF). CF is an autosomal recessive condition. Working with the diagram, the parents C and D must both be heterozygous, as they are both unaffected but they have affected children. We can not state the genotype for A, B or E specifically. This means that the only answer choice that must be true is (C). 2. (F) The right side of the equation has 4 iron atoms, so the left side must have the same; the value of x is thus 4. This means that the left side of the equation has 8 sulfur atoms; the right side must have the same, so the value of z is 8. On the right side, then, there are 16 oxygen atoms in the SO2, and 6 oxygen atoms in the Fe2O3, for a total of 22 oxygen atoms. The left side of the equation must have the same; since all the oxygen on the left side is O2, the value of y must be 22 ÷ 2 = 11. Answer choice (F) is correct. 3. (B) This question about antibiotics focuses on the differences between bacterial and human cells. Answer (D) is the quickest to eliminate; a ‘self-destruct’ enzyme specific to bacteria would clearly be a good target. Since bacteria don’t have a membrane-bound nucleus, a drug that attacks DNA but can’t cross a nucleus membrane would be good; eliminate (C). The remaining answers, (A) and (B), are quite similar. Bacteria have a cell wall and cell membrane, whereas

5. (D) Since only one member of the 3rd generation of the family is affected, the condition must be recessive. The disease allele is d, so individual 8 must have a genotype of dd. Eliminate (A) and (E). For a recessive condition to manifest in an individual born to unaffected individuals, both parents must be carriers of the disease allele, so individuals 5 and 6 must be heterozygous, Dd. Eliminate (C). Either individual 1 or 2 must be Dd, as both are unaffected by the disease yet they have a heterozygous child; however, individual 1 could also be DD, so eliminate (B). The correct answer is (D).

77

Lesson 3 Applied Science Basics

6. (A) Since this is a matching table, start with the columns you can narrow down most easily. When oxygenated blood enters the heart, it passes through the atria, then the ventricles, before exiting to the other organs via the aorta. The 2nd, 3rd and 4th columns must therefore be atria, ventricles and aorta; eliminate (B), (C) and (E). The two remaining answers both end with renal artery, which delivers oxygenated blood to the kidneys. Oxygenated blood enters the heart from the pulmonary vein (remember, the pulmonary artery delivers deoxygenated blood to the lungs), so the answer is (A). 7. (D) The quickest approach here is to calculate the percentage of mass that comes from carbon in each answer choice. (A) includes 6 carbon atoms, so its carbon mass is 12 × 6 = 72; its total mass is 72 + 32 = 104.

However, both sides of the reaction must be balanced, so the total number of carbon atoms on both sides must be the same. There are a total of 2x carbon atoms on the right side, and a total of 7w carbon atoms on the left side. Thus, 7w = 2x. This means that x must be a multiple of 7. There’s only one multiple of 7 in the answer choices, so (E) must be correct. 10. (B) The first step is to find the mass of one mole of water: Mr: H2O = 18 g. The reaction produced 360 g of water, or 360 g ÷ 18 g = 20 moles of water. Since the combustion process produces 4 molecules of water for each molecule of propane, a total of 20 ÷ 4 = 5 moles of propane were burnt. Next, find the mass of one mole of propane: Mr: C3H8 = (12 × 3) + (8 × 1) = 36 + 8 = 44 g. Finally, multiply the molar mass of propane by the number of moles burnt: 44 g × 5 = 220 g. The answer is (B).

72

The carbon is 104 , or about 72%, of the total mass, so eliminate (A). (B) has 8 carbon atoms, with a carbon mass of 12 × 8 = 96; its total mass is 96 + 30 = 96

90

126. Carbon is 126 , or about 120 or 75% of the total mass; eliminate (B). (C) has carbon mass of 12 × 10 = 120

120, and total mass of 120 + 28 = 148; carbon is 148 , 120

or about 150 or 80%, so eliminate (C). The answer must therefore be (D). (For the record, (D) includes 12 carbon atoms, with a mass of 12 × 12 = 144; its total 144

mass is 144 + 26 = 170. 170 = 0.8471.) 8. (A) A molecule of ethanol contains 6 hydrogen atoms, and each has a mass of 1. Set up an equation, with T representing the total mass of one mole of ethanol, and write the percentage as a fraction (rounding to 13%):

13 1×6 = T 100

Cross multiply to solve for T: 13T = 600

600

T = 13 = 46.15

Since the actual percentage was slightly more than 13%, the correct answer will be slightly smaller than 46.15. The answer is therefore (A). 9. (E) This question looks challenging, as all the coefficients on both sides of the reaction are given as variables.

78

11. (C) Start with statement 1, as it appears in the most answer choices. Halogens can react with alkali metals to form salts; NaCl is perhaps the most common example of such a salt. Statement 1 is correct; eliminate (D) and (G). Statement 3 appears the most in the remaining answers, so check it next. An element in Group VII with a lower atomic number will always be more reactive than one with a higher atomic number. Since chlorine has a lower atomic number than iodine, chlorine is more reactive. Statement 3 is correct; eliminate (A) and (B). Statement 4 is very similar to statement 2; you may not recall the exact position of astatine in the periodic table, but you will likely remember that fluorine is at the top of the column for Group VII, meaning that it is the most reactive. Thus, fluorine must be more reactive than astatine, and statement 4 is incorrect; eliminate (F). Statement 2 is incorrect; reduction is the process in which a halogen gains an electron to stabilise its electronic structure. Eliminate (E). Answer (C) is therefore correct. 12. (F) There are five statements, and any of them could be correct. The quickest approach is to check the statements in order, and eliminate answers after checking each statement. The first statement is not correct, for two reasons: the kidneys only remove excess sodium ions from the blood, and of course, some sodium ions would leave the body through sweat,

Lesson 3 Applied Science Basics

not through the bladder. Eliminate (A), (B) and (C). The second statement is not correct, as it’s the liver that converts excess amino acids into carbohydrates and fats. Eliminate (D) and (E). Answer (F) is correct. For the record: If you weren’t sure about statements 1 and 2 being incorrect, then you could have checked each of the remaining statements; as you determine that a statement is correct, then you can eliminate any answer choices that do not include that statement. It’s correct that membranes in the kidneys (between the blood vessels and the Bowman’s capsule) act like filters, so you can eliminate (C) and (E), which omit statement 3. Statement 4 correctly describes the kidneys’ function in removing urea from the bloodstream and storing it in the bladder; eliminate (A) and (D), which omit statement 4. As liquid flows along the nephron, sufficient water is reabsorbed into the blood. Thus, statement 5 is correct; eliminate (B), which omits it, and the correct answer is (F). 13. (E) The synapse is the gap between the neurons; transmitter chemicals are released at the end of one neuron (in the direction of transmission), crossing the synapse to bind to receptor molecules on the other side. Thus, the transmitter chemicals are released at 2; the receptor molecules are at 4. Eliminate all answers except (C) and (E). MDMA interferes with serotonin removal by blocking the spots in the brain’s synapses that remove serotonin. Thus, MDMA interferes in the synapse, which is 3 in the diagram. Answer (E) is correct.

so the total mass of sodium used in the experiment is 23 g × 7 = 161 g. Answer (C) is correct. 16. (B) This question asks polydactyly, which is caused by a dominant allele, D. An individual will have polydactyly (extra fingers or toes) if one of their parents does—that is, whether their genotype is DD or Dd. If an individual is not affected, their genotype must be dd. As a result, we know that two individuals who do not have polydactyly can only have children without the condition, since the parents’ genotypes would be dd and dd, yielding only children that are dd. On this basis, we can see that the three grandchildren at the right of the bottom row—the son of the middle couple, and both children of the couple at the right—must be dd. The children of the couple at the left could be Dd or dd, since they must have a d allele from the mother (who does not have polydactyly) and they could have a D or d allele from their father (who is polydactylic). Thus, 2 of the 5 grandchildren—or 40%—could have polydactyly. Answer (B) is correct. 17. (A) Bromine has an atomic mass of 80, and the answers include 2 or 3 bromine atoms. 2 × 80 = 160, and 3 × 80 = 240. The answers with 3 bromine atoms would have an Mr value well above 260, and consist of more than 90% bromine; eliminate (C) and (D). (B) has an Mr value that’s slightly too small (as its carbon has a mass of 12 × 4 = 48, and its fluorine has a mass of 19 × 2 = 38, which sums to less than the 100 needed). The answer is therefore (A).

14. (B) For statement 1, you could state a counterexample, as glucose can be used in a cell in anaerobic respiration without oxygen. Thus, statement 1 is incorrect; eliminate (A), (D) and (E). Statement 2 appears to test something unfamiliar, so move on to statement 3. Statement 3 says glucose contains nitrogen—this is false, as glucose is a carbohydrate and contains only carbon, hydrogen and oxygen. Eliminate (C) and (F); the correct answer is (B). 15. (C) H2 has a molar mass of 2, so the experiment produced 7 g ÷ 2 g/mol = 3.5 moles of H2. Thus, the experiment must have used 7 moles of cyclopentadiene and 7 atoms of sodium. Sodium has an atomic mass of 23,

79

LESSON 4

Writing Basics LEARNING OBJECTIVES By the end of this session, you will be able to:

• Explain the breakdown of Section 3 of the BMAT. • Describe the considerations used in grading the essay. • Use the Kaplan Method for Writing to plan and write a high-scoring essay.

Lesson 4 Writing Basics

INTRODUCING WRITING The Basics ·· 30 minutes ·· Write 1 essay from a choice of 3 prompts of general, medical, veterinary or scientific interest ·· Final product strictly limited to one A4 page

The Task ·· The questions will be in the form of a short proposition ·· The prompt will likely require you to: ∘∘ Explain or discuss the proposition’s implications ∘∘ Suggest a counter-proposition or argument ∘∘ Suggest a resolution

The Skills This section tests your ability to select, develop and organise ideas and communicate them in writing in a concise and effective way.

BMAT SKILLS IN PRACTICE Two students in Year 13 are both delighted to be offered a place at medical school. Naomi says she is thrilled with her offer from Scone College, where she will follow a three year preclinical programme that covers basic-medical science and research techniques in detail. She then will have the choice of four major teaching hospitals for her final three years of clinical placements. Afshan has gained a place at her first choice: St James’ Hospital Medical School. She is looking forward to having real patient contact from the very first week of her five year course at the medical school that has pioneered problem-based learning for medical education. “I wouldn’t swap my offer with yours in a million years!” What are the advantages of each course? What are the disadvantages? Who is on the better course?

83

Lesson 4 Writing Basics

Warm-Up Exercise ·· This page contains a set of four BMAT essay titles. ·· In the next two minutes, look over the essay titles and select the one for which you think you could write the best essay.

1

All 16-year-olds should be required to complete first aid training, as part of the National Curriculum. Write an essay in which you address the following points: What are the advantages in training all 16-year-olds in how to give CPR and deal with heart attacks, strokes and other major injuries? What are the drawbacks of requiring such training? To what extent do you agree that some first aid training should be compulsory, and at what age?

2

Research is what I’m doing when I don’t know what I’m doing. Wernher von Braun, German scientist, 1912-1977 Write an essay in which you address the following points: Explain what this statement means. Argue that a scientist can only undertake research with a clear goal or objective in mind. To what extent do you agree with von Braun that research is possible, or preferable, when scientists don’t know what they’re doing?

3

He who conceals his disease cannot expect to be cured. Ethiopian proverb Write an essay in which you address the following points: Why would a patient conceal his or her disease? How should a doctor deal with a secretive patient? How can a doctor balance the need to treat the patient with the patient’s preference for privacy?

84

Lesson 4 Writing Basics

SCORING In determining the mark for quality of content, the BMAT examiners from Cambridge Assessment consider how well the essay: ·· Addresses the question in the way demanded; ie, has the essay: ∘∘ rephrased the proposition or explained its implications? ∘∘ set out reasonable or plausible counter-propositions? ∘∘ proposed reasonable ways of assessing the competing merits of the propositions or resolved their conflict logically? ·· Organises thoughts clearly ·· Uses general knowledge and opinions appropriately In determining the mark for quality of written English, the BMAT examiners from Cambridge Assessment consider how well the essay: ·· Expresses ideas using concise, compelling and correct English To guide examiners in determining these marks appropriately, Cambridge Assessment provides a series of normative statements that indicate the essential qualities found in an essay of each score. Here are the normative statements for quality of content: Score 0: An essay that is judged to be irrelevant, trivial, unintelligible or missing. Score 1: An essay that some bearing on the questions but which does not address the question in the way demanded, is incoherent or unfocussed. Score 2: An essay that addresses most of the components of the question and is arranged in a reasonably logical way. There may be significant elements of confusion in the argument. The candidate may misconstrue certain important aspects of the main proposition or its implications or may provide an unconvincing or weak counter-proposition. Score 3: A reasonably well-argued answer that addresses all aspects of the question, making reasonable use of the material provided and generating a reasonable counter-proposition or argument. The argument is relatively rational. There may be some weakness in the force of the argument or the coherence of the ideas, or some aspect of the argument may have been overlooked. Score 4: A good answer with few weaknesses. All aspects of the question are addressed, making good use of the material and generating a good counter-proposition or argument. The argument is rational. Ideas are expressed and arranged in a coherent way, with a balanced consideration of the proposition and counter-proposition. Score 5: An excellent answer with no significant weaknesses. All aspects of the question are addressed, making excellent use of the material and generating an excellent counter-proposition or argument. The argument is cogent. Ideas are expressed in a clear and logical way, considering a breadth of relevant points and leading to a compelling synthesis or conclusion. Here are the normative statements for quality of written English: Band A – good use of English fluent good sentence structure good use of vocabulary sound use of grammar good spelling and punctuation few slips or errors

Band C – reasonably clear use of English reasonably fluent/not difficult to read simple/unambiguous sentence structure fair range and appropriate use of vocabulary acceptable grammar reasonable spelling and punctuation some slips/errors

Band E – rather weak use of English hesitant fluency/not easy to follow at times some flawed sentence structure/paragraphing limited range of vocabulary faulty grammar regular spelling/punctuation errors regular and frequent slips or errors

85

Lesson 4 Writing Basics

THE KAPLAN METHOD FOR WRITING STEP 1: STEP 2: STEP 3: STEP 4: STEP 5:

Writing Triage In the Writing Task, triage is all about preparation, and disciplined use of time. Follow Kaplan’s timing guidelines for each step: STEP 1: STEP 2: STEP 3: STEP 4: STEP 5:

BMAT TEST TIP Most test-takers make the same mistake on the Writing Task: they start writing their essay within 30 seconds of starting the section. Most of these students are still writing when time is called. Failure to plan a BMAT essay often results in failure to get into medical school. Don’t let it happen to you!

86

Lesson 4 Writing Basics

STEP 1: IDENTIFY THE TASK Answer one of the following questions.

1

All 16-year-olds should be required to complete first aid training, as part of the National Curriculum. Write an essay in which you address the following points: What are the advantages in training all 16-year-olds in how to give CPR and deal with heart attacks, strokes and other major injuries? What are the drawbacks of requiring such training? To what extent do you agree that some first aid training should be compulsory, and at what age?

2

Research is what I’m doing when I don’t know what I’m doing. Wernher von Braun, German scientist, 1912-1977 Write an essay in which you address the following points: Explain what this statement means. Argue that a scientist can only undertake research with a clear goal or objective in mind. To what extent do you agree with von Braun that research is possible, or preferable, when scientists don’t know what they’re doing?

3

He who conceals his disease cannot expect to be cured. Ethiopian proverb Write an essay in which you address the following points: Why would a patient conceal his or her disease? How should a doctor deal with a secretive patient? How can a doctor balance the need to treat the patient with the patient’s preference for privacy?

87

Lesson 4 Writing Basics

STEP 2: BRAINSTORM

Brainstorm Statement           Alternative

Thesis:

·· Take notes on the statement and an alternative position. ·· Think about how to reconcile them. ·· Remain aware of time (approx 6 minutes to brainstorm).

88

Lesson 4 Writing Basics

STEP 3: OUTLINE

Brainstorm Statement           Alternative

Thesis:

Outline

Paragraph 1



Paragraph 2



Paragraph 3

·· The essay should be 2–3 paragraphs. ·· Note the main point of each paragraph and where specific examples will go. ·· Do not write out the essay! ·· Refer to your outline repeatedly as you write the essay! ·· Take approximately 2 minutes to outline.

89

Lesson 4 Writing Basics

STEP 4: WRITE THE ESSAY ·· Vary sentence structure and word choice. ·· Paraphrase language from the essay title—don’t copy it word-for-word! ·· Use keywords to make transitions and link ideas, examples and explanations. ·· Avoid repetition. ·· Follow a three-step process for each example in your essay: ∘∘ Introduce the example ∘∘ Give the example ∘∘ Explain how the example illustrates the point you are trying to make ·· Use sophisticated, appropriate language.

BMAT TEST TIP If you’re not sure what a word means, don’t use it.

90

Lesson 4 Writing Basics

STEP 5: REVIEW ·· Approximately 3 minutes

·· Scroll through essay looking for:

∘∘

∘∘

∘∘

BMAT TEST TIPS Answer the specific questions asked! Remember the tasks set by the examiner: ·· Explain the proposition ·· Consider an alternative position ·· Propose a resolution

91

Lesson 4 Writing Basics

APPLICATION Step 1: Identify the Task 1

In questions of science, the authority of a thousand is not worth the humble reasoning of a single individual. (Galileo) What does this statement imply? How does this idea contrast with the reality of modern science? Can you reconcile the value of the individual’s convictions with the need for peer review and recognition by the larger scientific community?

2

Wherever possible, you should avoid providing medical care to anyone with whom you have a close personal relationship.  (UK General Medical Council, Good Medical Practice 2009) Why should a doctor avoid providing medical care to those close to him? Under what circumstances should a doctor treat a close friend or relative? How can a doctor most effectively contribute to the care of those close to him?

3

Medicine sometimes snatches away health, sometimes gives it. (Ovid) What are the implications of this statement? Describe how clinical interventions introduce both hazards and benefits. How can we best assess the efficacy of a particular medical therapy?

My selection:

92

Lesson 4 Writing Basics

Steps 2/3: Brainstorm/Outline

Brainstorm Statement           Alternative

Thesis:

Outline

Paragraph 1



Paragraph 2



Paragraph 3

93

Lesson 4 Writing Basics

Step 4/5: Write/Review

Your answer must be contained within this area.

94

Question answered 

Lesson 4 Writing Basics

THE KAPLAN METHOD FOR WRITING STEP 1: Identify the task (2 minutes) STEP 2: Brainstorm (6 minutes) STEP 3: Outline (2 minutes) STEP 4: Write (17 minutes) STEP 5: Review (3 minutes)

ATTACKING THE WRITING TASK ·· Forget Section 1 and 2. ·· Brainstorm on paper, not in your head. ·· Write your outline in the space provided in the question booklet and refer back for each paragraph.

WRITING CHALLENGES ·· Building a strong argument ·· Coming up with relevant, constructive examples ·· Learning from your practice essays

95

S T R AT EG Y SE S SI O N 2

Pacing and Practising LEARNING OBJECTIVES By the end of this session, you will be able to:

• Identify question types that are conducive to triage. • Adjust your time management strategy, based on the remaining time in a given section. • Properly use your paper-based practice tests.

Strategy Session 2 Pacing and Practising

PACING The BMAT takes 2 hours to complete. Let’s consider the time pressures and related factors: ·· You have about a minute and a half to answer each question in Section 1. ·· You have about a minute to answer each question in Section 2. ·· You have 30 minutes to plan, write and proofread your essay in Section 3. ·· Time passes very quickly in each section—you must pace yourself to ensure you have enough time to answer all the questions. ·· The test is designed to get you to waste time on a difficult question or set—doing so ensures a low score. ·· Occasionally, the test will present data in a graph that looks unusual, or a table that looks enormous and scary, both of which are designed to make you panic or ‘freeze’—doing so ensures a low score. ·· You are unlikely to have much luxury time to go back and review/re-try questions in any section. Consider how much luxury time you will have in each section, if you follow Kaplan’s recommended pacing: Section 1

Qs

Critical Reasoning

15

Problem Solving

20

TOTAL

35

Section 2

Qs

Biology, Chemistry, Physics and Maths

27

Time/Q

Time Needed for Qs

Luxury Time

Time/Q

Time Needed for Qs

Luxury Time

In reality, are you likely to have much luxury time in either Section 1 or Section 2? Why or why not?

BMAT TEST TIP In recent years, the BMAT has included an unusual graph or very complicated table with one of the very first questions in Section 1. Many test-takers get caught out, and waste valuable time trying to work out how to read the data—or, worse, they panic. There’s an easy way to avoid this: if the data looks complicated or confusing, circle the question number in your test booklet, skip it and come back to it later. This is especially important if it’s the first question in Section 1, as you’d do much better to start your BMAT experience with a question that makes for quick work and an easy mark. Doing so will give you a psychological ‘edge’ over your competition, who aren’t prepared to take control of the exam.

99

Strategy Session 2 Pacing and Practising

SECTION MANAGEMENT Your Kaplan BMAT practice tests will allow you to practise for pacing, and also for section management. ·· Section management means managing your time and work so you can maximise your marks in each section. ·· Pacing guidelines are essential to section management. ·· Question triage is also essential. There are two general reasons to triage a question: ∘∘ You have spent the allotted time on the question, but are unsure of the answer. ∘∘ On first glance, you suspect that working out the answer will require more than the allotted time. ·· When a question is very difficult, or very time-consuming, don’t hesitate to Scoop and Run: ∘∘ Spend no more than 10-15 seconds attempting to solve. ∘∘ Mark an answer, mark for review and move on.

Triage: The Usual Suspects What kinds of questions are you most likely to triage? Question Type

Likely Candidates for Triage

Critical Reasoning Problem Solving Biology Chemistry Physics Maths

When Time Is Running Out ·· If you are running out of time in a section, adjust your time per question: ∘∘ Try to answer the remaining questions in half the usual time. ∘∘ This will require you to eliminate very quickly, based on minimal work/reasoning. ∘∘ If a question appears very difficult/time-consuming, guess without attempting any work and circle the question number in your test booklet. Come back to any such questions in any luxury time.

BMAT TEST TIP If you are running out of time in a section, maximise your marks on the questions that are easy-tomedium difficulty. There are very few high-difficulty questions (in terms of the level of time/work involved to answer correctly) in any one section, so don’t feel bad about triaging these when time is short. Students who triage successfully and ruthlessly earn higher scores than those who don’t.

100

Strategy Session 2 Pacing and Practising

PRACTISING WITH PAPER-BASED TESTS Practising with paper-based tests is essential to maximising your BMAT scores on Test Day. ·· Every time you practise with a test paper, try to simulate the Test Day experience. Some top tips: ∘∘ ∘∘ ∘∘ ∘∘ ∘∘

Use a printed copy of the test paper and the answer sheet. Mark questions that you skip in the test booklet. Practise ‘bubbling’ your answer sheet. Pace yourself, which is essential to maximise marks in each section. Train to build endurance, which is essential to combat fatigue—it’s common for students to feel especially tired after Section 2, so be prepared to take a ‘mini-break’ before starting the essay. ∘∘ Don’t skip the essay—complete the Writing Task with each full practice test you sit. Download all the official past papers from the BMAT website, and familiarise yourself with the following: ·· The directions for each section (from the front page of each test booklet)—these are always the same. ·· The directions for filling in the answer sheets—these are always the same. ·· The GCSE content covered in Section 2 questions—this can help guide your revision. Be careful to make the most of each practice test you complete. This means that you MUST: ·· Review all the explanations within a day or so of sitting the test. ·· Check the explanations for questions you got correct—make sure you got them right for the right reasons. ·· Revise any GCSE content that appeared in Section 2 that you did not remember quickly and accurately. ·· Take the time to review your essay, or have it peer-reviewed, or send it to Kaplan for feedback.

BMAT TEST TIP Most students find it helpful to sit one full BMAT practice test a week, in the weeks leading up to Test Day. You have five Kaplan BMAT practice tests, plus all the past papers from the BMAT website. That’s enough practice tests for more than 2 months! If you decide to sit more than one full BMAT practice test a week, make sure you’re allowing sufficient time to review each test in full before sitting the next one. Giving yourself a bit of time between practice tests will help you to ‘digest’ and analyse your performance fully, so you can optimise your efforts on your next BMAT.

101

LESSON 5

Advanced Critical Reasoning LEARNING OBJECTIVES By the end of this session, you will be able to:

• Apply the Kaplan Method for Critical Reasoning to Weaken, Strengthen, Inference and Explain Questions.

• Use the components of an argument to identify the assumption and valid strengtheners and weakeners.

Lesson 5 Advanced Critical Reasoning

KAPLAN METHOD FOR CRITICAL REASONING STEP 1: STEP 2: STEP 3:

Fundamentals of Arguments

·· C:

·· E:

·· A:

Warm-Up Exercise ·· The next two pages contain three Critical Reasoning questions. ·· Try to complete the exercise, and answer all the questions, in 5 minutes.

105

Lesson 5 Advanced Critical Reasoning

These days, every boy wants to grow up to be a professional footballer, and every girl wants to become a pop star. Young people assume that they are exceptionally talented, and that their singular talents will lead to fame and fortune. No-one seems to realise that only a very rare few will ever make it big; nearly all of us lead ordinary lives. Young people must learn sooner or later to accept their lot in life and find happiness in small things—in the things they have and can reasonably hope to attain. Footballers and pop stars! It wasn’t like that back in my day. We were happy just to be alive, and find our own way in the world. 1

Which one of the following best expresses the central assumption in the above argument? A

Young people think they are exceptionally talented, but are not.

B

Young people think they are exceptionally talented, but most are not.

C

Happiness comes from accepting you will never be rich and famous.

D

Boys and girls have different dreams.

E

Values change over time.

Exam boards have introduced A* grades at A-level because of the high number of pupils earning A grades in recent years. Despite these changes, research shows that certain subjects are more likely than others to reward a high proportion of A* grades. For instance, 24% of Latin pupils and 77% of classical Greek pupils are expected to win A* results this year, whilst only 2% of pupils sitting the A-level in English are likely to do the same. The likely reason for this is that Latin and classical Greek are more difficult subjects and therefore only taken by relatively few (and academically, largely exceptional) students—with total figures of 1000 and 200 per year, respectively—as compared to the 90,000 per year who sit the A-level in English, currently the most popular of all the A-levels. 2

106

Which one of the following can be most properly concluded from the argument above? A

This year, more A-level students will win A* grades in Greek and Latin than in English.

B

The number of students earning A grades on their A-levels is excessive.

C

Fewer students win A grades on popular A-levels, such as English.

D

This year’s results for A-levels in Latin and Greek are not likely to be disproportionately high.

E

More students are expected to earn an A* on the A-level in Greek than on the A-level in Latin.

Lesson 5 Advanced Critical Reasoning

The Japanese island of Okinawa has the largest per capita population of people aged 100 and over in the world. These centenarians experience longevity and good health at advanced ages due to their diet, which consists of mostly plant-based meals, prepared fresh every day. Centenarians on Okinawa do not eat processed food, preserved food or takeaways. Therefore, it follows that anyone who eats plant-based meals, prepared fresh, and stops eating processed and preserved food and takeaways, will live to age 100. 3

Which of the following, if true, most weakens the argument in the passage above? A

Okinawans live in remote, traditional communities, isolated from the stress of modern life.

B

Japanese people who eat takeaways sometimes live to age 100.

C

Okinawans who eat processed food do not live to age 100.

D

Japanese people who eat processed food do not live to age 100.

E

A plant-based diet has been proven to reduce stress and incidence of cancer and heart trouble.

STOP. IF YOU FINISH BEFORE TIME IS UP, CHECK ANY QUESTIONS YOU HAVE MARKED FOR REVIEW. YOU MAY GO BACK TO QUESTIONS IN THIS EXERCISE ONLY.

107

Lesson 5 Advanced Critical Reasoning

Weaken Questions Weaken questions ask you to weaken the argument’s assumption. ·· The first step is to find the conclusion and the evidence, and identify the assumption ·· Find the answer that makes the conclusion less likely to follow from the evidence ·· Make sure your weakener is attacking the assumption, and not the conclusion or the evidence

A Buddhist monastery in Nepal teaches its nuns kung fu. While the monastery is 800 years old, the nuns have only started learning kung fu recently. The kung fu practised by the nuns, most of whom are aged 30 or under, is just as vigorous as the kung fu you might expect to see in popular films. The nuns spend two hours a day doing kung fu, with profoundly positive impacts on their health and their meditation. It’s a shame that only one sect of nuns at one monastery have taken up kung fu, as their results clearly show that all nuns should practise kung fu for two hours a day. 4

Which of the following, if true, would most help to weaken the argument above? A

Buddhist nuns at another monastery developed long-term health problems from their daily kung fu exercises, which had to be curtailed.

B

The Buddhist nuns who practise kung fu run 3-5 miles a day, and eat an especially healthy diet that is high in fibre and low in fat.

C

The average age of nuns worldwide is 52, and nuns have a higher incidence of arthritis and osteoporosis than women who are not nuns.

D

The average age of Buddhist nuns worldwide is 27, and Buddhist nuns have a lower incidence of heart disease and obesity than Buddhist women who are not nuns.

E

Women who practise kung fu an average of 2 hours a day are calmer and happier than women who do not practise kung fu.

BMAT TEST TIP A weakener must attack the argument on the argument’s own terms. Thus, wrong answer choices on Weaken questions will usually include terms that do not match those in argument’s evidence or conclusion. Often, these answer can be eliminated because their terms are broader than those in the argument.

108

Lesson 5 Advanced Critical Reasoning

Leader: We must follow strict moral codes in our behaviour and appearance. Tens of thousands of our citizens have been killed in earthquakes over the last ten years. At the same time, we have seen many women start to dress immodestly. When women do not dress modestly, they lead men into adultery and other depravities, which cause earthquakes. 5

Which of the following would weaken the leader’s argument?

1 Shifting tectonic plates under the Earth’s surface cause earthquakes.

2 Hundreds of thousands died in a series of earthquakes between 10 and 20 years ago, during a time when all women dressed modestly. 3 A nearby country has had strict moral codes, and no earthquakes, in the last ten years. 4 Most women lost their homes and clothing in the earthquakes, which also destroyed all manufacturers of modest clothing. A

1 and 3

B

1 and 4

C

2 and 3

D

2 and 4

E

1, 2 and 3

F

1, 2 and 4

G

1, 3 and 4

H All

BMAT TEST TIP There are three ways to weaken a claim of causation (‘X caused Y’) ·· Show there’s another cause (‘Z caused Y’) ·· Show that cause and effect are reversed (‘Y caused X’) ·· Show that there is a correlation, but no causality (‘X and Y are not connected’)

109

Lesson 5 Advanced Critical Reasoning

Strengthen Questions Strengthen questions ask you to strengthen the argument’s assumption. ·· Strengthen questions are less common than Weaken questions, and the approach is nearly the same ·· The only difference: the correct answer to a Strengthen question will make the argument more likely to be correct

The world’s supply of chocolate comes from cacao trees, which must be grown near the Equator; unfortunately, this supply is in danger, thanks to an emerging fungal threat to cacao trees. A fungus called witches’ broom that spread quickly among cacao plantations in Brazil in the 1980s reduced its chocolate production by 80 per cent; now, another fungus called frosty pod rot is expected to attack Brazil’s cacao trees within the next year. Frosty pod rot is much more difficult to eradicate than witches’ broom. If frosty pod rot spreads to cacao trees in Africa, experts fear that a worldwide chocolate shortage will result. 6

Which one of the following pieces of evidence would strengthen the claim about fears of a worldwide chocolate shortage? A

Less than 5 per cent of the world’s chocolate is grown on cacao trees in Brazil.

B

Frosty pod rot reduced coffee production in Indonesia by 90 per cent.

C

Frosty pod rot can be avoided by moving cacao trees to higher altitudes.

D

Nearly all the land in Africa’s chocolate-producing countries is within 1 km of sea level.

E

Cacao trees in four countries in central Africa provide 70 per cent of the world’s chocolate.

BMAT TEST TIP Some strengtheners may seem too ‘easy’ or ‘obvious,’ as they simply assert that the assumption is true. Any evidence that confirms the assumption is a valid strengthener. Sometimes, then, you can find the strengthener simply by finding the assumption.

110

Lesson 5 Advanced Critical Reasoning

Fish and chips have been a staple of British life since the 19th century, and helped us defeat the Germans in the First World War. The wartime cabinet thought it essential to provide home comforts to keep the nation in good spirits during the long and difficult conflict, and safeguarding the supply of fish and chips was a top priority. Because of this Government initiative, the nation was well-fed and content, and as a result Britain won the war. 7

Which of the following, if true, would most help to strengthen the argument above? A

Britain won the First World War because of our superior military alliance.

B

The wartime cabinet did not safeguard the supply of tea, which had to be rationed.

C

The Germans did not safeguard the supply of their favourite foods during the war.

D

Eating fried foods, such as fish and chips, does not always make you feel full and content.

E

The Germans had a safe and regular supply of their favourite foods during the war.

BMAT TEST TIP Correct answers to Weaken and Strengthen questions will sometimes introduce information that is not part of the original argument. This is perfectly acceptable, so long as this information has the effect of weakening/strengthening the argument on its own terms. Answer choices that do not include any terms from the argument are wrong.

111

Lesson 5 Advanced Critical Reasoning

Inference Questions Inference questions ask you to find the answer that must be true, based on information in the argument. ·· You need the answer choice that must follow, not the choices that might or could follow ·· Be careful not to bring in any outside information ·· Be careful not to go beyond the terms of the argument

According to the British Crime Survey, the number of criminal offences in 2008 was 10 million. This reflects a drop of 48 per cent from the figures in 1995, which saw the highest number of criminal offences in recent decades. From these figures, one can conclude that Britain has experienced a significant decrease since the mid-1990s. 8

Which one of the following can be safely concluded from the information above? A

More people committed crimes in Britain in 1995 than in 2008.

B

The British Crime Survey is always administered on the same day of the week.

C

There were 18 million criminal offences in 1995.

D

There were more criminal offences in 1995 than in 2000.

E

More people were victims of crimes in 1995 than in 2008.

BMAT TEST TIPS Be careful not to confuse actual value with percentage. Any answer choice that goes beyond the terms of the argument cannot be a valid inference.

112

Lesson 5 Advanced Critical Reasoning

In the last ten years, more people than ever before have been consuming coffee made with a coffee capsule machine, which prepares a single cup of coffee from a small plastic or aluminium capsule containing the coffee grounds. The coffee capsules are topped with foil or special paper; since most coffee capsules contain a mix of different materials, they are not recyclable. For example, only one of the four most popular coffee capsule machines in the UK uses coffee pods that are biodegradable, resembling tea bags; other coffee pods sold in the UK cannot be recycled. This has led to a sharp rise in the amount of waste generated by coffee consumption, giving the coffee capsules something of a negative reputation: in a recent survey, 10% of Britons agreed that ‘coffee pods are very bad for the environment’, yet 22% of people in the same survey said that they own a coffee capsule machine. Sales of coffee capsules in the UK in 2015 increased by a third from the previous year, exceeding £112 million, and are expected to increase threefold by 2020, by which time coffee capsules could outsell tea bags. In Western Europe, coffee drinkers buy €6 million (£4.63 million) of coffee capsules per year, accounting for one-third of coffee sales. In the past five years, coffee sales in Western Europe have grown by 1.6% per year, but sales of coffee capsules have grown by 9% per year. 9

Which one of the following is a conclusion that can be drawn from the above passage? A

The rise in sales of coffee capsules in the UK is driven by people who are concerned about the environment.

B

People in the UK are expected to drink less tea in 2020 than they do today.

C

The rise in sales of coffee capsules in the UK is driven by the popularity of capsules that are biodegradable or recyclable.

D

In a recent survey, 2.2% of people in the UK said that they owned a coffee capsule machine, despite also stating that coffee pods are very bad for the environment.

E

The rise in sales of coffee capsules may be partially (but not entirely) explained by the overall rise in coffee sales.

BMAT TEST TIPS Sometimes, a valid inference will combine two pieces of information from the argument. Sometimes, a valid inference will describe information from the argument in different terms than those used in the argument. For example, a valid inference may make a broad point about specific information in the argument, or a valid inference may identify a trend based on data in the argument.

113

Lesson 5 Advanced Critical Reasoning

Explain Questions Explain questions ask you to explain some aspect of the data. Usually this will involve the logic of the argument, though it could also involve numerical data. ·· These questions will usually include the word explain or explanation; in most cases, they will include further keywords that you can use to find the relevant details in the data. ·· The correct answers to Explain questions are a bit like valid strengtheners. They can include info not mentioned in the original argument, but they must directly impact the terms of the original argument. ·· You may be asked to explain the discrepancy, which is a bit like attacking a Flaw question. The discrepancy isn’t exactly a flaw; rather, it’s something that seems inconsistent or illogical, but that will make sense if further information – the correct answer – is added to it. ·· It may help to underline the key terms in the part of the argument you are asked to explain. It may also help to find the assumption, though you may not need the assumption to work out the correct answer.

Austria changed its laws to allow up to two years of paid parental leave (from the previous maximum of one year) for children born in July 1990 and after. Research into the exam results of children born before and after this change in the law has revealed that taking a full year of maternity leave leads to higher exam results in children of mothers with one or more university degrees. Perhaps paradoxically, the same amount of maternity leave yields poorer exam results in the children of mothers who did not attend university. These results were valid for mothers who took a full year of maternity leave after the birth of the child; there was no notable effect on the children’s exam results if the mothers took two years of maternity leave. 10

Which of the following, if true, offers a convincing explanation for the divergent exam results for children of mothers that took a year of maternity leave after they were born? A

Parents who score highly in exams will tend to have children who score highly in exams.

B

Mothers who have completed higher education have an average of fewer than two children, while mothers who do not complete higher education have an average of three children.

C

Parents who have not completed higher education are less likely to foster a positive attitude towards education in their children than those who have completed higher education.

D

Mothers who take two years of maternity leave spend more time with their children before age 3 than mothers who take only a year of maternity leave.

BMAT TEST TIP Incorrect answers to Explain questions usually make the same errors: ·· They will be much broader than the details you are asked to explain. ·· They will be much narrower than the details you are asked to explain. ·· They are not logically linked to the details you are asked to explain.

114

Lesson 5 Advanced Critical Reasoning

There are 10,500 fewer pubs in the UK in 2016 than in the year 2000, a decline of 17%. The amount of beer sold in pubs, clubs and restaurants in the UK has fallen by 46% since 2000, whilst the amount of beer sold in UK supermarkets and off-licences has risen by 27% in the same period, surpassing beer sales in pubs, clubs and restaurants for every year since 2014. The drinking habits of British adults have also changed in the last two decades. In 2005, 65% of British adults reported drinking alcohol the previous week, but this figure dropped to 57% in 2016; 2% of all British adults switched from drinking alcohol the previous week in 2005 to never drinking alcohol in 2016. Some 10.6 million British adults, or 16.1% of the total population of the UK in 2016, never drink alcohol. 11

Which of the following, if true, would explain the apparent discrepancy in these figures?



1 Pub closures have caused people in the UK to drink less alcohol.



2 People in the UK were more likely to drink beer at home in 2016 than in 2000.



3 The proportion of people in the UK who drink alcohol less than once a week has increased.



4 The changes in beer sales in the UK are directly proportional to the declining number of pubs.

A

1 and 2 only

B

1 and 3 only

C

1 and 4 only

D

2 and 3 only

E

2 and 4 only

F

3 and 4 only

G

2, 3 and 4 only

H

none of them

BMAT TEST TIP An answer that contradicts the original information cannot explain it. Do not fret if you cannot spot a discrepancy when reviewing the argument or the data. Sometimes a discrepancy will be obvious; in many cases, it won’t. You will have to compare each answer to the argument to see if it offers an acceptable explanation. Focus on the details you are asked to explain, and it should be a relatively straightforward process.

115

Lesson 5 Advanced Critical Reasoning

LESSON 5 QUIZ (6 minutes)

Please read this page carefully, but do not start the quiz until you are told to do so. Speed as well as accuracy is important in this section. Work quickly, or you may not finish the quiz. There are no penalties for incorrect responses, only points for correct answers, so you should attempt all questions. All questions are worth one mark. You will have 6 minutes to answer the 4 questions in this quiz. Record your answers in this book. Show your choice clearly by circling the corresponding letter in your book. If you make a mistake, erase thoroughly and try again. Any rough work should be done in this book. At the invigilator’s prompt, turn the page and begin the practice quiz.

117

Lesson 5 Advanced Critical Reasoning

The birth rate in England has risen by 22 per cent since 2001, yet there is currently a shortage of 5,000 midwives in England. As a result, births by Caesarean now account for 25% of all births in England, a figure that has more than doubled in the past 20 years. The shortage of midwives is thought to have led to an increase in Caesarean births because many pregnant women in England do not have a chance to discuss their concerns and anxieties about childbirth with a midwife, and wrongly assume that giving birth by Caesarean will be easier, less painful, and allow a quicker recovery than natural childbirth. When pregnant women in England are informed by midwives who are able to answer their questions and concerns, almost all (except for those with health concerns that encourage delivery by Caesarean) will opt for a natural birth. 12

Which one of the following can be drawn as a conclusion from the passage above? A

There is no legitimate reason for a woman in England to opt for a Caesarean birth.

B

Twenty years ago, 12.5% of babies in England were delivered by midwives.

C

Midwives in England are trained to address every possible health concern of a pregnant woman.

D

It takes 4 to 6 weeks for the average woman to recover fully from giving birth by Caesarean.

E

None of the above.

Ingesting a small amount of cinnamon bark each day may help to prevent the onset of Alzheimer’s disease, according to a recently completed study. The professor who initiated the successful research into cinnamon decided to look into its abilities to deter illness because of a story from the Bible, in which cinnamon was included in ointments applied by priests that resulted in a curative effect. In this instance, an idea from a non-medical book led to a potentially preventative treatment; thus, ideas for possible food and drink to test for preventative effects should originate in non-medical literature. 13

118

Which one of the following, if true, most weakens the argument in the passage above? A

Research into the preventative effects of drinking red wine has never been undertaken by scientists who regularly read the Bible.

B

Because other spices have been known to inhibit viruses since pre-literate times, cinnamon had been tested for its ability to prevent transmission of HIV long before the start of the Alzheimer’s study.

C

Many of the researchers who discovered the preventative effects of drinking red wine liked to read romance novels in which the characters drank red wine.

D

All previous research into the preventative effects of cinnamon was initiated by the same professor who conducted the Alzheimer’s study.

E

Most medical researchers are unlikely to read non-medical literature on a regular basis.

Lesson 5 Advanced Critical Reasoning

Commuter trains (those travelling into the city centre on journeys starting 35 to 75 minutes away) are overcrowded when it’s snowing or raining heavily, when it’s rush hour, or when there’s a major sporting or cultural event in the city centre. So, if commuter trains were to run with extra carriages in times of snow and heavy rain, in the rush hour, and on days when there are major sporting events in the city centre, then they would not be overcrowded. 14

Which one of the following illustrates a flaw in the argument above? A

It overlooks, and fails to address, other reasons that trains may be overcrowded.

B

It wrongly assumes that weather conditions and reasons for passenger travel are necessary conditions for commuter train overcrowding.

C

A condition necessary for a certain result does not guarantee that result.

D

It overlooks the need to add extra commuter train capacity on days when there is a major cultural event in the city centre.

Right-wing newspapers and politicians are always carrying on about ‘broken Britain’, whilst their counterparts on the left are always fretting about the broken planet. The complaints are always backed up with stories and statistics about the evils of hooded youths and immigrants, or of global warming and capitalism, depending which side the complainer is on. In fact, there is little difference between these two sides. Everyone who buys into this way of thinking—which, these days, seems like most of the country—starts from a position of certainty that things are, in fact, grievously and horribly broken. Facts, such as they are, play a limited role in such views of the world. 15

Which one of the following best summarises the main conclusion of the argument? A

The modern world is broken.

B

Right-wing and left-wing politics are the same.

C

People like to think the worst.

D

Facts are irrelevant.

E

Statistics show that Britain is broken.

STOP. IF YOU FINISH BEFORE TIME IS UP, CHECK ANY QUESTIONS YOU HAVE MARKED FOR REVIEW. YOU MAY GO BACK TO QUESTIONS IN THIS QUIZ ONLY.

119

Lesson 5 Advanced Critical Reasoning

A FEW WORDS FROM THE EXPERTS Kaplan Method for Critical Reasoning STEP 1: ID the question type. STEP 2: Attack the argument. STEP 3: Eliminate and select.

NEXT STEPS ·· Finding the assumption is the key to answering most Critical Reasoning questions. The first step to finding the assumption is finding the conclusion. Focus on finding conclusion, evidence and assumption quickly and accurately as you work through each Critical Reasoning question in your Kaplan BMAT practice tests. ·· Don’t stress about Formal Logic—it won’t be a huge part of Section 1, but picking up the marks in the few Formal Logic questions you encounter will help to boost your Section 1 score. Look through the Section 1 past papers on the BMAT website to find the Formal Logic questions and see what exactly is involved. Most will simply require you to form a contrapositive correctly—though it may be from an advanced FL statement. ·· Practise attacking Critical Reasoning questions for speed as well as accuracy. With a bit of preparation, you should be able to answer most Critical Reasoning questions in a minute or less on Test Day.

BMAT TEST TIP Evidence + Assumption → Conclusion

120

Lesson 5 Advanced Critical Reasoning

ANSWERS AND EXPLANATIONS Answer Key 1. (C)

9. (E)

2. (D)

10. (B)

3. (A)

11. (D)

4. (C)

12. (E)

5. (F)

13. (B)

6. (E)

14. (D)

7. (C)

15. (C)

8. (D)

Explanations 1. (C) To find the assumption, first find the conclusion. The conclusion is the strongest statement of opinion, or the main point of the argument. Here, it comes near the middle: young people must learn to accept their lot in life and find happiness in small things. The evidence for this is that most young people dream of becoming footballers or pop stars, yet very few people ever become rich and famous due to their talents. The central assumption links the conclusion with the evidence, connecting the idea of happiness with something other than being rich and famous. The answer choice that does this is (C), so (C) is therefore correct. All the other answers restate various pieces of evidence (some more accurately than others), but none touches on the central assumption. 2. (D) This is an Inference question, as it’s asking us to make a conclusion from the argument (rather than asking us to find the conclusion in the argument). The argument concerns the introduction of A* grades at A-level, and a claim that some subjects are more likely to reward a higher proportion of A* grades. The evidence given suggests that while some A-levels, such as Greek and Latin, will likely have a higher proportion of A* grades, these A-levels are taken by a very small number of students, and these students tend to be academically exceptional. Thus, we can conclude that the results on these rarer A-levels are not really disproportional. Answer (D) matches this inference, and is therefore correct. Note that the argument does not actually state that the Greek and Latin results are not likely to be

disproportionate—because it’s not directly stated, but clearly implied, this deduction qualifies as an inference. Answers (A), (C) and (E) all make incorrect deductions based on the figures given in the argument, and answer (B) makes an extreme claim that is far stronger than the language in the argument; therefore, it cannot be inferred. 3. (A) This is a Weaken question, so start by finding the conclusion, indicated by the keyword therefore. The argument concludes that anyone who eats a plantbased diet, without processed food, preserved food or takeaways, will live to age 100. This is a bizarre claim, and we see in the evidence that the diet recommended here is followed by people on the island of Okinawa, which has a disproportionate population of centenarians. The assumption is that if you eat an Okinawan diet, you will live as long as Okinawans, even though you are not Okinawan. The obvious weakener here is that Okinawans do not live so long simply because of their diet, but that there are other factors in their lifestyle that contribute to longevity. The answer that gives us such factors is (A): if Okinawans live in remote and traditional communities, isolated from modern stress, that would be a factor in their longevity, and one you could not recreate for yourself simply by giving up meat and takeaways. A quick check of the other answers shows that none offers a weakener: (B) and (D) mention Japanese people, but our assumption is about Okinawans; (C) and (E) are strengtheners, as they make the assumption more likely to hold. The correct answer is therefore (A). 4. (C) This question asks for a weakener, so start by finding conclusion, evidence and assumption. The conclusion is at the very end: all nuns should practise kung fu two hours a day. The evidence has to do with a sect of Buddhist nuns in Nepal who do just that, with extremely positive results for their health and meditation. The assumption is that all nuns will have positive results for their health and meditation if they do kung fu for two hours a day. The correct weakener will attack this assumption, and show that it is less likely to be correct. Since the assumption is about all nuns, we want a weakener that’s about all nuns, and the only answer that gives us one is (C): if most nuns in the

121

Lesson 5 Advanced Critical Reasoning

world are older than the kung fu nuns, and have more bone and joint problems than women who are not nuns, as (C) suggests, then they would not necessarily benefit from daily kung fu; in fact, daily kung fu could be hazardous to their health! (C) directly attacks the assumption, so it is correct. As for the others: answers (A), (B) and (D) cannot weaken the assumption here, because they only address Buddhist nuns, rather than all nuns; answer (E) talks about women generally, but our assumption is only about nuns. 5. (F) This is a Weaken question, so start by finding the conclusion, which is the first sentence: we must follow moral codes in our behaviour and appearance. The evidence is a high number of earthquakes in the country in the last decade, and a claim of causality: immodestly dressed women cause immoral behaviour, which causes earthquakes. The assumption is that moral codes would alter this chain of causality. There are a number of ways to weaken a claim of causality. One is to show that something else caused the result. Statement 1 does so: if shifting plates under the Earth’s surface cause earthquakes, then moral codes will not stop them. Eliminate answers (C) and (D), which do not include Statement 1. Statement 2 functions in much the same way: if there were a series of earthquakes 10 years earlier than the most recent decade of earthquakes, and women dressed modestly at the time of these earlier, deadlier earthquakes, then women’s appearance is not necessarily connected to earthquakes. Eliminate (A), (B) and (G), which do not include Statement 2. Statement 3 says that a nearby country had strict moral codes and no earthquakes, but this would strengthen the leader’s argument, as it would solidify the link between immoral behaviour and earthquakes. Eliminate (H). The correct answer must therefore be (F), and indeed Statement 4 would weaken the claim of causality, by showing that cause and effect are reversed (eg the earthquakes caused women to be immodestly dressed). 6. (E) This is a Strengthen question, so start by finding conclusion, evidence and assumption. The conclusion is the final sentence: there will be a worldwide chocolate shortage if frosty pod rot spreads to cacao trees in Africa. The evidence is about two different fungi that have threatened cacao trees in Brazil. The obvious disconnect between evidence and conclusion is that the evidence is about cacao trees in Brazil, and the

122

evidence is about cacao trees in Africa. The conclusion also introduces the idea of a worldwide chocolate shortage, which is not necessarily linked to the evidence. The assumption, then, is that so much of the world’s chocolate comes from cacao trees in Africa that a worldwide shortage could result if these trees were damaged. A valid strengthener will make this assumption more likely to be true. The only answer choice that has the effect of strengthening the central assumption is (E): if 70% of the world’s chocolate comes from cacao trees in four African countries, then the assumption is more likely to be true. The answer is therefore (E). 7. (C) This is a Strengthen question, which works the same as a weaken question, only the answer that makes the assumption more likely to hold (rather than less likely, as on a weaken question). The conclusion is that Britain won the First World War because the nation was well-fed and content. The evidence is a Government initiative to safeguard the supply of fish and chips during the war, as home comforts were needed to keep the nation in good spirits. The assumption is that fish and chips kept the nation well-fed and content. Answers (A), (D) and (E) are weakeners; if true, each of these answers would make the assumption less likely to hold. (B) is irrelevant, as tea does not match the terms of fish and chips in the argument—tea may make you feel content, but it can hardly make you feel well-fed. (C) is the strengthener, as it links the German loss to a lack of favourite foods. If the Germans lost the war because they did not have a supply of home comforts, it’s more likely that the Brits won because we did have such a supply. The correct answer is therefore (C). 8. (D) This is an Inference question, so find the answer that follows from the argument. Eliminate any answers that don’t fit the argument, or that don’t necessarily follow. Answer (A) mentions the number of people who committed crimes in Britain, but the argument only mentions the number of criminal offences, so eliminate (A). (B) mentions the format of the survey. The argument assumes that the statistics from the survey are accurate, so we can infer that the survey must be representative in each year it was administered. Does this mean it must have been administered on the same day of the week each year? Not necessarily. (B) could be true, but we can’t infer that

Lesson 5 Advanced Critical Reasoning

it necessarily is true based on the argument, so eliminate (B). (C) says that there were approximately 18 million criminal offences in 1995. The argument says that the 10 million offences in 2008 reflected a drop of 48% from the figure in 1995. 48% is a little less than 50%, so calculate: 10 million ÷ 50% = 10 million ÷ 1/2 = 10 million × 2 = 20 million. The actual total for 1995 must be a little more than 20 million, as 10 million was a little less than half the total. (C) gives a lower figure, so eliminate (C). Answer (E) mentions the number of people who committed crimes, but our figures deal with the number of crimes—we can’t infer anything about the number of criminals, so eliminate (E). Answer (D) must be correct, and indeed we can see that the argument says that 1995 recorded the highest figure for criminal offences in recent decades; thus, we can infer that the figure in 2000 was lower. 9. (E) This is an Inference question, so find the answer that follows from the argument. The first answer states that the increase in coffee capsule sales in the UK is driven by people who are concerned about the environment, meaning that people concerned with the environment are buying coffee capsules in the UK. The final paragraph states that sales of coffee capsules increased by one-third from 2014 to 2015, which is a significant rise. However, the first paragraph states that only one of the four most popular coffee capsule machines in the UK uses biodegradable pods; the other coffee pods sold in the UK cannot be recycled, leading to a sharp rise in the amount of waste. For this reason, it does not follow that people concerned about the environment are driving the increased sales of coffee capsules, since they are bad for the environment. Eliminate (A). The second answer states that people in the UK are expected to drink less tea in 2020 than they do today. The year 2020 is mentioned in the first sentence of the final paragraph; we are told that coffee capsules ‘could’ outsell tea bags in the UK by 2020. However, this does not mean that people will be drinking less tea; it could simply be that they are drinking a lot more coffee. Eliminate (B). The information from the first paragraph that disqualified (A) also disqualifies (C), since 3 of the 4 most popular coffee capsules in the UK are not biodegradable or recyclable. (D) is a wrong answer trap, waiting for those who multiply the two percentages given in the last sentence of the first paragraph. However, this sentence actually states that 10% of Britons agreed

that coffee pods are very bad for the environment, whilst 22% of people in the same survey own a coffee pod machine. The 10% and the 22% could overlap wholly or partially, or they could be entirely different people among the Britons who answered that survey – we don’t know, so we can eliminate (D). Thus, answer (E) must be correct. Indeed, the information in the last sentence supports the inference drawn in (E): In the past five years, coffee sales in Western Europe have grown by 1.6% a year, but sales of coffee pods have grown by 9% a year. Hence, it follows that the overall popularity of coffee may explain the rise in coffee pod sales, but this cannot explain it entirely, since there has been a much greater rise in coffee pod sales than in coffee sales. 10. (B) This question asks you to explain the divergent exam results for children of mothers that took a year of maternity leave after they were born. The keyword paradoxically in the argument highlights the issue you must explain: mothers with at least one university degree who took a year of maternity leave had children with higher exam results, whilst mothers who took a year of maternity leave but did not attend university had children with poorer exam results. The paradox is that a year of maternity leave leads to divergent results for the children, seemingly based on the mother’s educational history. The argument links the two concepts (a year of maternity leave + children’s exam results), so we need an answer that would relate to both concepts. (A) does not relate to maternity leave, shifting the focus instead to parents’ exam results; (C) commits a similar error, shifting instead to whether parents have a positive attitude towards education. (D) mentions maternity leave, but shifts to a twoyear maternity leave, which the argument states did not link to a notable effect in exam results. Answer (B) must therefore be correct; indeed, (B) provides a rationale for the divergent results: if mothers with university degrees have fewer children on average than mothers without a university degree, then there could be more time and resources available to the children in smaller families. This would help to explain the divergent results in a way that is linked to maternity leave, since the mothers with larger families would be taking more maternity leaves (one per child). 11. (D) This Explain question is a bit harder, since the initial text presents a lot of data about drinking alcohol in

123

Lesson 5 Advanced Critical Reasoning

the UK, but it is not a proper ‘argument’ in that it does not have a conclusion or an assumption. Thus, you must check each statement against the data to see whether it fits; eliminate any that do not fit the data. Statement 1: We are not given figures about alcohol consumption in the UK, only relative rates of alcohol sales and people who did or did not drink in the previous week (or do not drink at all). Pub closures may have caused people in the UK to drink less, but we don’t know that people actually have been drinking less. Eliminate (A), (B) and (C). Statement 2: Beer sales in UK supermarkets and offlicences have risen 27% from 2000 to 2016, whilst beer sales in UK pubs, clubs and restaurants have fallen 46% in the same period. The clear inference is that people in the UK have significantly changed their beer-drinking habits, consuming less beer in pubs, clubs and restaurants and more at home. Statement 2 fits the data, so it would explain the discrepancy. Eliminate (F) and (H). Statement 3: From 2005 to 2016, the proportion of British adults who drank the previous week (when surveyed) dropped from 65% to 57%, a decline of 8%. That 8% includes 2% who switched to never drinking alcohol, so the other 6% must still drink alcohol, but not every week. Statement 3 fits the data; eliminate (E). Statement 4: There are 17% fewer pubs in the UK in 2016 than in 2000; at the same time, beer sales in pubs, clubs and restaurants declined by 46%, whilst beer sales in supermarkets and off-licences increased by 27%. We can’t say that the changes in beer sales are proportional to the declining number of pubs, because the beer sales in pubs, clubs and restaurants declined nearly 3 times the rate of pub closures, whilst beer sales in supermarkets and off-licences actually grew by more than 1.5 times the rate of pub closures. Statement 4 does not fit. The answer is (D). 12. (E) This is an Inference question, so check each answer against the argument to see if it is a valid inference. The argument suggests that too many women in ­England are having Caesarean births unnecessarily, but states in the final sentence that there are health reasons that encourage women to opt for a Caesarean birth. (A) contradicts this, so it can not be correct. The argument gives no statistics about the percentages of babies in England delivered by midwives now or 20 years ago, so (B) is not a valid inference. (C) makes

124

a very strong claim about the training of midwives in England, but this is not supported by the argument. (D) is a factually correct statement about the length of time usually required to recover fully from a Caesarean birth, but there is no way to infer this based on information in the argument. Answer (E) is correct. 13. (B) Since this is a Weaken question, the first step is to find conclusion, evidence and assumption. The conclusion is introduced by the keyword ‘thus’: ideas for medical research into food and drink should originate in non-medical literature. That’s an unusual conclusion, and it’s supported by evidence about research into cinnamon, which can potentially help to prevent Alzheimer’s disease; the researcher got the idea to look into cinnamon’s preventative properties from the Bible. The assumption is that ideas for successful medical research can originate in non-medical literature. Anything that attacks the likelihood of this assumption is a weakener. (A) may seem tempting at first, but it doesn’t say that the idea for researching red wine came from somewhere other than the Bible; since it doesn’t attack the argument on its own terms, it can’t be a weakener. (B), however, severs the link between the original idea for researching cinnamon and the Bible, or any other literary sources: if researchers started looking into cinnamon’s preventative properties due to pre-literate knowledge of spices, then the research is not connected to non-medical literature. If (B) were true, the argument would fall apart. (B) is therefore correct. 14. (D) The first sentence in this Flaw question lists several occasions when commuter trains are overcrowded: when it’s snowing or raining heavily; when it’s rush hour; when there’s a major sporting event or a major cultural event. The conclusion (indicated by the keyword ‘so’) addresses all these occasions except major cultural events. Overlooking this problem is a flaw in the argument, as indicated by correct answer (D). 15. (C) This is a Conclusion question, so scan the argument for the conclusion. The conclusion keywords in this case are ‘in fact’, so the conclusion is that there is little difference between the two sides, the political right-wing and left-wing, both of which think things are broken. None of the answers matches this exactly,

Lesson 5 Advanced Critical Reasoning

so eliminate those that get it wrong. The conclusion does not find that things are in fact broken, so eliminate (A) and (E). The last sentence says that facts play a limited role for people who think things are broken, but this is not the conclusion, so eliminate (D). Answer (B) may be tempting, but the argument gives the differences between right and left-wing politics; it does not say they are the same, so eliminate (B). The correct answer is therefore (C), and this fits with the main point of the argument: the two sides are very similar, as both like to think the worst about the modern world.

125

LESSON 6

Advanced Problem Solving LEARNING OBJECTIVES By the end of this session, you will be able to:

• Convert percentages into fractions. • Use the Kaplan Methods to solve Standalone Data Questions and Multi-Question Sets.

Lesson 6 Advanced Problem Solving

KAPLAN METHOD FOR CALCULATE QUESTIONS STEP 1: STEP 2: STEP 3: STEP 4:

THE KAPLAN METHOD FOR SPATIAL AND REPEATING QUESTIONS STEP 1: STEP 2: STEP 3: STEP 4:

Minimise Maths ·· Estimate: ·· Eyeball: ·· Eliminate:

Warm-Up Exercise ·· The next two pages contain three Problem Solving questions. ·· Try to complete the exercise, and answer all the questions, in 5 minutes.

127

Lesson 6 Advanced Problem Solving

1

I have four pets: a dog called Barnaby, a bird called Calypso, a cat called Darwin and a horse called Evergreen. Their birthdays (in alphabetical order) fall on the 121st, 197th, 263rd and 309th days of any year that is not a leap year.

Which two pets will have their birthdays on the same day of the week in any year that is not a leap year?

2

128

A

Barnaby and Calypso

B

Barnaby and Darwin

C

Barnaby and Evergreen

D

Calypso and Darwin

E

Calypso and Evergreen

F

Darwin and Evergreen

A quantity of sugar is to be divided into five packages. Each package (except for the smallest) must weigh twice as much as another package; no two packages may be of the same weight. If the weight of the smallest package (in kg) is an integer, which one of the following could be the weight of the total quantity of sugar? A

90 kg

B

93 kg

C

96 kg

D

99 kg

Lesson 6 Advanced Problem Solving

The graph below shows the percentage of people who responded ‘Yes’ when asked if they regularly took the environment/energy saving actions listed.

Using energy-saving light bulbs All windows double-glazed Insulation on hot water tank Never leave TV on standby Never leave lights on Never overfill kettle Take own shopping bag 0%

3

20%

40%

60%

80%

100%

Which of the following statements can be safely concluded from the data above?

1 Most people never leave the lights on and never overfill the kettle.



2 Most people use energy-saving lightbulbs and have insulation on their hot water tanks.



3 Some people who use energy-saving lightbulbs also have double-glazed windows.

4 Some people who never leave the lights on also take their own shopping bag. A

1 and 2

B

1 and 3

C

1, 2 and 4

D

1, 3 and 4

E

2 and 3

F

2 and 4

G

2, 3 and 4

H

3 and 4

STOP. IF YOU FINISH BEFORE TIME IS UP, CHECK ANY QUESTIONS YOU HAVE MARKED FOR REVIEW. YOU MAY GO BACK TO QUESTIONS IN THIS EXERCISE ONLY.

129

Lesson 6 Advanced Problem Solving

Data Questions Some Data questions will be grouped into sets (usually sets of 4), and some will appear on their own. ·· Data questions are either Calculate questions or Critical Reasoning questions, and thus require the same skills, methods and strategies. ·· It’s usually best to answer single Data questions as they come up, just as you would for other Problem Solving and Critical Reasoning questions. ·· If a Data question is a Critical Reasoning question, you must complete the reasoning task based on the data. Usually, this will involve making a valid inference from the data.

Question 4 is based on the following information. Household spending (as a percentage of income) in 1971 and 2007 is shown in the table below. 1971

2007

7%

3%

Recreation and culture

10%

12%

Clothing and footwear

9%

5%

Restaurants and hotels

10%

12%

Transport

10%

15%

Housing, water and fuel

15%

21%

Other

18%

23%

Food and non-alcoholic drinks

21%

9%

Alcoholic drinks and tobacco

4

Which one of the following can safely be concluded from the data given in the table? A

People spent more than twice as much on alcohol and tobacco in 1971 as in 2007.

B

Restaurant meals were more expensive in 2007 than in 1971.

C

Transport was a greater proportion of household spending in 2007 than in 1971.

D

Households spent less of their annual income on drinks in 2007 than in 1971.

E

Households spent less of their annual income on recreation and culture in 2007 than in 1971.

BMAT TEST TIP Be careful to distinguish between percentages and actual values.

130

Lesson 6 Advanced Problem Solving

Questions 5–6 refer to the following information: There has been an explosion in the popularity of selfies—photographs taken of oneself while holding up the digital camera or smartphone that is snapping the shot—which are then shared on social media. Unfortunately, there’s been an accompanying trend: the rise in selfie fatalities. According to data from the start of 2014 through the end of the following year, there were a total of 49 cases of people who died while taking a selfie. The most common causes of death were falling from a height (16 deaths) and drowning (14 deaths); accidents involving trains (8 deaths) and plane and car crashes (2 deaths each) were also reported. Of the selfie deaths reported in this time period, there were 36 males and 13 females among the fatalities. All of the selfie fatalities were aged 32 or under; the breakdown by age is included in the table below. Age

% of Selfie Fatalities

14–17

14%

18–20

27%

21

27%

22–23

16%

25–32

14%

There were no reported fatalities of people aged 24, 26, 28 or 30.

The problem has become so severe in Mumbai—where 14 of the 49 selfie fatalities have occurred— that the city has imposed 16 ‘no-selfie zones’ in areas that have are dangerously attractive to selfie takers. 5

6

Which of the following is a safe inference to draw from the above data?

1 More than half of all people who died taking selfies in a two-year period were aged 18 to 21.



2 The ratio of female to male selfie fatalities in 2014–2015 was less than 1 to 3.



3 If someone who died taking a selfie in 2014 or 2015 were chosen at random, they would be more likely not to have died in Mumbai than to have died in Mumbai.

A

1 only

C

3 only

E

1 and 3 only

G

1, 2 and 3

B

2 only

D

1 and 2 only

F

2 and 3 only

H

None of the statements

The average age of a selfie fatality in the 25–32 age group in 2014 and 2015 was 28.7 years, and three of the ages in this age group that had at least one selfie fatality had the same number of selfie fatalities. If this information is correct, then which of the following could be the number of selfie fatalities for each age in the 25–32 age group that had selfie fatalities in 2014 and 2015?

A B C D E

25 1 1 2 2 2

27 1 1 1 1 1

29 1 2 1 1 1

31 2 1 1 2 1

32 2 3 2 2 3

BMAT TEST TIP x% of y = y% of x

131

Lesson 6 Advanced Problem Solving

Multi-Question Sets BMAT Section 1 normally includes three multi-question sets, with 4 questions based on the same data and text. Some sets may include 3 or 5 questions. These multi-question sets usually account for one-third of Section 1. ·· These multi-question sets usually account for one-third of Section 1 questions (12 out of 35). ·· A multi-question set may include Critical Reasoning or Problem Solving questions. Inference and Calculate questions are most common. ·· Your percentage, fraction and algebra skills will serve you well in working through these sets. ·· When a question involves a percentage, convert it into an easy-to-work-with fraction.

Questions 7–10 refer to the following information: The table below shows the results of the May 2015 parliamentary election in England, Scotland and Wales. Parties that did not win any seats in Parliament are included in the ‘Others’ category. Seats

Gain

Loss

Net

Votes

% of Vote

+/–% from 2010 Vote

Conservative

331

35

11

+24

11,334,576

37.8

+0.8

Labour

232

22

48

–26

9,347,304

31.2

+1.5

Party

56

50

0

+50

1,454,436

4.9

+3.1

Liberal Democrat

8

0

49

–49

2,415,862

8.1

–15.2

Plaid Cymru

3

0

0

0

181,704

0.6

0.0

UKIP

1

1

0

+1

3,862,775

12.9

+9.5

Green Party

1

0

0

0

1,150,791

3.8

+2.8

0

0

0

0

232,284

0.8



Scottish National Party

Others TOTALS

632

29,979,732

A total of 66.35% of eligible voters in England, Scotland and Wales turned out to vote in the 2015 election. The results of the election came as a surprise, since no polling prior to the date of the election indicated that the Conservatives would win a majority. Prior to the election, there had been widespread speculation about which parties would (or wouldn’t form a coalition), as all the polling indicated that the election would result in a hung parliament. The day before the election, the final polls predicted that the Conservatives would win 273–286 seats, Labour 262–274, the SNP 46–56, and the Lib Dems 21–28 (with 326 seats required to win an outright majority). At about 10pm the night of the election, the TV networks broadcast the results of the exit polls (based on voters telling pollsters who they had just voted for, as they left the polls), which predicted the Conservatives would win 316 seats, with Labour winning 239. The exit polls shocked the nation, since they diverged so significantly from the pre-election polling. However, while the exit polls were more accurate, even they underestimated the strength of the Conservative victory. Why were the pre-election polls so wide of the mark, when the exit polls were more reliable? The most common explanations have to do with the polling methodology, including whether the surveys were conducted online or by phone— traditionally, most pollsters only survey voters with landline phones, which can oversample certain populations. Most pre-election polls were conducted by landline phones; the rest were conducted online.

BMAT TEST TIP Percentage change = difference original

132

Lesson 6 Advanced Problem Solving

7

How many people in England, Scotland and Wales were eligible to vote in the 2015 election? A 43,448,889 B 45,183,730 C 46,843,331 D 47,586,876

8

9

10

What was the ratio of Labour to Scottish National Party seats after the 2010 election? A

25 to 13

B

29 to 7

C

37 to 2

D

43 to 1

Which one of the following could be an alternative explanation of the reason that the pre-election polls were not as accurate as the exit polls conducted on the day of the election (‘Why were the pre-election polls so wide of the mark…’)? A

The exit polls were more accurate because they were conducted once the voters were fully informed about the issues that would decide the results of the election.

B

The pre-election polls were less accurate because most voters had not made up their minds.

C

The exit polls were more accurate because they were conducted by calling people on a landline telephone.

D

The pre-election polls were less accurate because they did not include enough voters who voted for the SNP in the election.

E

The exit polls were more accurate because they surveyed people who had actually voted.

What was the net percentage decrease in seats held by the Liberal Democrats? A 81% B 83% C 86% D 88%

BMAT TEST TIPS When the data involves multiple dates, double-check the date involved in each question before you set up and solve. Ratios and proportions are fractions in a different format. Rewrite as a fraction before doing maths.

133

Lesson 6 Advanced Problem Solving

Minimising Maths Be sure to use the ‘Three Es’—Estimate, Eyeball and Eliminate—throughout Section 1. Some further tips: ·· Convert percentages and decimals to fractions BEFORE doing any further maths. ·· When in doubt, write a percentage as a value over 100. Round so it will reduce. Example: 27.7% = 28

100

·· When a question involves solving for an unknown, set it up as an equation before doing any maths, to ensure that you put the variable in the right place. ·· Be ready to divide, but only do enough division to ‘see’ the significant digits in the correct answer. ·· Sometimes, you can avoid long division by using fractions. For example, instead of dividing by 12, 1 you could multiply by . 12

Questions 11–12 refer to the following information: The Government has set a target of reducing net migration—the difference between people moving to and moving from the UK—to less than 100,000 per year by 2020. However, the latest figures show that net migration was 323,000 in 2015, so there is some way to go towards meeting the target. Net migration is driven by the migration of EU citizens to the UK—172,000 of these people came to live in the UK in 2015, and of course, they have freedom of movement, so these figures are virtually impossible to control. Even so, it means that nearly half of the total migration to the UK in 2015—46.7%, or 151,000 people—came from countries outside the EU, where the Government can restrict entry. Of the migrants from the EU, there were 165,000 who came to the UK for work-related reasons— 58% of these people had a job to go to, whilst the other 42% came to the UK in the hopes of finding a job. Excluding UK citizens, the number of EU nationals employed in the UK in 2015 increased to 2 million, of which 10.75% were employed in the UK for the first time in that year. At the same time, the number of non-EU nationals employed in the UK increased by 38,000, to 1.2 million. 11

How many of the EU nationals (excluding UK citizens) who were employed in the UK in 2015 started their UK employment in that year? A 215,000 B 217,500 C 220,000 D 221,500

12

How many non-EU nationals were employed in the UK for the first time in 2015? A 3.125% B 3.143% C 3.167% D 3.182%

BMAT TEST TIP When your initial equation involves one or more fractions, be careful not to invert the numerator and denominator when you set up and solve. This may seem obvious, but it is easy to make a mistake when you are rushing, particularly when there is a variable in your fraction. Wrong answer traps will be waiting for those who ‘flip’ their fractions in the initial set-up.

134

Lesson 6 Advanced Problem Solving

Questions 13–14 refer to the following information: The following data shows the journeys to school taken by children aged 5 to 16 in the UK. 60%

50%

40% Ages 5–10 Ages 11–16

30%

20%

10%

0%

Walk

Car

Bus

Other

‘Other’ includes journeys to school by cycle and rail.

There are 4 million schoolchildren aged 11–16, and 4.5 million schoolchildren aged 5–10.

13

How many children walk to school? A 1,510,000 B 2,645,000 C 3,935,000 D 4,155,000

14

What percentage of children travel to school by car? A 24% B 27% C 31% D 35%

BMAT TEST TIPS Calculating an exact figure in millions is a waste of time. The first few digits will be sufficient to find the correct answer.

135

Lesson 6 Advanced Problem Solving

LESSON 6 QUIZ (6 minutes)

Please read this page carefully, but do not start the quiz until you are told to do so. Speed as well as accuracy is important in this section. Work quickly, or you may not finish the quiz. There are no penalties for incorrect responses, only points for correct answers, so you should attempt all questions. All questions are worth one mark. You will have 6 minutes to answer the 4 questions in this quiz. Record your answers in this book. Show your choice clearly by circling the corresponding letter in your book. If you make a mistake, erase thoroughly and try again. Any rough work should be done in this book. At the invigilator’s prompt, turn the page and begin the practice quiz.

137

Lesson 6 Advanced Problem Solving

Questions 15 to 18 refer to the following information: The table below gives figures on road deaths and drink-driving in various countries in the European Union. The Czech Republic is the only country in the EU with a total ban on drink-driving. Country

Estonia

Ireland

France

UK

Sweden

Germany

Czech Republic

1.34

4.42

62.49

61.38

9.26

82.5

10.51

Road Deaths per Million Inhabitants

98

77

67

43

45

54

106

Percentage of Road Deaths Attributable to Alcohol

48%

37%

27%

17%

16%

12%

3.4%

Maximum Legal Blood Alcohol Concentration (mg per 100ml)

0.2

0.8

0.5

0.8

0.2

0.5

0.0

Population (Millions)

Source: World Health Organisation and European Transport Safety Council—2008

15

How many road deaths in France were attributable to alcohol in 2008? A 1130 B 1240 C 1320 D 1410

16

138

What can you safely infer about a total ban on drink-driving, based on the data from the Czech Republic, as compared to the data from other countries without a total ban? A

A total ban has no effect on the number of road deaths.

B

A total ban has no effect on the number of road deaths attributable to alcohol.

C

A total ban significantly reduces the number of road deaths attributable to alcohol.

D

A total ban significantly reduces the rate of road deaths.

E

A total ban significantly reduces the rate of road deaths attributable to alcohol.

Lesson 6 Advanced Problem Solving

Questions 15 to 18 refer to the following information: The table below gives figures on road deaths and drink-driving in various countries in the European Union. The Czech Republic is the only country in the EU with a total ban on drink-driving. Country

Estonia

Ireland

France

UK

Sweden

Germany

Czech Republic

1.34

4.42

62.49

61.38

9.26

82.5

10.51

Road Deaths per Million Inhabitants

98

77

67

43

45

54

106

Percentage of Road Deaths Attributable to Alcohol

48%

37%

27%

17%

16%

12%

3.4%

Maximum Legal Blood Alcohol Concentration (mg per 100ml)

0.2

0.8

0.5

0.8

0.2

0.5

0.0

Population (Millions)

Source: World Health Organisation and European Transport Safety Council—2008

17

Which country experienced the most road deaths in 2008? A

Czech Republic

B Estonia C France D Germany

18

How many 2008 road deaths in the Czech Republic were not attributable to alcohol? A 1023 B 1076 C 1114 D 1159

STOP. IF YOU FINISH BEFORE TIME IS UP, CHECK ANY QUESTIONS YOU HAVE MARKED FOR REVIEW. YOU MAY GO BACK TO QUESTIONS IN THIS QUIZ ONLY.

139

Lesson 6 Advanced Problem Solving

A FEW WORDS FROM THE EXPERTS Kaplan Method for Calculate Questions STEP 1: ID the target. STEP 2: Research relevant data. STEP 3: Set up and solve. STEP 4: Eliminate and select.

Kaplan Method for Spatial and Repeating Questions STEP 1: ID the target. STEP 2: Check components. STEP 3: Count/Match/Rotate. STEP 4: Eliminate and select.

NEXT STEPS ·· When you encounter a data question, check whether it’s a Critical Reasoning or Problem Solving question, then follow the appropriate method. Most Critical Reasoning questions involving data are Inference questions—the correct answer is something that must be true, based on the data. Anything that could be true, or might not necessarily be true in all circumstances, is a wrong answer. ·· When you encounter a multi-question set, it’s usually best to mark an answer for all questions in the set before moving on. Work through the questions in the set in any order you like, but it’s not advisable to skip questions and come back to them on a second pass through the section. If you do so, you will waste valuable time re-reading and re-analysing the data. You will almost certainly miss out easier marks elsewhere in the section. ·· It might be helpful to work through the Problem Solving questions in the Section 1 past papers on the BMAT website, so you can practise solving by estimating/eyeballing and using fractions. It’s OK to do this as untimed practice—even if you have already tried sitting the past papers as practice tests—as it will help you to learn to ‘see’ the best ways of finding the fractions that make quick work of actual BMAT questions.

BMAT TEST TIP All the calculations required on the BMAT can be completed in a minute or two, without a calculator—so long as you are prepared to use fractions and the three E’s.

140

Lesson 6 Advanced Problem Solving

ANSWERS AND EXPLANATIONS Answer Key 1. (E)

7. (B)

13. (C)

2. (B)

8. (D)

14. (C)

3. (E)

9. (E)

15. (A)

4. (C)

10. (C)

16. (E)

5. (E)

11. (A)

17. (D)

6. (C)

12. (C)

18. (B)

Explanations 1. (E) This repeating question involves days of the week, but it is not clear which day of the week is which. However, whichever day starts the year—as the 1st day—will occur again as the 8th day, the 15th day, 22nd day, and so on. A week equals 7 days, so you can find a particular numbered day’s position in the week by dividing by 7, and comparing the decimals. The two pets with the same decimals will have birthdays on the same day of the week. Barnaby was born on the 121st day of the year; 121 ÷ 7 = 17.28. Calypso was born on the 197th day of the year; 197 ÷ 7 = 28.14. Darwin was born on the 263rd day of the year; 263 ÷ 7 = 37.57. Evergreen was born on the 309th day of the year; 309 ÷ 7 = 44.14. Answer (E) is correct.

our graph. Statement 1 says that most people never leave the lights on (50%) and never overfill the kettle (62%). Most means a majority, and 50% is not a majority, so we cannot infer Statement 1 from the data. Eliminate answers (A), (B), (C) and (D). Statement 2 says that most people use energy-saving light bulbs (78%) and have insulation on their hot water tanks (90%). It’s not clear how many people answered ‘Yes’ to both questions, so let’s assume that the 22% who do not use energy-saving light bulbs are part of the 90% who do have insulation on their hot water tanks. That would leave 90% – 22% = 68% of people who have insulation on their hot water tanks and also use energy-saving light bulbs, and 68% is a majority. Statement 2 is a valid inference; eliminate (H), which does not include it. Statement 3 says that some people who use energy-saving light bulbs (78%) also have double-glazed windows (72%). Since more than 50% of people answered ‘Yes’ to both questions, there must be some people who belong to both groups, so Statement 3 is a valid inference. Eliminate (F). Statement 4 says that some people who never leave the lights on (50%) also take their own shopping bag (38%). Since neither figure is more than 50%, we cannot infer that anyone necessarily answered ‘Yes’ to both questions. Statement 4 is not a valid inference, so eliminate (G). The correct answer is therefore (E).

2. (B)

4. (C)

To find the total quantity of sugar, work out the relative weights of the five packages. Each package except the smallest must weigh exactly twice as another package, and no two packages may have the same weight. Thus, if the smallest package has a weight of x, the next largest weighs 2x, and the next three largest would weigh 4x, 8x, and 16x, respectively. The total quantity of sugar weighs x + 2x + 4x + 8x + 16x = 31x. If the weight of the smallest package is an integer, then the value of x must be an integer. This means that 31x must be a multiple of 31. The only answer that is a multiple of 31 is (B), so (B) is correct.

This is an Inference question, so eliminate any answers that do not follow from the data. Answer (A) may seem correct at first glance, but there are two problems with its wording. (A) mentions people, but the data is about households. More problematically, we don’t know anything about the actual amounts spent from the data—only the relative amounts spent. Eliminate (A). (B) mentions the cost of restaurant meals, but we only have data on proportional household spending, so eliminate (B). (C) makes an accurate observation about proportional household spending on transport, which matches the data. (C) looks like a winner. Quickly check (D), which talks about spending on drinks. Note that drinks fall into two categories—alcoholic and non-alcoholic. There is not enough data to infer this statement, so (D) cannot be correct. (E) does not match the data, as spending on recreation and culture was a greater share of annual income in 2007 than in 1971. The answer is therefore (C).

3. (E) This data question is trickier than it may first seem, once you realise that people who responded to the survey may have responded ‘Yes’ to more than one question, and thus some of the same people may (or may not) be included in more than one of the bars in

141

Lesson 6 Advanced Problem Solving

5. (E) This is an Inference question, so evaluate each of the numbered statements in turn, eliminating answers that include any statements that are not a safe inference from the data. The first statement is a valid inference, because 27% of selfie fatalities were in the 18–20 age group and 27% of the selfie fatalities were among people aged 21. Thus, 54% of selfie fatalities in a two-year period (the data covers 2014 and 2015) were aged 18 to 21. Eliminate (B), (C), (F) and (H), which do not include statement 1. The paragraph above the table states that there were 36 male and 13 female fatalities in the two years covered by the data; thus, the ratio of female to male fatalities was 13 to 36. Statement 2 says that this ratio was less than 1 to 3, so compare the two: 36 is a little bit less than 39, 13

1

and 39 reduces to 3 . Comparing the two values, it is 13 13 1 clear that 36 is greater than 39 , since 36 is greater 1

than 39 . Hence, the ratio of female to male fatalities was greater than 1 to 3, and statement 2 is not a valid inference. Eliminate (D) and (G). Statement 3 refers to the number of selfie deaths that were not in Mumbai versus those that occurred in Mumbai. There were 49 selfie fatalities in the 2 years covered by the data, and 14 of these occurred in Mumbai. Thus, a majority of the selfie fatalities (33) took place somewhere other than Mumbai, so if you chose one of these fatalities at random, they would be a little more than twice as likely to have died somewhere other than in Mumbai than to have died in Mumbai. Statement 3 is a valid inference; answer (E) is correct. 6. (C) This question asks for a possible distribution of the number of selfie fatalities for each age in the 25–32 age group that had fatalities. The information under the table states that there were no selfie fatalities of people aged 26, 28 or 30, so there are only 5 ages in the 25–32 age group that had selfie fatalities; furthermore, this question tells us that three of those ages had the same number of fatalities. Looking at the answer choices, it will be the case that it’s either three ages with 1 fatality, or three ages with 2 fatalities. The average age of the fatalities in this age group is 28.7; you could multiply this by the total number of fatalities in this age group to determine the total age of all these people, which would be useful to know in evaluating the answer choices. There were a total of 49 selfie fatalities that are broken down by age group

142

in the table, so 14% × 49 of these people are in the 25–32 age group. Since you don’t have a calculator, it’s probably easiest to think of this as 49% of 14, and round 49% to 50%: 50% of 14 is 7. Thus, there must be 7 people total in the 25–32 age group. After quickly summing up the number of people in each answer choice, you can eliminate (B), (D) and (E), since each includes 8 people. Next, you could calculate the total age of all 7 people, or you could eyeball the average by drawing a vertical line through the answer choices. The average age is 28.7, so draw a vertical line through the answers just to the left of the numbers in the 29 column. This line represents the average; for the average to be properly balanced, you’d need roughly the same number of people on each side of the line, and they’d need to be roughly evenly spaced. Too many people on either side, and the average would clearly be higher/lower than 28.7. This is sufficient to eliminate (A), since 5 of the 7 people have ages higher than the average; thus, the average age of those people must be higher than 29. By comparison, the 7 people in (C) are evenly spaced across the ages, with 2 each in the youngest and oldest ages. Therefore, it’s fairly evident that the average would come out very near the line. Answer (C) is correct. For the record: If you did not see this and tried to solve by calculating, you would have multiplied the average age by the number of people: 28.7 × 7 = 200.9, which rounds to 201. Each of the remaining answers has 1 person of each age, except for age 32, which has 2, so add up the 6 people that are common to both answers: 25 + 27 + 29 + 31 + 32 + 32 = 176. Subtracting these 6 people from the total age: 201 – 176 = 25, so the seventh person must be aged 25. Answer (C) is correct. 7. (B) Estimate to make quick work of the maths here. We’re told that 66.35% of eligible voters in England, Scotland and Wales voted in the 2015 election, and the total turnout was 29,979,732, which is just a little bit less than 30,000,000. Similarly, you can round the percentage up to 66.67%, or two-thirds. To find the total number of eligible voters, divide: 30,000,000 ÷ 2 3 90,000,000 = 30,000,000 × 2 = = 45,000,000. The 3 2

answer must be slightly larger than 45,000,000, since 30,000,000 is a smaller percentage, 66.35%, of the correct answer. Thus, answer (B) is correct.

Lesson 6 Advanced Problem Solving

8. (D) The target here is the ratio of Labour to SNP seats after the 2010 election. Labour had a net loss of 26 seats after the 2015 election, so they had 232 + 26 = 258 seats after the 2010 election. The SNP had a net gain of 50 seats after the 2015 election, so they had 56 – 50 = 6 seats after the 2010 election. The ratio of Labour to SNP seats after the 2010 election is therefore 258 to 6. This reduces to 129 to 3, and further reduces to 43 to 1, answer (D). Note that wrong answer trap (B) was waiting for those who calculated the ratio of Labour to SNP seats after the 2015 election, which is 232 to 56; both sides are divisible by 8, so the simplest form of this ratio is 29 to 7. 9. (E) This Critical Reasoning questions asks for an alternative explanation for why the pre-election polls were not as accurate as the exit polls conducted on the day of the election. This is an unusual question (there was one like it in the 2014 test paper, but never before), and it is something of a hybrid of a Flaw and an Inference question. The correct answer must follow from the information provided, and it will point out a reasoning error in the information—remember, an overlooked explanation is a common type of flaw. The two paragraphs under the table detail the projections of the pre-election polling, which foretold a hung parliament, with the Conservatives narrowly ahead of Labour (or in a virtual tie), in projected seats. However, the exit polls suggested a much stronger showing by the Conservatives and a much worsened result for Labour. The final paragraph attributes the failure of the pre-election polling, contrasted with the success of the exit polls, to the fact that most of the pre-election polls were conducted by landline phone, which can oversample certain populations. The exit polls were conducted as voters left the polls, so there is an overlooked explanation given in answer (E): it’s likely that the exit polls were more accurate since they surveyed people who actually voted, and were reporting the results of the votes immediately after voting. For this reason, answer (E) is correct. For the record: There is no evidence in the information provided that voters were ‘fully informed about the issues’ on election day, so (A) cannot be inferred as an alternative explanation. Similarly, there is no information that most voters had not made up their minds at the time that the pre-election polls were taken; this would seem unlikely, though it is wholly unsupported,

so it is not valid as an alternative explanation. (C) contradicts the information provided—the exit polls were conducted in person, not by telephone. (D) limits the alternative explanation to an underrepresentation of people who voted for the SNP. However, the data contradicts this suggestion, since the pre-election polling projected that the SNP would win 46–56 seats, and they won 56 seats—the very top of the projected range. Thus, the SNP projection was accurate; if anything, SNP voters may have been overrepresented in the preelection polling data, since the actual number of seats won by the SNP was the top of the projected range. 10. (C) Percentage change is equal to the value of the change over the original value. The original value is the total number of Lib Dem seats after the election, plus the net change: 8 + 49 = 57. The value of the change is 49, 49 so the percentage change is equal to 57 . To minimise the effect of rounding, estimate the denominator as 49 7 56; 56 reduces to 8 , or 87.5%. Since we rounded down, the actual value will be a little less than 87.5%, as our actual denominator includes smaller segments of the whole. Hence, answer (C) must be correct. For the record: Note that if you had estimated the fraction as 50 —rounding both parts up—you would have found a 60     value of 83.3%. This is more challenging to compare to the actual value, but remember, the denominator will have a greater impact on the actual value, since we are talking about 49 or 50 portions—the size of the portions is quite significant. One-sixtieth is smaller than one-fifty-seventh, so if you have 49 of the lat49 ter, it will be more than 50 of the former. Thus, 57 is 50 greater than 60 , and the correct percentage must be a bit larger than 83.3%. NB how much easier the final step is if you minimise your rounding—look for common factors, and try to round to the closest common factor whenever possible. 11. (A) The target here is the number of EU citizens (excluding UK citizens) who were employed in the UK in 2015 that started their UK employment in that year. The second paragraph explains that 2,000,000 EU citizens (excluding UK citizens) were employed in the UK in 2015, of which 10.75% were employed in the UK for the first time that year. Thus, our target is 10.75% of 2,000,000. It might be quicker to break it into 10% and then 0.75%. Ten per cent of 2,000,000 is 200,000. To multiply 0.0075 (or 0.75%) by 2,000,000, you might

143

Lesson 6 Advanced Problem Solving

first reduce 2,000,000 by a factor of 10 for each place that you move the decimal point on 0.0075 to the right. For example, 0.0075 × 2,000,000 = 75 × 200, which equals 15,000. 200,000 + 15,000 = 215,000. The answer is (A). 12. (C) The target here is a percentage: the number of nonEU nationals employed in the UK for the first time in 2015 divided by the total number of non-EU nationals employed in the UK. There were a total of 1,200,000 non-EU nationals employed in the UK in 2015, of which 38,000 were employed in the UK for the first 38,000

time. Before dividing, see if you can reduce: 1,200,000 38

= 1,200 . You could reduce further, or you could do some long division, or you could break the fraction 38 36 into two parts, then reduce each in turn: 1,200 = 1,200 2 + 1,200 . 36

3

The first fraction, 1,200, reduces to 100, or 0.03, or 3%. 2

1

1

1

The second fraction, 1,200 , reduces to 600 , or 6 × 100. 1 0.167 = 0.167, so 100 = 0.00167 = 0.167%. 6

Adding the two values together: 3% + 0.167% = 3.167%. Answer (C) is correct. 13. (C) Estimate and eliminate to find the answer quickly. 51% of children 10 and under walk to school; that’s approximately 50%, and there are 4.5 million ­children in this group. 50% of 4.5 million is 2.25 million. 41% of children 11 and over walk to school, which is approximately 40%, and there are 4 million children in this group. 0.4 × 4 million is 1.6 million. Add the two groups of children who walk for the approximate total: 2.25 million + 1.6 million = 3.85 million. Eliminate (A) and (B), which are clearly too small. Note that we estimated downwards from the actual percentages, so the actual total will be just slightly higher than 3.85 million. The correct answer is therefore (C). 14. (C) Eyeball here to find the answer quickly. The two overall groups of children are nearly the same size; the 10 and under group is slightly larger. 41% of children in this younger group travel to school by car, and 20% of children in the older group travel by car. If the two groups were exactly the same size, then 30.5% of all children would travel to school by car (as this final percentage would be an evenly weighted average). Since the younger group is slightly larger, and is the larger of

144

the two original percentages, the overall p ­ ercentage that travels by car is just slightly higher than 30.5%. The correct answer is therefore (C). 15. (A)

1

Estimate 27% of 67 as 25% of 68: 4 × 68 = 17. 17 × 60 (rounded down from 62.49) = 1020. 17 × 3 = 51 (to account for the 2.49), so 1020 + 51 = 1071. Note that this is slightly smaller than the actual number of road deaths attributable to alcohol in France, as this figure reflects 25% rather than 27%. The correct answer is therefore (A), which is the nearest answer. 16. (E) The Czech Republic is the only country with a total ban on drink-driving, and the percentage of road deaths attributable to alcohol in the Czech Republic is far below the rate in any other country. Thus, we can safely infer that a total ban on drink-driving significantly reduces the rate of road deaths attributable to alcohol, which matches answer (E). Note that answers (A) and (D) are not logical, as one would not expect a ban on drink-driving to have an effect on road deaths in general (beyond those attributable to alcohol), and answers (B) and (C) mistakenly address the number of road deaths attributable to alcohol, rather than the rate of these deaths. The correct answer is (E). 17. (D) At first glance, the Czech Republic may appear to have experienced the most road deaths in 2008, as it has the highest figure in the Road Deaths per Million Inhabitants row of the table. However, you must multiply that figure by the Population in Millions to determine the total number of road deaths in each country. The figure for the Czech Republic will be just over 1,000 (estimated as 100 × 10 = 1,000), while the figure for Germany will be approximately 80 × 55 = 4,400, which is a lot higher. The figures for France (approximately 60 × 70 = 4,200) and Estonia (100 × 1.34 = 134) are lower than Germany’s, so the correct answer is (D). 18. (B) The Czech Republic had 106 road deaths per million inhabitants, and 3.4% of these were attributable to alcohol. Thus, 96.6% of road deaths were not attributable to alcohol; estimate 96.6% of 106 as 100. 100 × 10.51 = 1051, which is slightly less than the ­correct answer (as 96.6% of 106 would be slightly more than 100). 1023 is too small, so the correct answer must be 1076, answer (B).

S T R AT EG Y SE S SI O N 3

Advanced Triage LEARNING OBJECTIVES By the end of this session, you will be able to:

• Identify common time management mistakes made by BMAT test-takers. • Use the Kaplan Triage Strategy to maximise your marks and time in Sections 1 and 2.

Strategy Session 3 Advanced Triage

TRIAGE CONSIDERATIONS What mistake is made by most test-takers in BMAT Section 1 and Section 2?

How can you avoid making this mistake?

Should you answer the standalone questions in the order in which they appear in Section 1?

Should you answer the Multi-Question Sets in the order in which they appear in Section 1?

How and when should you attempt the Multi-Question Sets in Section 1?

Will you have time for a 2nd pass through Section 1?

How quickly should you aim to answer each of the questions in Section 2?

Will you have time for a 2nd pass through Section 2?

What mistake is made by most BMAT test-takers in Section 3?

How can you avoid making this mistake?

BMAT TEST TIP Don’t discount the value of partial work. Working on a question for 30 to 60 seconds is often sufficient to eliminate 2 or 3 wrong answers. At that point, it may be better to make a best guess from the remaining answers and move on to the next question, than to continue for another minute or longer to verify the correct answer.

147

Strategy Session 3 Advanced Triage

KAPLAN TRIAGE STRATEGY FOR BMAT SECTION 1 AND SECTION 2 Quickly assess each question at first glance—decide whether to Try it or Skip it. ·· If you Skip it—circle entire question, mark an answer randomly, and come back to it if you have time for a 2nd pass through the section ·· If you Try it—decide whether to Stay and Play, or Scoop and Run To Stay and Play: ·· Follow Kaplan timing guideline for Q type ·· Follow Kaplan Method for Q type ·· If time is up and you don’t have an answer—eliminate and guess To Scoop and Run: ·· Have a go for 10-15 seconds ·· Eliminate based on your partial work, then guess ·· Circle question number, so you can review if you have time for a 2nd pass

When should you skip a question?

When should you ‘Scoop and Run’ a question?

When should you ‘Stay and Play’ a question?

Why is it essential to triage aggressively in Section 1 and Section 2?

What is the advantage in deciding to Stay, Scoop or Skip?

BMAT TEST TIP Circle the letter for each correct answer in your test booklet. Fill in your answer sheet once every two pages. ‘Bubble’ the answers when you finish a right-hand page. This will help ensure you don’t make a series of errors by misbubbling your answer sheet.

148

LESSON 7

Advanced Applied Science LEARNING OBJECTIVES By the end of this session, you will be able to:

• Identify the types of Physics and Maths questions asked in Section 2 of the BMAT. • Use the Kaplan Methods to solve Physics and Maths Questions.

Lesson 7 Advanced Applied Science

KAPLAN METHOD FOR APPLIED SCIENCE STEP 1: STEP 2: STEP 3:

When will you use the Kaplan Method for Calculate Questions?

What will you use to revise for the science content in Section 2?

How will you use that resource?

Applied Science Questions vs GCSE Questions ·· Content: ·· Format: ·· Speed:

Warm-Up Exercise ·· The next two pages contain two Applied Science questions. ·· Try to complete the exercise, and answer both questions, in 2 minutes.

151

Lesson 7 Advanced Applied Science

1

For the equation x y = 6  ∙ 3 + 2∙2 – 5

Which of the following is a correct expression of x  ?

152

A ±3



y+5 +2 6

B ±3



y+5 –2 6

C ±3



y+5 +6 6

D ±3



y+5 –6 6

E ±3



y + 15 +6 6

F ±3



y + 15 –6 6

Lesson 7 Advanced Applied Science

A student is conducting an experiment to determine the half-life of radium-229, which decays into actinium-229 through a process of β-emission. The student starts the experiment with a 64 mg sample of radium-229, and takes a reading of the mass of radium-229 every 2 minutes for an hour, by which time the remaining mass of radium-229 in the sample is negligible. The student’s readings are shown in the graph below. 70

Mass of ra-229 (mg)

60 50 40 30 20 10 0 0

4

8

12

16

20

24

28

32

36

40

44

48

52

56

60

Time (min)

2

Based on the information in the graph, what is the half-life of radium-229? A

2 minutes

B

4 minutes

C

6 minutes

D

8 minutes

E

12 minutes

F

16 minutes

STOP. IF YOU FINISH BEFORE TIME IS UP, CHECK ANY QUESTIONS YOU HAVE MARKED FOR REVIEW. YOU MAY GO BACK TO QUESTIONS IN THIS EXERCISE ONLY.

153

Lesson 7 Advanced Applied Science

Velocity and Acceleration Physics concepts tested on the BMAT involve a few basic formulae. Start your list with these: ·· Velocity: v = ∆ d , where ∆d = change in displacement and t = time; usually measured in m/s t ·· Acceleration: a = ∆ v , where ∆v = change in velocity and t = time; usually measured in m/s2 t

A train accelerates as it reaches the high-speed portion of the track (where it can sustain speeds of 200 kilometres per hour or greater), and then slows again once it reaches the end of the high-speed track. The train’s speed and time throughout its entire journey are graphed below. 250

150

100

Speed (km/hr)

200

50

0

3

5

10

15

20

25

30 35 40 Time (min)

45

50

55

60

65

7

0

What is the train’s acceleration as it reaches the high-speed track, in m/s2? 1 A 9 1 B 6 1 C 3 2 D 5

BMAT TEST TIPS Most of the physics formulae you’ll use on the BMAT are three-part formula. Expect to be given two parts of the formula, and solve for the third. You will have to do so once on most physics questions. Some will require you to do so twice.

154

Lesson 7 Advanced Applied Science

Force, Work and Power These concepts are linked, as power is defined in terms of work, and work is defined in terms of force: ·· Force: F = ma, where m = mass (in kg) and a = acceleration (in m/s2); usually measured in newtons (N) ·· Work: W = Fd, where F = force (in N) and d = distance (in m); usually measured in joules (J) ·· Power: P = W , where W = work (in J) and t = time (in sec); usually measured in watts (W) t

Formal gardens include a water feature in which jets of water shoot up vertically, through various tubes each powered by its own pump. The shortest jet of water reaches a height of 3 metres once it leaves its tube. Each second, 4 kg of water passes through each tube in the water feature. Assume all pumps are 100% efficient. 4

What is the height of the tallest jet of water, if its pump is 7 times more powerful than the pump of the shortest jet?

(g = 10 m/s2)

A

7m

B

12 m

C

14 m

D

18 m

E

21 m

F

24 m

BMAT TEST TIP If something is falling, or being thrown/launched/fired into the air, the acceleration is gravitational pull (g), or a force equal and opposite to gravitational pull. The BMAT will always tell you to assume the value for g, which is usually 10 m/s2.

155

Lesson 7 Advanced Applied Science

Kinetic Energy and Potential Energy Kinetic energy involves movement, and potential energy involves height. ·· Kinetic energy: K.E. = 1 mv2, where m = mass (in kg) and v = velocity (in m/s) 2    ·· Potential energy: P.E. = mgh, where m = mass (in kg), g = gravity and h = height (in m) ·· Both kinetic energy and potential energy are measured in joules.

5

An object at rest of mass 12 kg falls from a height of 20 m. What is the object’s speed at the instant it reaches the ground? (Assume air resistance is negligible.)

(g = 10 m/s2)

A

2.4 m/s

B

10 m/s

C

12 m/s

D

20 m/s

E

400 m/s

BMAT TEST TIP Kinetic energy and potential energy are often tested together on the BMAT. Most commonly, you may have to set up an equation where K.E. = P.E.

156

Lesson 7 Advanced Applied Science

Circuits Circuits will usually include a cell and a lamp, along with one or more ammeters and resistors. ·· Current: the movement of electrons through the circuit; measured in amperes (A) ·· Voltage: the force that moves electrons through the circuit; measured in volts (V) ·· Resistance: any force in the circuit that slows the movement of electrons through the circuit; measured in ohms (Ω) ·· The relevant formula is I = V , where I = current, V = voltage and R = resistance. R  ·· In a series circuit, add the resistance of each resistor together to find total resistance. ·· In a parallel circuit, add the reciprocal of the resistance of each resistor together, then take the reciprocal of this sum, to find total resistance.

A circuit is arranged as shown:

A1 T R1

6

What will happen to the reading at ammeter A1 and to the bulb when a second resistor identical to R1 is placed across the terminals marked T?

reading at A1 bulb

A increases

dims

B

decreases

no change

C

no change

brightens

D

no change

dims

E increases

brightens

F decreases

brightens

BMAT TEST TIP Be sure to review your GCSE Physics notes, to ensure that you recognise circuit symbols that are likely to appear in diagrams on the BMAT. You might also practise calculating for current, resistance and voltage. At the same time—don’t stress too much about circuits. Past BMAT test papers usually feature just one circuit question—so be sure to balance for this in your prep time, and in your thinking about Test Day. Circuits will likely be only one mark out of a possible 27.

157

Lesson 7 Advanced Applied Science

Waves The same principles apply to all waves, including sound, light, microwaves and waves in the ocean. ·· Frequency: the number of full waves passing a fixed point each second; measured in hertz (Hz) ·· Wavelength: the length of a full wave (crest to crest); usually measured in m ·· Wave speed: v = fλ, where f = frequency (in Hz) and λ = wavelength (in m)

7

All colours of visible light travel at the same speed (speed of light = 3.0 × 108 m/s), but each colour of visible light has a different frequency and a different wavelength. Violet light has a frequency of 7.5 × 1014 Hz, and its wavelength is 53.3% of the wavelength of red light. What is the wavelength of red light?

(1 nm = 10 –9 m)

A

400 nm

B

450 nm

C

725 nm

D

750 nm

E

800 nm

F

825 nm

BMAT TEST TIP When waves encounter an obstacle, they can be reflected, refracted or diffused. Review your GCSE Physics notes so you are comfortable with these concepts. There aren’t any formulae involved—so just be sure you know the basics of each.

158

Lesson 7 Advanced Applied Science

Radioactive Decay Radioactive decay questions may involve the half-life of an isotope, or the particles released as an isotope decays. ·· Half-life: the time required for half the radioactive atoms in a sample to decay ·· α particle: 2 protons and 2 neutrons; each α particle emitted decreases the atomic number of the isotope by 2 and decreases the mass number by 4 ·· β particle: 1 electron, emitted when a neutron converts into an electron and a proton; each β particle emitted increases the atomic number of the isotope by 1

8

Some radioactive isotopes go through a series of alpha and beta disintegrations as they decay, before reaching a stable isotope. These series, known as decay chains, may release several α particles and β particles in a very particular sequence before a stable isotope is achieved. The decay chain for uranium-238 ( 238 92U) emits a total of 8 α particles and 6 β particles, before achieving a stable isotope. Once the stable isotope is reached, what are its atomic number and mass number?

atomic number

mass number

A

72

200

B

72

224

C

76

206

D

76

230

E

78

200

F

78

224

G

82

206

H

82

230

BMAT TEST TIP Beta decay does not affect mass number, but it increases atomic number.

159

Lesson 7 Advanced Applied Science

Algebra The algebra topics tested most frequently in BMAT Section 2 are solving/rearranging equations and inequalities, the FOIL method, powers and roots. ·· When solving/rearranging an equation or inequality, do the same thing to both sides. ∘∘ Be prepared to isolate a variable or to find a specific value. ∘∘ If you divide both sides of an inequality by a negative number, you must reverse the inequality sign. (eg, If –2b > 6, then b < –3) ·· FOIL method: Expand two brackets into a quadratic expression by multiplying the first two terms, then the two outside terms, then the two inside terms, then the last two terms, and add like terms together: ∘∘ (x + 3)(x + 4) = x2 + 4x + 3x + 12 = x2 + 7x + 12 ·· To solve a quadratic for a value of x, set it equal to 0 and then reverse-FOIL into brackets: ∘∘ If x2 – 7x + 6 = 0, then (x – 1)(x – 6) = 0, and x = 1 or 6. ∘∘ NB Once a quadratic is set equal to 0, the two numbers that go into the brackets must multiply together to make the final term (+6) and add together to make the coefficient of x(–7). ·· There are a few basic rules about powers and roots that you must know for the BMAT: ∘∘ (x  a  )(x b  ) = x  a+b a ∘∘ x b = x  a–b x  

∘∘ (x  a  )b = x  ab ∘∘ x –a = 1a x   ∘∘ √ cd            = (√ c          )(√ d         ) ∘∘

∙ dc

= √ c          √ d         1

∘∘ √ c       = c  2 ∘∘ (√ c          )2 = c ∘∘ If a negative number is raised to an even power, the result is always positive. If a negative number is raised to an odd power, the result is always negative.

9

If 3(x 2 + 2) = 11x and x 2 =

x + 15 , what is the value of x  ? 2

A –2 5 B –   2 2 C 3 D 3

BMAT TEST TIPS There are a few very common, very basic FOIL factors you should memorise for Test Day: (a + b)2 = a2 + 2ab + b2     (a – b)2 = a2 – 2ab + b2     (a + b)(a – b) = a2 – b2 

160

Lesson 7 Advanced Applied Science

10

Evaluate (√ 13   + √ 11    )2    (√ 13   – √ 11    )2. A 4 B 16 C 48

11

D

576 – 32√ 143  

E

4 + 32√ 143  

F

576 + 32√ 143  

If

2 × 10a = 4 × 108, then a – b = 50 × 10b

A 4 B 6 C 8 D 10 E 12

BMAT TEST TIP If the answers on an algebra question are numbers, work backwards from the answers. Before plugging the answers into the equation, check whether it’s actually possible for the answer to be negative, or a fraction (if there are negative or fractional answers).

161

Lesson 7 Advanced Applied Science

Geometry The geometry topics tested most often in BMAT Section 2 are area, volume and graphing lines, quadratics and inequalities. ·· Review and memorise the formulae for all of the following shapes: ∘∘ Area of a circle, triangle, square, rectangle, parallelogram and trapezium ∘∘ Volume of a cuboid, prism, sphere, pyramid and cone ·· Review and practise how to work out the following types of calculations: ∘∘ ∘∘ ∘∘ ∘∘

Perimeter of a polygon and circumference of a circle Sum of interior angles of a polygon An exterior angle of a polygon Alternate angles, supplementary angles and corresponding angles, when a line crosses two parallel lines

·· Review how to graph a straight line, using both the ‘x = 0, y = 0’ approach and y = mx + c (where m is the gradient of the line, and c is the y-intercept). ·· Review how to find the gradient of a straight line, if given coordinates for two points on the line. ·· A circle’s diameter, area and circumference are all defined in terms of radius. If you know one of these four, you can work out all the others. ·· Three special right triangles will help you avoid using Pythagoras’s theorem: ∘∘ 45 : 45 : 90 triangle: A triangle with angles of 45° : 45° : 90° has sides of x : x : x√ 2       ∘∘ 30 : 60 : 90 triangle: A triangle with angles of 30° : 60° : 90° has sides of x : x√ 3       : 2x ∘∘ 3 : 4 : 5 triangle: Also known as a Pythagorean triple. If a triangle has two sides in the ratio of 3 : 4 : 5, then you can work out the third side based on the ratio. This works for other triangles (eg, 6 : 8 : 10, 15 : 20 : 25), so long as all three sides are multiplied by the same factor.

12

What is the greatest possible distance between two points on a cube with sides of unit length? A

√ 3        2   

B

√ 2  

C

√ 3   3 √ 2       

D

E 3

BMAT TEST TIPS Complex figures are easier to work with if you break them down into simpler shapes. Look for ways you can use the properties of circles and special triangles. Any quadrilateral can be broken into two triangles. Sometimes, this is the only way to work with an unusual quadrilateral.

162

Lesson 7 Advanced Applied Science

13

A student graphs the following equations: 1. y = 6 – x2 2. y = 11 – x 3. y=x+4 4. y = 3x 2 + 16 Which two of the graphs intersect exactly twice? A

1 and 3

B

1 and 4

C

2 and 3

D

2 and 4

E

3 and 4

17 cm p

q

23 cm

14

The trapezium above has an area of 160 cm 2. What are the lengths of p and q, in cm?

A

p

q

4

 5

B

4

 6

C

6

10

D

6

12

E

8

10

F

8

12

BMAT TEST TIP The BMAT is unlikely to ask you to come up with a graph for a particular parabola (quadratic). You should, however, know which way a parabola will ‘open’ (up if the x2 is positive, down if it’s negative), 1 and whether it will be ‘wide’ or ‘narrow’ (eg, 3x2 is ‘narrower’ than x2, and 3  x2 is ‘wider’ than x2   ).The BMAT may expect you to work out the graph of a particular line (based on its slope, x-intercept and y-intercept) or inequality (ie, you must know which side of the line would be shaded).

163

Lesson 7 Advanced Applied Science

Probability Probability questions appear in both Section 1 and Section 2. The only real difference is that you have half as much time to answer the probability questions in Section 2. ·· Probability of all possible events adds up to 1. ·· To find the probability of two independent events both happening, find the probability of each event happening, and multiply these together. ·· To find the probability of either of two mutually exclusive events happening, find the probability of each event happening, and add these together. ·· If one event is dependent on another, you must consider this in working out its probability. The red and blue marble question from Lesson 2 is a good example of dependent probability.

15

A game involves a spinner with spaces of equal size, numbered from 1 to 10. Players may bet on 1, 2 or 3 numbers, and then they win 10 times the amount bet if they spin one of the numbers they picked.

You should have equal probability (1 in 10) of spinning any particular number, but two spaces on the spinner have been weighted. The probability of spinning one of the weighted spaces is 1 in 6. If you know which two spaces are weighted and bet on those numbers and one other, what is the probability that you will spin one of your three numbers? A 33% B 36% C 42% D 45% E 48%

BMAT TEST TIP Probability can be written in a variety of formats: 1 33.3%    1 : 3     3       odds of 1 in 3 You can make easy, fast work of probability questions by ALWAYS using fractions.

164

Lesson 7 Advanced Applied Science

LESSON 7 QUIZ (4 minutes)

Please read this page carefully, but do not start the quiz until you are told to do so. Speed as well as accuracy is important in this section. Work quickly, or you may not finish the quiz. There are no penalties for incorrect responses, only points for correct answers, so you should attempt all questions. All questions are worth one mark. You will have 4 minutes to answer the 4 questions in this quiz. Record your answers in this book. Show your choice clearly by circling the corresponding letter in your book. If you make a mistake, erase thoroughly and try again. Any rough work should be done in this book. At the invigilator’s prompt, turn the page and begin the practice quiz.

165

Lesson 7 Advanced Applied Science

16

At a music festival, a band is performing on an elevated stage in front of an enormous crowd in a huge, flat field. On stage, the percussionist strikes a triangle 20 times a minute, and she tells the crowd to clap their hands once every other time she strikes the triangle. Once the crowd establishes its rhythm, the fans at the very back of the field clap their hands on the beats that the fans sitting at the front edge of the stage (immediately in front of the percussionist) are skipping, and vice versa. What is the minimum distance from the front of the stage to the fans at the very back of the field?

17



(speed of sound in air = 330 m/s)

A

220 m

B

495 m

C

660 m

D

990 m

E

1485 m

F

1980 m

A decorative square tile features an unshaded circle inscribed exactly within it. A shaded square is inscribed exactly within the circle in the following pattern:

What fraction of the tile is shaded? A

π 1 –   4

B

π–1 4 6–π 4 3 4 π 4

C D E

166

Lesson 7 Advanced Applied Science

18

19

What is the mass of a railway engine that produces a maximum tractive effort of 3000 N and accelerates from rest to a velocity of 36 m/s in one minute? A

600 kg

B

1800 kg

C

3000 kg

D

5000 kg

E

6000 kg

If x > 0, solve x   3 – 10x + 12 2 x = –   x – 1 4 2 x

∙ ∙

A

x=1

B

x=2

C

x=4

D

x = 10

E

x = 12

STOP. IF YOU FINISH BEFORE TIME IS UP, CHECK ANY QUESTIONS YOU HAVE MARKED FOR REVIEW. YOU MAY GO BACK TO QUESTIONS IN THIS QUIZ ONLY.

167

Lesson 7 Advanced Applied Science

A FEW WORDS FROM THE EXPERTS Kaplan Method for Applied Science STEP 1: ID the question type. STEP 2: Operate. STEP 3: Eliminate and select.

Kaplan Method for Calculate Questions STEP 1: ID the target. STEP 2: Research relevant data. STEP 3: Set up and solve. STEP 4: Eliminate and select.

NEXT STEPS ·· Be sure to allow time to revise your GCSE content physics and maths content, focusing on the topic areas listed in this lesson. Once it is available on the BMAT website (late summer), you can use the Assumed Subject Knowledge Guide to revise. But remember to use it strategically and revise judiciously. Some topics are far more likely to appear on the BMAT than others. ·· You should also look over the past papers available on the BMAT website, to see how these topics have been tested in actual BMAT questions. ·· When you check the past papers, you should also make a list of any biology or chemistry topics that are unfamiliar or that you otherwise need to revise. There will be many biology questions involving the human body and genetics, and many chemistry questions involving the periodic table and stoichiometry—so be sure to spend some time reviewing the relevant portions of your GCSE notes.

BMAT TEST TIPS Fractions are essential to making quick work of calculations on the BMAT. You should ALWAYS reduce fractions before multiplying or dividing.

168

Lesson 7 Advanced Applied Science

ANSWERS AND EXPLANATIONS Answer Key

3. (A) The train reaches the high-speed track after accelerat-

1. (D)

11. (D)

2. (B)

12. (C)

3. (A)

13. (A)

4. (E)

14. (E)

5. (D)

15. (C)

6. (E)

16. (F)

7. (D)

17. (C)

8. (G)

18. (D)

to m/s, so multiply by 1000 m/km and also by 3600 120 hr/s. The acceleration is therefore a = ∆v = 300 ×

9. (D)

19. (B)

1000 × 3600 = 36 = 9 = m/s2. Answer (A) is correct.

10. (A)

1. (D) Rearrange the equation to make x the subject: x

2

x

2

y = 6 ∙ 3 + 2∙ – 5



y + 5 = 6 ∙ 3 + 2∙



y+5 2 x = 3 +2 6





The fraction containing y correctly appears in only some of the answer choices; eliminate (E) and (F).



y+5

±     6

x

   = 3 + 2

Next, subtract 2 from both sides; eliminate (A) and (C), as they add something to the fraction on the left.



y+5



y+5

±     6 ±     6

20-minute mark, going from a speed of 100 km/hr to a speed of 220 km/hr. Thus, the change in velocity is 220 – 100 = 120 km/hr, and the time is 5 min × 60 s/ min = 300 s. However, the speed must be converted 1

1

4

1

t

4. (E)

Explanations



ing at a constant rate from the 15-minute mark to the

x

    + 2 = 3

Water shoots out of each jet with an acceleration of 10 m/s2, to counterbalance gravitational pull. Thus, the force of the pump that shoots out the shortest jet is F = m × a = 4 kg × 10 m/s2 = 40 N. The work done by this pump is W = F × d = 40 N × 3 m = 120 J. Since the time involved is 1 second, the power of this pump is 120 watts (as power equals work divided by time). The pump that shoots out the tallest jet is 7 times more powerful, so its power is 120 × 7 = 840 watts. Again, the time is 1 second, so the work done by this pump is 840 J. The distance (the height of the jet of water it shoots out) is the unknown, but its force is the same as the pump that shoots the shortest jet, as 4 kg of water passes through each pump each second; thus, the second pump also has a force of 40 N. Divide work by force to solve for distance: 840 J ÷ 40 N = 21 m. The answer is therefore (E). 5. (D) P.E. = mgh = 12 × 10 × 20 = (12 × 200)J

    – 6 = x

1

1

The correct answer is (D).

K.E. = 2 mv2 = 2 × 12 × v2    P.E. = K.E.

2. (B)

1 × 12 × v2 = (12 × 200)J 2

The half-life of an isotope is the time that it takes for half of the atoms of that isotope present in a sample to decay. Thus, you can pick any two different masses of ra-229—so long as one is half of the other—and the difference in the corresponding times on the x-axis will equal the half-life. For example, the sample starts with a mass of about 65 mg at 0 minutes, and reaches half this value (37.5 g) after 4 minutes. Thus, the half-life of ra-229 is 4 minutes. Answer (B) is correct.

v2 = 400 v = 20 ms–1 This question involves an object that falls from a height, and asks for the object’s speed at the instant it hits the ground. The change in the object’s potential energy will be equal to the object’s kinetic energy at that moment. This makes (D) the correct answer choice.

169

Lesson 7 Advanced Applied Science

6. (E)

9. (D)

This circuit in this diagram is to have a second identical resistor added in parallel. Adding an identical resistor in parallel will decrease the resistance in the circuit, and because V = I × R the current will increase. As such the ammeter reading will increase and the bulb will become brighter. This is represented by answer choice (E).

You could solve this question using reverse-FOIL; however, since there’s only one variable in the two equations, it would likely be faster to simply plug the answer choices in to see which is correct. Start by plugging (A), x = –2, into the first equation, and you will find the left side equals 18 and the right side equals –22. Eliminate (A); you can also eliminate (B), as the x2 on the left side will turn any negative value of x into a positive, while the right side will be negative. The next easiest answer to check is (D):

7. (D) The first step is to find the wavelength of violet light. Wavelength equals velocity divided by frequency, so plug in the values for violet light to solve for its 2 wavelength: λ v = 3.0 × 108 ÷ 7.5 × 1014 = 5 × 10 –6 = 0.4 × 10 –6 = 4 × 10 –7 = 400 nm. This value is 53.3% of the wavelength of red light, or just more than half of the correct answer; eliminate (A) and (B), which are too small to be correct; (E) would be correct if violet light was exactly 50% the wavelength of red light, so eliminate (E) and also (F), which would result in a percentage lower than 50%. To calculate the wavelength of red light without a calculator, estimate 53.3% as 52%, to minimise maths:

52

400 = 100 × λr 13

400 = 25 × λr 10,000 = 13 × λr





10,000 = λr 13

Dividing the first few figures gives an estimated λr just above 760 nm. Since 52% is slightly smaller than the actual percentage, the wavelength of red light will be just a bit less than 760 nm. Answer (D) is correct. 8. (G) The original isotope has a mass number of 238 and an atomic number of 92. An α particle consists of 2 protons and 2 neutrons, so each α particle emitted reduces the mass number by 4. Since 8 α particles are emitted, the mass number decreases by 8 × 4 = 32. Thus, the mass number of the resulting isotope is 238 – 32 = 206. Eliminate all answers except (C) and (G). Each α particle emitted will also reduce the atomic number by 2, so the total α decay in the decay chain decreases the atomic number from 92 to 76. However, the emission of the 6 β particles results in a net gain of 6 protons, so the stable isotope has an atomic number of 76 + 6 = 82. The answer is (G).

170

3(x2 + 2) = 11x 3(32 + 2) = 11(3)

3(9 + 2) = 33



3(11) = 33

The two sides are equal, so answer (D) is correct. If you wanted to double-check, you could also plug in 3 for x in the second equation, and both sides would equal 9. But you would be unlikely to have time to do so on Test Day. 10. (A) This expression is easier to work with if you rewrite it: [(√ 13    + √ 11        )(√ 13    – √ 11       )]2 Use FOIL to simplify within the square brackets: [13 + √ 143    – √ 143    – 11]2 Simplify to solve: [13 – 11]2 = [2]2 = 4. Answer (A) is correct. 11. (D) This question gives a formula containing a and b as powers of 10 and asks for a – b. Our goal is to rearrange the equation to give an expression for a – b, and this should activate your knowledge about how to work with powers in fractions, e.g. that

10a = 10a               –          b. 10b

We should use this approach to rearrange the equation in question:



2 × 10a = 4 × 108 50 × 10b



2 × 10a = 200 × 108 10b



10a = 100 × 108 10b



10a    –   b = 102 × 108

10a    –   b = 1010 So a – b = 10, and answer (D) is correct.

Lesson 7 Advanced Applied Science

12. (C) The greatest possible distance on a cube is between two opposite vertices. To calculate this distance, first work out the length of a diagonal on a face. The diagonal is the longest side on a 45˚-45˚-90˚ triangle, so it is √ 2  units long. The distance between two opposite vertices is the longest side on a right triangle with shorter sides of 1 and √ 2  units. Use Pythagoras’s theorem to find the length of the longest side: x2 = (√ 2      )2 + 12 x2 = 3 x = √ 3      

15. (C)

1

The probability of spinning a weighted space is 6 , and there are two such spaces. Thus, the probability of

spinning a space that is not weighted is 1 – 1 – 1 = 6 6 4 2 = 3 . Since there are eight unweighted spaces on 6   

Answer (C) is correct.

the spinner, the probability of spinning any one of the 2 2 1 2 unweighted spaces is ÷ 8 = × = = 1 . The 3

13. (A) Equations 2 and 3 are both straight lines, with gradients of –1 and 1, respectively. Since the gradient of one of these lines is the negative reciprocal of the gradient of the other, the lines are perpendicular, and intersect exactly once. Eliminate (C). The quickest way to compare these lines with the parabolas that will result from equations 1 and 4 is to consider the y-intercept of each equation, ie the value of y when x = 0. The y-intercepts of the 4 equations are 6, 11, 4 and 16, respectively. Moving up the y-axis from (0, 0), then, you’d encounter the graphs of the equations in this order: equation 3, equation 1, equation 2, equation 4. Furthermore, the parabola resulting from equation 1 will open downwards (as –x2 has a negative coefficient) and the parabola resulting from equation 4 will open upwards (as 3x2 has a positive coefficient). Thus, equation 4 will not intersect any of the graphs of the other equations, as it is well above the other three. Eliminate (B), (D) and (E). Answer (A) is correct. For the record: the graphs of equations 1 and 3 intersect at (1, 5) and (–2, 2). 14. (E)

Eliminate (A), (B), (C) and (D). To find the value of q, divide the figure into a rectangle (of 17 cm × 8 cm) and a triangle. The triangle has a base of 23 – 17 = 6 cm, and a height of 8 cm. You could use Pythagoras’s theorem to solve for q, or you may have noticed that this is a 3 : 4 : 5 triangle, as 6 = 3 × 2 and 4 = 4 × 2. Thus, q = 5 × 2 = 10. The correct answer is (E).

1

The formula for the area of a trapezium is 2 (a + b)    h, where h is the height and a and b are the bases (the parallel sides). The question gives the area, 160 cm2, and the diagram gives the lengths of the bases as 17 cm and 23 cm. The height of the trapezium is the same as p, one of the two unknown sides. Plug these values into the area formula, and solve for p: 1 (17 + 23)p = 160 2    40p = 320 p = 8

3

8

24

12

question asks for the probability of spinning one of three possible numbers: either of the weighted numbers or one of the unweighted ones. Since the spinner is to be spun only once, these are mutually exclusive events; you must therefore add the probability of each 1 of the three events together: 6 + 1 + 1 = 5 . The 6 12 12 answer choices are given as percentages rather than 4

1

6

1

fractions, but consider that 12 = 3 and 12 = 2 . Hence,

the correct answer must not only be between 33.3% and 50%, but it must be equidistant between them, as each twelfth is of the same size as all the others. The midpoint between 33.3% and 50% is very close to 42%, so the correct answer must be (C). 16. (F)

To find the wavelength of the sound produced when the percussionist strikes her triangle, use the formula v = fλ. The velocity is 330 m/s. The percussionist strikes the triangle 20 times a minute, so the frequency is 20 ÷ 60 sec/min = 1 Hz. Divide velocity 3 by frequency to find wavelength: v ÷ f = λ = 330 m/s ÷ 1 Hz = 330 × 3 = 990 m. Since the people at the 3 very back are off-rhythm by a full cycle, double the distance: 990 m × 2 = 1980 m. The answer is (F). 17. (C) This question asks for the shaded area in the diagram. 3 By eyeball, the area shaded is around 4 , and π ≈ 3.14, so eliminate (A) and (B). Similarly, the unshaded area 3 is bounded by a circle, so the area 4 itself can not be correct—eliminate (D). If the circle is of radius r, its area is πr2.

171

Lesson 7 Advanced Applied Science

The circle fits inside the tile exactly, so the side of the tile is equal to 2r and the area of the tile is (2r)2 = 4r2. The small shaded square fits exactly inside the circle, so its diagonal is 2r. The square is formed of two rightangled triangles of base 2r and height r, so the area 1 of the shaded square is 2 × ( 2 × base × height) = 2r2. Putting it all together, the shaded area is:



shaded tile (6 – π)r2 = 4r2 (6 – π) = 4

4 2

12 2

22

= – ∙ 2 ∙ – 1

8 – 20 + 6 4 = –  2 – 1 2

∙ ∙



–6 = –2 – 1 2

This equation simplifies to –3 on both sides, so the answer is (B).

Answer (C) is correct. 18. (D) This question uses a number of unfamiliar terms, but you may have noticed that it gives a force in newtons and describes the engine accelerating, while the correct answer will be the mass of the engine. This information should remind you of the formula, F = ∆v

ma. To find the acceleration, use the formula a = t , 36 – 0 with 60 seconds as the time: a = 60 = 3 m/s2. 5 To solve for the mass of the engine, divide force by 3

5

acceleration: m = 3000 ÷ 5 = 3000 × 3 = 5000 kg. The correct answer is (D).

172

∙ ∙

23 – 10 × 2 +

You could choose the correct answer as a partial solve at this stage, or complete the workings step-by-step by stating the shaded area as a fraction of the tile:



12 x3 – 10x + x x2 = –  2 – 1 4 x

Answer choice (B), x = 2, is the most convenient substitution.

4r2 – π r2 + 2r2 = 6r2 – π r2 = (6 – π)r2



This question requires rearranging a complex equation, and the answer choices are numerical values for x. The best approach is to backsolve, and for the equation:



(tile) – (circle) + (small square)



19. (B)

15,000 = 3

LESSON 8

Advanced Writing LEARNING OBJECTIVES By the end of this session, you will be able to:

• Identify common writing mistakes made by BMAT test-takers. • Build an Idea Bank for use on Section 3 of the BMAT. • Analyse sample essays to determine the scores they would receive.

Lesson 8 Advanced Writing

WRITING REFRESHER Kaplan Method for Writing STEP 1: STEP 2: STEP 3: STEP 4: STEP 5:

Common Mistakes on the BMAT Essay ·· Planning:

·· Examples:

·· Language:

Warm-Up Exercise ·· The next two pages contain a BMAT essay title, sample essay and the essay scoring rubric. ·· In the next 7 minutes, read the essay, make notes on its strengths and weaknesses (in terms of quality of content and written English), and assign a number and letter score.

175

Lesson 8 Advanced Writing

Essay Title 2

Wherever possible, you should avoid providing medical care to anyone with whom you have a close personal relationship.  (UK General Medical Council, Good Medical Practice 2009) Why should a doctor avoid providing medical care to those close to him? Under what circumstances should a doctor treat a close friend or relative? How can a doctor most effectively contribute to the care of those close to him?

Sample Essay

Doctors should avoid providing medical care to those close to him, because it violates the oath that a doctor takes to protect his patients and to do no harm. A doctor could harm someone like a friend if he was a surgeon and operated on the friend. The surgeon could start worrying about the friend or even crying if the surgery did not go well. Under no circumstances should a doctor treat a close friend or relative. However, sometimes a doctor might have no choice but to help a close friend or relative. Like in an emergency, if a doctor is at a restaurant with his sister and starts choking on some food. Or if the sister is climbing a mountain and falls and injures her arm. A doctor can be most effective in supporting the care of those close to him by making sure another doctor is their GP. He can also keep close by involving them in patient care. For example, in my nan’s village, the GP’s wife is also his nurse, so they contribute to the care of all patients together.

176

Lesson 8 Advanced Writing

SCORING In determining the mark for quality of content, the BMAT examiners from Cambridge Assessment consider how well the essay: ·· Addresses the question in the way demanded; ie, has the essay: ∘∘ rephrased the proposition or explained its implications? ∘∘ set out reasonable or plausible counter-propositions? ∘∘ proposed reasonable ways of assessing the competing merits of the propositions or resolved their conflict logically? ·· Organises thoughts clearly ·· Uses general knowledge and opinions appropriately In determining the mark for quality of written English, the BMAT examiners from Cambridge Assessment consider how well the essay: ·· Expresses ideas using concise, compelling and correct English To guide examiners in determining these marks appropriately, Cambridge Assessment provides a series of normative statements that indicate the essential qualities found in an essay of each score. Here are the normative statements for quality of content: Score 0: An essay that is judged to be irrelevant, trivial, unintelligible or missing. Score 1: An essay that has some bearing on the questions but which does not address the question in the way demanded, is incoherent or unfocussed. Score 2: An essay that addresses most of the components of the question and is arranged in a reasonably logical way. There may be significant elements of confusion in the argument. The candidate may misconstrue certain important aspects of the main proposition or its implications or may provide an unconvincing or weak counter-proposition. Score 3: A reasonably well-argued answer that addresses all aspects of the question, making reasonable use of the material provided and generating a reasonable counter-proposition or argument. The argument is relatively rational. There may be some weakness in the force of the argument or the coherence of the ideas, or some aspect of the argument may have been overlooked. Score 4: A good answer with few weaknesses. All aspects of the question are addressed, making good use of the material and generating a good counter-proposition or argument. The argument is rational. Ideas are expressed and arranged in a coherent way, with a balanced consideration of the proposition and counter-proposition. Score 5: An excellent answer with no significant weaknesses. All aspects of the question are addressed, making excellent use of the material and generating an excellent counter-proposition or argument. The argument is cogent. Ideas are expressed in a clear and logical way, considering a breadth of relevant points and leading to a compelling synthesis or conclusion. Here are the normative statements for quality of written English: Band A – good use of English fluent good sentence structure good use of vocabulary sound use of grammar good spelling and punctuation few slips or errors

Band C – reasonably clear use of English reasonably fluent/not difficult to read simple/unambiguous sentence structure fair range and appropriate use of vocabulary acceptable grammar reasonable spelling and punctuation some slips/errors

Band E – rather weak use of English hesitant fluency/not easy to follow at times some flawed sentence structure/paragraphing limited range of vocabulary faulty grammar regular spelling/punctuation errors regular and frequent slips or errors

177

Lesson 8 Advanced Writing

IDEA BANK You can prepare for your BMAT essay by brainstorming some possible examples to use. You can then mentally store up an ‘idea bank’ of examples, so that you’re set for any essay title you may encounter on Test Day. Use the categories and suggestions in the table to brainstorm possible examples, and start building your Idea Bank.

WORK/VOLUNTARY EXPERIENCE

FAMILY/FRIENDS/COMMUNITY

A patient you encountered who struggled to deal with a diagnosis A time that a doctor you worked with helped you see something about the practice of medicine or dentistry that you hadn’t realised before A friend or colleague whose behaviour on a work/ voluntary experience scheme surprised or disappointed you

How a relative felt before and after an important medical test or hospital visit A relative or close friend or neighbour who had a negative experience with a doctor, and how they dealt with it A doctor or dentist that your family knows socially, and interacts with in the community (ie, in a context other than just in a clinical setting)

CURRENT EVENTS

HISTORY

Your views on a current political issue A legal decision or court case involving an issue impacting medicine or science A local or national election campaign involving an important issue

A historical decision you believe was incorrect A defining moment in history that you personally have lived through A historical figure who had a major impact (positive or negative) on science or medicine

PERSONAL EXPERIENCE

LITERATURE/POPULAR CULTURE

A time when you did something you now regret A disappointment you turned into an advantage A time when you were a patient and encountered a difficult or challenging situation

A book that changed the way you see something A fictional character with whom you identify An incident in a book, film or TV show that parallels a real-life situation

BMAT TEST TIP You must use a specific example in each paragraph of your BMAT essay, to illustrate the point you are making. You must also include at least one sentence after each example, to relate it back to the point you are making. Don’t just ‘info dump’ an example without explanation.

178

Lesson 8 Advanced Writing

HOW TO PRACTISE Practising your technique for the BMAT Writing Task will mean you can approach any selection of prompts on test day and produce a high-quality, well thought-out piece of writing in 30 minutes.

Practice Essentials ·· Select other prompts and work through the Kaplan Method under timed conditions. Set a timer! ·· Write your practice essays on official BMAT Section 3 answer sheets from the test maker’s website. ·· When you’re done, take a fresh look at your planning and writing. Is there another position you could take to improve your essay? ·· Review your own work and have other people read and critique your essays.

Reviewing Your Work A structured review will help you identify exactly how to improve on your next practice attempt ·· Did you take a clear position? ∘∘ What is it?

·· Is the essay well organised? ∘∘ Did each paragraph make a clear point? ∘∘ Did the essay have a clear direction overall?

·· How appropriate are the examples?

·· How clear is the expression?

·· Identify two strengths you want to replicate: 1. 2.

·· Identify two weaknesses you want to improve on: 1. 2.

·· Give yourself a number and letter score (from the Scoring rubric on p. 177):

179

Lesson 8 Advanced Writing

REVIEW EXERCISE Practice Essay He who conceals his disease cannot expect to be cured. (Ethiopian proverb) Why would a patient conceal his or her disease? How should a doctor deal with a secretive patient? How can a doctor balance the need to treat the patient with the patient’s preference for privacy?

“He who conceals his disease cannot expect to be cured” means that if you don’t tell anyone about your problem then it will be impossible to expect to have any reasonable treatment. However, the quote ignores why people might not tell the doctor about their medical disease. For example, a patient might not know he is actually seriously ill, for example thinking a cough is just a winter cold when it might actually be lung cancer. This means that concealing a disease is not necessarily secretive, but might just be down to the patient not being as much as an expert as the doctor. When dealing with a secretive patient, a doctor should run tests to look out for diseases the patient might have. In the case of the patient with a cough, because it might be lung cancer the doctor needs to ask how long it has been a problem and then take an X ray. The other thing is being thorough. As a patient might not realise they are seriously ill, the doctor needs to ask about any serious symptoms the patient might have had to be able to make that diagnosis. As a result it is down to the doctor to be able to treat the diseases the patient has. Privacy is important but the main duty of the doctor is to care for the patient. As a result, even though a medical consultation is private the doctor needs to be able to get to the bottom of a problem on the basis of a hunch or a reasonable pattern, even if the patient is being secretive about it. After all, doctors should work to make sure they achieve the best possible outcome for the patient.

180

Lesson 8 Advanced Writing

Review Notes ·· Did the student take a clear position? ∘∘ What is it?

·· Is the essay well organised? ∘∘ Did each paragraph make a clear point? ∘∘ Did the essay have a clear direction overall?

·· How appropriate are the examples?

·· How clear is the expression?

·· Identify two strengths to replicate: 1. 2.

·· Identify two weaknesses to improve on: 1. 2.

·· Decide on a number and letter score (from the Scoring rubric on p. 177):

181

Lesson 8 Advanced Writing

Peer Review ·· Did the student take a clear position? ∘∘ What is it?

·· Is the essay well organised? ∘∘ Did each paragraph make a clear point? ∘∘ Did the essay have a clear direction overall?

·· How appropriate are the examples?

·· How clear is the expression?

·· Identify two strengths to replicate: 1. 2.

·· Identify two weaknesses to improve on: 1. 2.

·· Decide on a number and letter score (from the Scoring rubric on p. 177):

182

Lesson 8 Advanced Writing

A FEW WORDS FROM THE EXPERTS Kaplan Method for Writing STEP 1: ID the task. STEP 2: Brainstorm. STEP 3: Outline. STEP 4: Write. STEP 5: Review.

NEXT STEPS ·· Ignore everyone else in the testing room, as they’re all likely to start writing within the first minute of Section 3. Take 2 minutes to ID the task for which you can write the strongest essay. These are the most important 2 minutes of the Writing Task. ·· Follow the Kaplan Method for Writing, including timing guidelines, on all your practice essays. This will help build confidence and also allow you to develop a ‘feel’ for the timing. Initially, it may seem like quite a while to spend 8 minutes on the first ten minutes on the first 3 steps of the Kaplan Method, before you start writing—but this will feel much more comfortable by Test Day, with a bit of practice. ·· Find a friend who is also sitting the BMAT, and peer review one or more of each other’s essays. Making a ‘study buddy’ in the run-up to Test Day can be useful for both of you as you work to manage stress and keep perspective. ·· Send your practice essays to Kaplan, via your Online Study Plan, as you complete each practice test. We ask that you wait till you get feedback before submitting more essays. This way, you can ‘digest’ our feedback and incorporate it as you write further essays. We are only able to mark essays received by the deadline. Essays sent earlier in the BMAT prep season will normally receive feedback within a week. You may send in up to 6 essays for feedback during the BMAT season (one essay from each practice test), but we must limit this to 6 essays per student, to ensure the quality of feedback for all students.

BMAT TEST TIP You can write an excellent BMAT essay in 17 minutes, if you take 10 minutes to prepare before you write.

183

S T R AT EG Y SE S SI O N 4

Preparing for Test Day LEARNING OBJECTIVES By the end of this session, you will be able to:

• Plan your study schedule between the end of the course and Test Day. • Identify and use common stress management techniques.

Strategy Session 4 Preparing for Test Day

KNOWING TEST DAY IS KEY Knowing what to expect is the key to Crisis Prevention ·· Know yourself. ·· Know your habits. ·· Know the test format. ·· Know the test maker. ·· Know Kaplan Methods. ·· Know you have prepared. ·· Understand Test Day.

Preparing for Test Day ·· Remember, it is normal to see things on the BMAT that you have never seen before. ·· That is the whole point of the exercise, to prove you are capable of thinking through new problems and to demonstrate competence.

Physical, Mental and Emotional State ·· Make sure you are healthy and ready to go. Put aside your fears and stress with the stress management exercise following this page.

The Rules Revisited: 1. No self-defeating language. 2. Positive attitude. 3. Focus on being a GREAT doctor.

Focus on Success ·· Use the visualisation, imaging and positive self-talk and understand what it feels like to succeed.

Keep Moving ·· Use the Kaplan Triage Strategy and don’t be afraid to take breaks during a section. Like sharks, you must keep moving or you will drown.

187

Strategy Session 4 Preparing for Test Day

KNOW THE DIFFERENCE BETWEEN FOCUS AND FATIGUE It is incumbent on you to perform and succeed regardless of the situation. Throughout your training, learn to focus through issues and insure your success on Test Day. Even if there is an abnormality on test day, your score is still dependent on your performance. Maximise your score by maintaining focus and understanding that aberrations will happen on Test Day. Prepare for the worst. Rather than hope you are one of those students who can make it through the test, or hope that your endurance will increase; our experience in helping students with standardised tests has established:

It is okay to take breaks in the middle of the test. Let’s take a look at a common problem. Many students take a section and do worse as the section progresses. By the last problem, they are not doing well at all. It is more likely than not a topic that is reasonable, and if it had been earlier in the test, the student would not have missed so many marks. Students might say, “I keep making stupid mistakes on the test. When I look at the explanations, it makes perfect sense, but on the test I miss them anyway.” There are a lot of reasons this can happen, including distractors in the answer choices, and not knowing the fundamental knowledge well enough yet. However, it could also be a focus issue. Note, we did not say a ‘fatigue’ issue. Students may also say they get burned out in a section or get tired. A student who consistently misses questions in the last passages may believe that they are getting tired. It is possible, but more likely than not they are losing focus.

Take a breath To regain your focus, physically put down your pencil between passages and take a deep breath before you continue. You waste two seconds at most, but the act of taking a break can reset you. Because some passages take longer than others, this method of pacing allows you to maintain focus and provides you with a tactic to enable you to succeed on each passage on the test.

Stretching In-seat stretching is very effective. Stretch out your limbs in front of you. Focus on spreading your fingers and toes as you stretch. Breathe out as you stretch. Hold for a few seconds, then relax. Then lean your head forward curving your back. Roll your head upwards till you are looking up at the ceiling. Hold then repeat. Don’t hit anyone along the way.

STRESS MANAGEMENT Crisis prevention is about managing stress before it becomes a serious issue. The first thing to realise is that stress is a state of mind, an attitude that can be debilitating or encouraging. Nervousness is normal and can add to performance on Test Day, but being overly nervous can be debilitating on the same day.

188

Strategy Session 4 Preparing for Test Day

BREAKING FREE TO SCORE HIGHER The Comfort Zone In competition or testing, it is normal to settle into a comfort zone. The comfort zone is the set point in taking practice tests at which students have reached a plateau. In their minds they have preprogrammed themselves to never go higher, and as a result they can sabotage their own performance. Most students are already capable and trained to perform better, but get caught up in their comfort zone. Freeing yourself from the comfort zone can lead to almost immediate gains in performance.

Fear of Perfection Culturally, we have a fear of perfection. When we are confronted with a perfect score on a section of a practice test, we are inclined to believe there is some mistake. Why isn’t it the reverse, where mistakes are the aberration? Many times it is simply because we have allowed ourselves to have an out or an excuse.

Not Performing as Well as Your Best Performance We have a tendency to believe that a strong performance is a fluke (see above). Realise that you can always have a bad day, but you can never have a better than your best day. That means even if you performed well at one point, that is only a starting place, not the best—it is likely you can do even better. But if your performance has decreased since then, then it is a block and not a reflection of your ability. You are capable on Test day of at least your best performance on practice exams. ·· Visualisation and Imaging are essential. At every success it is necessary to visualise and remember what that success feels like. How it felt to you as you took the test. If you had a bad day, it is important to move past it, understand what happened but not dwell upon it lest you programme yourself for failure. Visualise strong performances. Remember the feeling of success. ·· Maintain concentration through practise. ·· Image the ideal Test Day and reinforce your success with positive self-talk.

Peaking Too Early Intelligent people have a tendency to be unable to study right before the test. This is often because they are subconsciously afraid of failure and need to give themselves an excuse in case something goes wrong. ·· Get over it. Such behaviour is self-destructive and is self-imposed. It is important to take a breather and rest, but letting go right now is self-defeatist. Steel yourself and take it through the final stretch. If something does go wrong, it is something you couldn’t control. Don’t let this be the thing that stops your success. ·· You were meant to be GREAT doctors, not just doctors. You have the training necessary to do it. Don’t let the mind game keep you from your dream.

189

Strategy Session 4 Preparing for Test Day

WHAT TO DO UNTIL A WEEK BEFORE THE TEST ·· Work on your weak areas. ·· Build endurance. ·· Push your ‘comfort zones’. ·· Review any GCSE science content you may have forgotten.

STRESS MANAGEMENT EXERCISE While Studying

1) To help manage BMAT-related stress, it is important to make a list of all the things that are causing you stress.

2) Take the list and separate them into things you can address now and things that you cannot. 3) Then take the list of things you can address now and prioritise it into things that can wait until after Test Day and what cannot wait.

4) Then take the list of things that cannot wait and prioritise those into what that can easily be taken care of and what will take longer.

5) Take care of what you can and then return to studying, something that cannot wait and needs to be done. 6) Take your studying in bite-sized chunks and achieve ‘little’ goals one at a time to build on success.

BMAT TEST TIP Everyone who takes the BMAT finds it a stressful experience. You can prepare to manage your stress on Test Day—minimising stress will help you maximise your score.

190

Strategy Session 4 Preparing for Test Day

WHAT TO DO A WEEK BEFORE THE TEST ·· Study your strengths. You’ve been working on your weak areas, now it is time to shore up the strong ones and boost your confidence level. ·· Review questions and explanations of the ones you missed on the Full-Length Exams. Make sure you don’t repeat these mistakes on Test Day. ·· Visit the Test Site. You don’t want to get lost on Test Day. Find out where your loos will be. ·· Get your body on schedule for Test Day. The week before the Test, go to bed at a reasonable hour and get up at the time you’ll be getting up on Test Day. Mimic Test Day habits so that you are not groggy on that day. ·· Get a good night’s sleep the nights before the test. The night-before-the-night-before’s sleep contributes greatly to the ‘day-of’s’ performance.

WHAT TO DO THE DAY BEFORE THE TEST ·· Relax! By preparing with Kaplan, you’re in great shape for the BMAT. It is normal not to feel like you got to everything. NO ONE can get to everything. Just remember that they are all testing a basic concept that you HAVE studied. ·· Have a quiet evening—have a relaxing dinner, see a movie (not the late show), visit friends. Again, relax and rest up for the big day. ·· Get together materials you need to bring with you on Test Day: (Bring No. 2 pencils, black pens (for the Writing Task), identification (see BMAT Registration Announcement for ID requirements), a non-beeping watch or timer, snacks (simple sugars, complex carbs), tissues, analgesics and an antacid. ·· Eat a lot of carbohydrates. As with any big event, have a good reserve of energy the day of the test, it will also help you sleep. Don’t do this every day, just the day before the BMAT. ·· Get to bed early.

191

Strategy Session 4 Preparing for Test Day

TEST DAY ·· Wake up early. You don’t want to have to rush to get there on time (if you have been getting your body accustomed to getting up early, this will be easy). ·· Read something during breakfast. You want to jump-start your brain so you don’t start the test cold. ·· Eat a good breakfast. However, don’t eat a bigger breakfast than you’re used to; you don’t want to be sluggish. Also, watch the caffeinated drinks (however, if you have not quit already, this is not the time to start). One serving of carbohydrates for quick energy, one serving of protein so you won’t get as hungry and a banana for potassium. ·· Wear comfortable clothing and dress in layers. You don’t know how hot or cold the room will be. ·· Bring high-energy foods for snacks (e.g. apples, bananas, nuts, granola, etc.). Complex carbohydrates are best. Avoid sugary snacks; your insulin will kick in and you’ll come off the ‘sugar buzz’ real fast. Sport drinks are good because the concentration and pH levels are similar to blood and you won’t need to go to the toilet as much. ·· Arrive at the test centre early. Review your Kaplan materials in line to jump-start your brain before the test. Bring headphones so that you do not have to talk to people. Crisis Prevention is key. ·· Remember to bring all your testing materials (see previous page).

192

Strategy Session 4 Preparing for Test Day

DURING THE TEST ·· Answer every question on the test. There’s no penalty for an incorrect answer choice. Eliminate answers and guess if necessary. And remember, you don’t have to get every question right to get a great score. ·· Grid appropriately. Make sure you grid one page at a time rather than back and forth. This will aid in your ability to keep to the correct number and save time. ·· Maintain focus. Don’t dwell on what’s in the past. If you have trouble keeping focus, physically put down your pencil and take a breath, the 5 seconds you use to do that far outweighs the time lost. ·· Remember to triage each section. You want to leave the hardest questions for last. Do all the problems in order except the ones that you know will challenge you. ·· Don’t get bogged down on questions and lengthy calculations. You can always come back to a difficult question after you’ve racked up all the points on other questions. Often, questions that appear to require calculations can be solved through some basic thinking. ·· During the exam, don’t worry about how you’re scoring. Focus on the question in front of you and not what happened in the last section or what might happen in a later section. Concentrate on each question and problem, and use the Kaplan strategies that you’ve worked hard to master. ·· Don’t let others intimidate you or discourage you by their words or actions. Pay no attention to people’s nervous chatter during breaks. If your neighbour says something was ‘easy’ or finishes early, don’t worry. Finishing early doesn’t give you marks; correct answers do. Different test forms may be administered, so your neighbour will probably have a different test form to you—with easier or harder questions. Distractions will hurt other people, you are Kaplan-trained, you will tune out those distractions.

AFTER THE TEST ·· Celebrate your success. ·· Email us on [email protected], to let us know how you scored.

All of us at KAPLAN wish you the very best of luck on Test Day!

193